Express 1B (FWS)
Express 1B (FWS)
Express 1B (FWS)
Class Activity 1
Objective: To discuss and explain how ratios are used in everyday life.
Questions
1. There are some 500 ml cans of red paint and white paint. When n can of red paint is mixed with m cans of white paint
to give pink paint, we say that the mixing ratio of red paint and white paint is n : m.
(a) If the mixing ratio of red paint to white paint is 1 : 3, what is the fraction of red paint in the mixture?
1
=
1+3
1
=
4
(b) Paint P is obtained by mixing 2 cans of red paint to 4 cans of white paint. Paint Q is obtained by mixing 3 cans of
red paint to 9 cans of white paint.
(i) Calculate the mixing ratio of red paint to white paint in paints P and Q.
(ii)
Compare the fractions of red paint in paints P and Q. Which paint will result in a lighter pink? Explain your
answer.
1
Fraction of red paint in paint P =
3
1
Fraction of red paint in paint Q =
4
1 1
Since < , paint Q will result in a lighter pink.
4 3
2. The diagram shows a gear system. The rotation of the driving wheel produces the rotation of the
driven wheel through the interlocking teeth. The ratio of the number of teeth on the driven wheel
to that of the driving wheel is called the gear ratio of the system.
(a) If the number of teeth on the driven wheel is 8 and the number of teeth on the driving wheel
is 16, driven
(i) what is the gear ratio? wheel
33
(ii)
what would be the number of revolution of the driven wheel when the driving wheel rotates one revolution?
Explain your answer.
When the driving wheel rotates 1 revolution, 16 teeth contacts with the driven wheel, the number of revolutions of the driven wheel
= 16 ÷ 8
=2
(b) A mechanic wants to have a gear system such that the driven wheel rotates 4 times when the driving wheel rotates
once. How would you design for him? Discuss your design with your peers.
Consider a driven wheel with 8 teeth. Since the driven wheel rotates 4 times when the driving wheel rotates once, the number of teeth on the
driving wheel
=8×4
= 32
(c) Find something with 2 gears in real life and determine its gear ratio.
Note: The concept of gear ratio is used in making the transmission system of motor cars and bicycles.
A fixed gear bike uses 2 gears. For a fixed gear bike with 45 teeth on the driving wheel and 15 teeth on its driven wheel, the gear ratio
45
=
15
=3
3. Some scientists have reported that tongue rolling ability (able to hold tongue in a tongue roller
U shape) may be due to genes.
(a) Try to find the number of tongue rollers and the number of non-tongue rollers
in your class.
(b) What is the genetic ratio of the number of tongue rollers to the number of
non-tongue rollers in your class?
(c) If your school has 1200 students, what is the estimated number of tongue
rollers in your school?
Note: Genetic ratio is used in breeding plants and animals. non-tongue roller
4. The aspect ratio of an image is the ratio of the width to the height of the image.
width
The designs of televisions, monitor screens and cinema screens are based on certain
aspect ratios. The popular aspect ratios are 16 : 9 and 4 : 3.
(a) Measure the widths and heights of your television screen and computer screen
height
at home. Calculate the aspect ratios of your television and computer screens.
(b) Compare your calculated aspect ratios with your classmates. What do you
realise?
Questions
1. An exchange rate is the current market price for which one currency can be exchanged for another. On 5 July 2011, 500
US dollars (USD) could be exchanged for 613.30 Singapore dollars (SGD).
(a) Express the exchange rate in number of Singapore dollars to one US dollar.
(b) The price of a book in an online bookstore is USD 40. If you bought this book on 5 July 2011, how much would
you have to pay in terms of Singapore dollars? Show your workings.
(c) Describe some occasions where the use of exchange rate is necessary.
Buying or selling a product that is in a different currency from what you use requires using an exchange rate to convert the cost of the product from
2. When you deposit money in a bank, the bank will pay you a fee, called interest, for the use of money you have deposited.
The interest in one year expressed as a percentage of the deposit is called annual interest rate. The amount you would
receive after a year depends on the annual interest rate. If a sum of $4000 deposited in a bank for 1 year can earn $120
interest,
120
the annual interest rate = × 100% = 3%.
4000
(a) If you deposit $6000 in a bank for 1 year at the annual interest rate of 3%, how much interest you will get? Show
your working clearly.
Interest
= 3% of $6000
3
=$ × 6000
100
= $180
(b) If the annual interest rate is 4% and you would like to earn $200 interest in a year, how much should you deposit?
Show your working.
4% of $x = $200
4x
= 200
100
The deposit required is $5000.
35
(c) Do a research and get some interest rates for the deposit in a bank. If you were an employee of the bank, how would
you explain to your clients the different deposit options available based on their interest rates?
3. If you earn income in Singapore over a certain amount, you have to pay the income tax to the government in each year.
The income tax is calculated based on a tax rate table. The table shown below is part of the tax rate table for the year of
assessment 2012 for the total income in 2011.
Note that the rates in the table may vary in each year, depending on the current economic conditions. The chargeable
income is the total income minus allowable deductions, approved donation and personal reliefs.
The table on the previous page shows that if your annual chargeable income is not more than $20 000, you do not need
to pay the income tax.
If a person’s chargeable income in 2011 was $53 000,
then for the first $40 000, the gross tax payable = $550,
for the next $13 000, the tax rate = 7%,
the gross tax payable = $13 000 × 7%
= $910,
\ the income tax = $550 + $910
= $1460.
(a) The chargeable income of Ali in 2011 was $68 000. Calculate
(i) his income tax,
For the first $40 000, the gross tax payable = $550,
for the next $28 000, the tax rate = 7%,
the gross tax payable = $28 000 × 7%
= $1960
the income tax = $550 + $1960
= $2510.
(ii)
the ratio of his income tax to his chargeable income, giving your answer in the form 1 : r, where r is correct
to 3 significant figures.
For the first $30 000, the gross tax payable = $200,
for the next $6 000, the tax rate = 3.5%,
the gross tax payable = $6 000
= $210
the income tax = $200 + $210
= $410.
(ii)
the ratio of her income tax to her chargeable income, giving your answer in the form 1 : r, where r is correct
to 3 significant figures.
Usain Bolt holds the world record time of 9.58s for 100m.
Speed of record holder
100
= m/s
9.58
= 10.4 m/s (correct to 3 s.f.)
(b) The speed of a car is 60 km/h. How would you find its time taken to travel 40 km?
(c) If the speed of the car in (b) increases, would its travelling time taken to travel a distance of 40 km increase or
decrease? Explain your answer.
The distance remains the same. Since the speed of an object is its distance covered per unit time, an increase in speed means that the object
covers a larger distance per unit time. Hence the time taken will decrease.
5. The rate of rotation of a wheel is usually expressed as the number of rotations per minute (rpm).
(a) A wheel P makes 252 rotations in 9 minutes. Find its rate of rotation.
(b) Another wheel Q makes 270 rotations in 10 minutes. Determine which wheel spins faster? Explain your answer.
37
(c) Discuss how the rate of rotation of the wheels of a bicycle affects the speed of the bicycle.
• One revolution covers a distance of C, the circumference of the wheel where C = 2�r.
• As the rate of rotation increases, the number of revolutions increases, covering a larger distance per unit time, thereby increasing the speed of
the bicycle.
• Similarly, as the rate of rotation decreases, the number of revolutions decreases, covering a smaller distance per unit time, thereby decreasing
6. Find out some other examples of the use of rates in our daily life.
Some other examples of the use of rate in our daily life include speed (e.g. 50 km/h), hourly wage rate (e.g. $7/h) and typing speed (e.g. 100 words
per minute).
Class Activity 3
Objective: To find and compare the speeds of bicycles, cars, trains, aeroplanes and spaceships.
Questions
1. Have you ever wondered how fast you walk? How can we find out our own walking speed?
distance
Since speed = , we can measure our own walking speed by walking for a particular distance and taking note of the time.
time
2. In the Cycle Singapore 2011 Super Challenge 60 km race, the record time for the men’s event was 1 hour 15 minutes and
38.4 seconds.
= 47.59
= 47.6 km/h (correct to 3 s.f.)
(b) Based on your NAPFA Test 2.4 km run, calculate your running speed in km/h.
(c) If you keep running at the speed in (b), how long would you take to run a distance of 60 km?
(c) If you complete the laps by driving a car at the speed in (b) above, how long would you take?
Time taken
5.073 × 61
=
90
= 3 hours 26 minutes 18.12 seconds
(d) In general, how much faster, in km/h, is the speed of a racing car compared to that of a normal car travelling along
the expressway?
157 – 90 = 67
In general, the racing car travels 67 km more in an hour compared to a car travelling along the expressway.
4. The Beijing-Shanghai High-speed Railway is a railway line connecting the cities Beijing and Shanghai in China. A train
can finish the journey in 4 hours.
(a) (i) What information do you need in order to find the speed of the train?
(b) How can you estimate the speed of our MRT train for travelling between two stations? Explain briefly and find your
estimated value together with your classmates.
(c) If an MRT train travels along the above railway from Beijing to Shanghai, how long would it take to arrive at the
destination? How much time can you save when travelling in high-speed train instead?
Travelling in the high-speed train instead of the MRT saves 11.5 hours.
39
5. The flight distance from Beijing to Shanghai is 1070 km.
(a) Given that the flight time of an aeroplane flying from Beijing to Shanghai is 2 hours 15 minutes, find the speed of
the aeroplane.
40
The speed of the aeroplane = 2570 ÷ 3 +
60
= 700.9
= 701 km/h (correct to 3 s.f.)
(b) If you have to travel from Beijing to Shanghai, would you take a high-speed train or an aeroplane? Explain your
choice.
6. The average radius of the Earth is 6371 km. A space shuttle flew in a circular orbit at an altitude of 242 km above the
earth surface. It took 90 minutes for the space shuttle to complete one full orbit.
41
7. White paint and red paint are mixed in the ratio 5 : 3 to 11. The number of $1 coins and the number of 50-cent coins
form pink paint. Find the volume of white paint needed in a purse are in the ratio 2 : 1. If 3 of the $1 coins are
to make 4 litres of such pink paint. exchanged for 50-cent coins, the ratio becomes 1 : 2.
Find the total value of the coins.
Solution
Volume of white paint : Volume of red paint = 5 : 3 Solution
Fraction of the volume of white paint in pink paint Let 2x and x be the number of $1 coins and 50-cent coins
5 respectively.
=
5+3 Number of $1 coins after exchanging = 2x – 3
5 Number of 50-cents coins after exchanging = x + 6
=
8 2x – 3 1
5 =
\ volume of white paint in pink paint = ×4 x+6 2
8
2(2x – 3) = x + 6
= 2.5 litres
4x – 6 = x + 6
3x = 12
8. The volumes of cup P and of cup Q are in the ratio 2 : 3. x = 4
If cup Q can hold 120 cm3 of water more than cup P, \ total value of the coins = $1(2 × 4) + $0.50(1 × 4)
find the volume of cup Q. = $10
Solution
Let 2x be the volume of cup P and 3x be the volume of Section 9.2
cup Q respectively. 12. Find the rate in each of the following.
Then 3x – 2x = 120 (a) Joe ran 100 m in 12.5 seconds. Find his rate of running
x = 120 in m/s.
3x = 360 (b) 375 ml of apple juice cost $6. Find the price rate
The volume of cup Q is 360 cm3. in $/l.
Solution
9. Three boys, X, Y and Z, share 132 copies of computer 100 m
games. The ratio of X’s share to Y’s share is 2 : 3. The (a) Joe’s rate of running =
12.5 s
ratio of Y’s share to Z’s share is 1 : 2. How many copies = 8 m/s
of computer games does each boy get?
$6
(b) Price rate of juice =
Solution 0.357l
X’s share : Y’s share = 2 : 3 = $16/l
Y’s share : Z’s share = 1 : 2 = 3 : 6
∴ X’s share : Y’s share : Z’s share = 2 : 3 : 6
13. On 5 July 2011, 3600 Chinese Yuan (CNY) could be
2
Number of copies that X gets = 132 × exchanged for 683.28 Singapore dollars (SGD).
2+3+6
= 24 (a) Find the exchange rate between the Chinese Yuan
3 and the Singapore dollars in
Number of copies that Y gets = 132 ×
2+3+6 (i) SGD/CNY, (ii) CNY/SGD.
= 36 (b) A Chinese tourist spent 4500 SGD at the Sentosa
6 Resort. Find his expenditure in CNY.
Number of copies that Z gets = 132 ×
2+3+6
Give your answers correct to 4 significant figures.
= 72
Solution
10. The prices of two stocks P and Q are in the ratio of 5 : 7. (a) (i) 3600 CNY = 683.28 SGD
When the price of P is increased by $12 and the price of 683.28
1 CNY = SGD
Q is decreased by $6, the prices of two stocks become 3600
the same. Find the original price of stock P. = 0.1898 SGD
The required exchange rate is 0.1898
Solution SGD/CNY.
Let $5x and $7x be the price of stocks P and Q 3600
respectively. (ii) 1 SGD = CNY
683.28
5x + 12 = 7x – 6 = 5.2687 CNY
2x = 18 = 5.269 CNY (correct to 4 s.f.)
x = 9 The required exchange rate is 5.269
5x = 45 CNY/SGD.
Stock P costs $45.
43
21. Cars A and B leave the same place and travel in the same
direction along a straight road. Car A travels at 60 km/h
and car B travels at 72 km/h. After how long will they
be 8 km apart?
Solution
Let t hours be the required time such that the cars are
8 km apart.
Distance travelled by car A in t hours = 60t km
Distance travelled by car B in t hours = 72t km
From the given information,
72t – 60t = 8
12t = 8
2
t =
3
2
The cars will be 8 km apart after hours.
3
22. The speed limit for a road is 60 km/h. Express this speed
limit in m/s.
Solution
The speed limit = 60 km/h
= 60 × 1000 m/h
1000
= 60 × m/s
3600
2
= 16 m/s
3
1
23. A man walks at a speed of 1 m/s. Express this speed
9
in km/h.
Solution
1
His walking speed = 1 m/s
9
10 1
= × km/s
9 1000
10 3600
= × km/h
9 1000
= 4 km/h
45
(b) x : y : z = 2.7 : 6 : 1.8 (c) a : b =
1
:1
= 27 : 60 : 18 2
1
27 60 18 b : c = 1 :
= : : 3
3 3 3
1 1
= 9 : 20 : 6 ∴ a : b : c = :1:
2 3
1 1
= ×6:1×6: ×6
2 3
6. Given that a : b : c = 20 : 35 : 15,
=3:6:2
(a) simplify a : b : c,
(b) find a : b, (d) a : b = 3 : 7
(c) find c : b. = 9 : 21
b : c = 3 : 7
Solution = 21 : 49
(a) a : b : c = 20 : 35 : 15 ∴ a : b : c = 9 : 21 : 49
20 35 15
= : :
5 5 5
=4:7:3 Level 2
(b) a : b = 4 : 7 9. If a = 20 and b = 36, find
(a) a : b,
(c) c : b = 3 : 7 (b) a2 : b2.
Solution 18. For a concert in Singapore Arts Festival 2012, the price
5 of a Category 1 ticket to that of a Category 2 ticket is
Mass of copper = 4.8 ×
5+3
5 : 4, and the price of a Category 2 ticket to that of a
= 3 kg Category 3 ticket is 2 : 1.
(a) Find the ratio of the price of a Category 1 ticket to
that of a Category 2 ticket to that of a Category 3
14. The numbers of junior and senior staff in a company
ticket.
are in the ratio 2 : 5. If the total number of staff in the
(b) If the price of a Category 3 ticket is $20, find the
company is 28, how many of them are junior staff?
price of a Category 1 ticket.
Solution
Solution
Number of junior staff
(a) Category 1 : Category 2 = 5 : 4
2
= × 28 Category 2 : Category 3 = 2 : 1
7
= 8 =2×2:1×2
There are 8 junior staff in the company. =4:2
Category 1 : Category 2 : Category 3 = 5 : 4 : 2
47
(b) Price of Category 1 ticket Number of students Number of students
20 from School Q : from School R
=$ ×5
2 who participated who participated
= $50 = 9 : 11
= 18 : 22
A category 1 ticket costs $50.
The numbers of students from Schools P, Q, and
R who participated are in the ratio 15 : 18 : 22.
19 . Chicken, potatoes and onions in a recipe are mixed in
Number of students from School P
the ratio 7 : 3 : 2 by mass. If Mrs Krishnan follows the
15
recipe and uses 350 g of chicken, how many grams of = 165 ×
15 + 18 + 22
(a) potatoes,
= 45
(b) onions,
does she use?
(b) Number of students from School Q
18
Solution = 165 ×
15 + 18 + 22
(a) Let x g be the mass of potatoes Mrs Krishnan uses.
= 54
x 3
=
350 7
x = 150 22. The prices of two stocks A and B are in the ratio
She uses 150 g of potatoes. 2 : 3. If the price of A increases by $12 while the price
of B decreases by $6, the ratio of their prices becomes
(b) Let y g be the mass of onions she uses. 10 : 11. Find the original prices of the stocks.
y 2
=
350 7
Solution
y = 100 Let the original prices of Stock A and Stock B be $2x
She uses 100 g of onions. and $3x respectively.
New price of Stock A = $(2x + 12)
20. In a hydrocarbon compound, glucose, the atoms of New price of Stock B = $(3x – 6)
carbon, hydrogen and oxygen are bonded in the ratio 2x + 12 10
1 : 2 : 1. If the total number of atoms in a molecule of =
3x – 6 11
the compound is 24, find the number of 11(2x + 12) = 10(3x – 6)
(a) carbon atoms, 22x + 132 = 30x – 60
(b) hydrogen atoms, 8x = 192
in the molecule. x = 24
2x = 48
Solution 3x = 72
1
(a) Number of carbon atoms = 24 × The original price of Stock A is $48 and the original
1 + 2 +1
price of Stock B is $72.
=6
2
(b) Number of hydrogen atoms = 24 × 23. The total number of English, Mathematics and Science
1 + 2 +1
= 12 books in a book store is 125. The ratio of the number
of English books to that of Mathematics books is 3 : 4.
21. A total of 165 students from schools P, Q and R The number of Science books is 15 less than the number
participated in a National Day Parade. The numbers of of English books. Find the number of English books.
students from schools P and Q who participated are in
Solution
the ratio 5 : 6 and the numbers of students from schools
Q and R who participated are in the ratio 9 : 11. Let 3x and 4x be the numbers of English books and
Find the number of students from Mathematics books respectively.
(a) school P, (b) school Q Number of Science books = 3x – 15
who participated in the National Day Parade. From the given information,
3x + 4x + (3x – 15) = 125
Solution 3x + 4x + 3x – 15 = 125
(a) Number of students Number of students 10x = 140
: x = 14
from School P from School Q
who participated who participated \ Number of English books
= 5 : 6 = 3 × 14
= 15 : 18 = 42
Solution Solution
Let 7x and 2x be the original numbers of technicians and If a = 3k, b = 4k and c = 5k, where k ≠ 0,
apprentices respectively. then a : b : c = 3 : 4 : 5.
1 1 When k = 2, a = 6, b = 8 and c = 10.
7x +
× 2x – 1 – × 2x = 36 When k = 3, a = 9, b = 12 and c = 15.
4 4
1 1
7 x – 1 x = 36
2 2
6x = 36 Exercise 9.2
x = 6 Level 1
\ Original number of technicians = 7 × 6 1. Find the rate in each case.
= 42 (a) A 2-litre bottle of peanut oil costs $7.
The price rate is $ /l .
26. The lengths of rods A and B are 120 cm and 96 cm (b) A company produces 360 vases in 5 days.
respectively. After a rod of equal length is cut from each
rod, the ratio of the length of A to that of B becomes The rate of production is vases/day.
10 : 7. Find the length of the rod cut from A. (c) Huili typed 376 words in 8 minutes.
Solution Her rate of typing is words/min.
Let x cm be the length of the rod cut from A. (d) A carpet of area 3 m costs $240. 2
120 – x 10
= The price rate is $ /m2.
96 – x 7
7(120 – x) = 10(96 – x) (e) The mass of a 2.5-m metal bar is 10 kg.
840 – 7x = 960 – 10x
10x – 7x = 960 – 840 The rate of mass is kg/m.
3x = 120
x = 40 Solution
$7
(a) The price rate =
\ The length of the rod cut from A is 40 cm. 2l
= $3.50/l
27. Chocolate beans are packed in 250 g and 750 g packages.
(b) A company produces 360 vases in 5 days.
The numbers of 250 g packages and 750 g are in the
360 vases
ratio of 1 : 2. If 2 of the 750 g packages are repacked Rate of production =
5 days
into 250 g packages, then the ratio becomes 5 : 3. Find = 72 vases/day
49
(c) Huili typed 376 words in 8 minutes. Level 2
376 words
Rate of typing = 5. The cost of laying 12 m2 of floor tiles is $270.
8 min
(a) Find the cost of laying floor tiles per m2.
= 47 words/min (b) Find the cost of laying 20 m2 of floor tiles.
(d) A carpet of area 3 m2 costs $240.
$240 Solution
Price rate = (a) The price of laying floor tiles per m2
3 m2
= $80/m2 $270
=
12
(e) The mass of a 2.5 m metal bar is 10 kg. = $22.50/m2
10 kg
Rate of mass =
2.5 m (b) The price of laying 20 m2 of floor tiles
= 4 kg/m = $22.50 × 20
= $450
(f) The mass of 12 eggs is 720 g.
720 g
Rate of mass = 6. A pipe delivers 9 l of water in 20 seconds.
12 eggs
= 60 g/egg (a) Find the amount of water it delivers per minute.
(b) An empty aquarium tank is 60 cm long, 45 cm wide
and 30 cm high. How long does it take for the pipe
2. Rahman works 5 days a week and 8 hours a day. His to fill the empty tank completely with water?
daily wage is $184. Note: 1 litre = 1000 cm3.
(a) Find his hourly wage rate.
(b) Find his weekly wage rate. Solution
(a) Amount of water delivered per minute
Solution 9
$184 = × 60 l/min
(a) Hourly wage rate = 20
8h
= 27 l/min
= $23/h
(b) Weekly wage rate = $184 × 5/week (b) Volume of the tank = 60 × 45 × 30 cm3
= $920/week = 81 000 cm3
81 000
= l
1000
3. A car travelled 225 km on 25 litres of petrol. The price
= 81 l
of the petrol consumed was $30.
(a) Find the rate of petrol consumption in km/litre. 81
The required time taken =
27
(b) What was the price of petrol per litre?
= 3 min
Solution
225 km
(a) Rate of petrol consumption = 7. A 5-kg pack of ‘Fragrant’ brand rice is sold for $9.60
25 l
and a 10-kg pack of ‘Premium’ brand rice is sold for
= 9 km/l
$18.60.
$30 (a) Find the price per kg of the
(b) The price of petrol per litre =
25 l
(i) ‘Fragrant’ brand rice,
= $1.20/l (ii) ‘Premium’ brand rice.
(b) Which pack of rice is more value for money if the
4. There are 6 wooden cubes, each of side 5 cm. Their total qualities of both brands of rice are the same?
mass is 525 g. Find the mass per
(a) cube, Solution
(b) cm3 of cube. (a) (i) Price per kg of ‘Fragrant’ brand rice
9.60
=$
Solution 5
525 g = $1.92
(a) Mass per cube =
6 cubes (ii) Price per kg of ‘Premium’ brand rice
= 87.5 g/cube 18.60
=$
10
(b) Volume of a cube = 53
= 125 cm3 = $1.86
87.5 g (b) Since $1.86 < $1.92, the ‘Premium’ brand rice is
Mass per cm3 = more value for money.
125 cm2
= 0.7 g/cm3
51
x(300) + 2.5x(100) = 4400 Solution
300x + 250x = 4400 (a) Property tax = $30 000 × 10%
550x = 4400 = $3000
x = 8 (b) $30 000 = $6000 + $24 000
The monthly rental rate of the warehouse is $8/m2. For the first $6000, the property tax = $0
For the next $24 000,
14. On a day, a bank had the following exchange rates: the property tax = $24 000 × 4%
1 SGD = 0.8163 USD and 1 USD = 0.6877 EUR. = $960
(a) Find the exchange rate between the Singapore \ the required property tax
dollars (SGD) and the Euros (EUR) in EUR/SGD. = $0 + $960
(b) On the Internet, a watch was sold for 430 EUR at = $960
site A in Germany and 600 USD at site B in United
States.
16. Write down two examples of average rates in daily life.
(i) Find the selling price of the watch at site A
Discuss these examples with your classmates.
in SGD.
(ii) Find the selling price of the watch at site B Solution
in SGD. Two examples of average rates in daily life are the
(iii) Which site offered a cheaper price for the average rate of rainfall and the average interest rate a
watch? bank charges.
Solution
(a) 1 SGD = 0.8163 USD
= 0.8163 × 0.6877 EUR
Exercise 9.3
= 0.56137 EUR Level 1
= 0.5614 EUR (correct to 4 s.f.) 1. Copy and complete the following table.
\ The exchange rate is 0.5614 EUR/SGD.
Distance Time Average
(b) (i) Selling price at site A travelled taken speed
= 430 EUR
(a) 64 km 2h 32 km/h
1
= 430 × SGD (b) 200 m 25 s 8 m/s
0.56137
= 765.98 SGD (correct to 2 d.p.) (c) 66 km 3h 22 km/h
(ii) Selling price at site B (d) 35 m 7s 5 m/s
= 600 USD (e) 75 km 1.5 h 50 km/h
1
= 600 × SGD (f) 39 m 13 s 3 m/s
0.8163
= 735.02 SGD (correct to 2 d.p.) Solution
64
(iii) Site B offered a cheaper price. (a) Average speed = = 32 km/h
2
200
(b) Average speed = = 8 m/s
15. The annual value of an apartment is $30 000. 25
(a) Find the property tax of the apartment if the tax (c) Distance travelled = 22 × 3 = 66 km
rate is 10%.
(b) With effect from 1 January 2011, if the property is (d) Distance travelled = 5 × 7 = 35 m
occupied by the owner, the property tax is charged 75
(e) Time taken = = 1.5 h
by a progressive owner-occupier’s tax rates as 50
follows: 39
(f ) Time taken = = 13 s
3
Annual Value ($) Tax Rate (%)
First 6000 0 2. Convert the following speeds to m/s.
Next 59 000 4 (a) 24 km/h
Amount exceeding 65 000 6 (b) 50 km/h
(c) 80 km/h
Find the property tax of the apartment according (d) 126 km/h
to the given tax rates.
53
9. Jack walks at an average speed of 4 km/h for one hour. 12. The distance between Singapore and Kuala Lumpur is
He then cycles 6 km at 12 km/h. Find his average speed 315 km. The average speeds of a car and an aeroplane
for the whole journey. from Singapore to Kuala Lumpur are 60 km/h and
900 km/h respectively. Find, for a trip from Singapore
Solution to Kuala Lumpur,
Total distance travelled = 4 × 1 + 6 (a) the time taken by the car and the aeroplane,
= 10 km (b) the ratio of the time taken by the car to that by the
aeroplane.
6
Total time taken = 1 + = 1.5 h
12
Solution
10 (a) Time taken by car in hours
Average speed =
1.5 315
2 =
= 6 km/h 60
3 1
=5
4
10. Mr Tan ran from his home to a provision shop 400 m Time taken by aeroplane in hours
away at an average speed of 2 m/s. He walked back to 315
=
his home at an average speed of 1 m/s. Find his average 900
speed for the whole journey if 7
=
(a) he did not stay in the shop, 20
(b) he stayed in the shop for 5 minutes. (b) Time taken by car : Time taken by aeroplane
1 7
Solution =5 :
4 20
400 400
(a) Total time taken = + =
21 7
:
2 1
4 20
= 600 s 21 7
= 20 × : 20 ×
2 × 400 4 20
Average speed =
600
= 105 : 7
1
= 1 m/s
3 105 7
= :
7 7
(b) Total time taken = 600 + 5 × 60
= 900 s =15 : 1
2 × 400
Average speed =
900 13. The flight distance between Singapore and Sydney is
8 about 6316 km. A plane takes 7 h 20 min to fly from
= m/s
9 Singapore to Sydney.
(a) Find the average speed of the plane from Singapore
to Sydney. Give your answer correct to the nearest
Level 3 km/h.
11. Sound travels at 340 m/s in air. (b) Sydney’s time is 2 hours ahead of Singapore’s time.
(a) Find the distance travelled by sound in air in If the plane departs from Singapore at 09 55, find
7 seconds. Give your answer correct to the nearest its time of arrival in Sydney.
0.1 km.
(b) A boy clapped his hands near a cliff. He could Solution
hear the echo of his claps 3 seconds later. Find his (a) Average speed of the plane
distance from the cliff. 6316
=
71
3
Solution
= 861 km/h (correct to the nearest km/h)
(a) Distance travelled in 7 s
= 340 × 7 m (b) 7 h 20 min = 7 h (5 + 15) min
= 2380 m Time of arrival in Sydney
= 2.4 km (correct to the nearest 0.1 km) = 11 55 + 7 h 20 min
3 = 19 15
(b) Distance from the cliff = 340 × m
2
= 510 m
Solution
Total time taken = 2 + 3
=5h
55
Revision Exercise 9 4. (a) Simplify each of the following ratios.
1 2
1. Two cubes are of sides 6 cm and 8 cm respectively. Find (i) a:b=1 :2
2 5
the ratio of
(a) their sides, (ii) b : c = 0.105 : 0.350
(b) their areas on one face, (b) Find the ratio a : b : c using the data in (a).
(c) their volumes. (c) Alan, Bob and Cathy share $500 in the ratio
a : b : c found in (b). Find Alan’s share correct to
Solution 2 decimal places.
(a) Ratio of their sides = 6 : 8
=3:4 Solution
1 2
(b) Ratio of their areas = 6 : 8 2 2 (a) (i) a : b = 1 : 2
2 5
= 9 : 16 3 12
= :
3 3 2 5
(c) Ratio of their volumes = 6 : 8
= 27 : 64 = 15 : 24
= 5 : 8
2. There are a total of 240 sheets of $5 notes and $10 notes. (ii) b : c = 0.105 : 0.350
The numbers of $5 and $10 notes are in the ratio 3 : 2. = 105 : 350
Find = 3 : 10
(a) the number of $5 notes, (b) a : b = 5 : 8
(b) the number of $10 notes, = 15 : 24
(c) the ratio of the values of the $5 notes and $10 notes. b : c = 3 : 10
= 24 : 80
Solution
3 ∴ a : b : c = 15 : 24 : 80
(a) Number of $5 notes = 240 × = 144
3+2 15
(c) Alan’s share = $500 ×
2 15 + 24 + 80
(b) Number of $10 notes = 240 × = 96
3+2 15
= $500 ×
119
(c) Ratio of values of $5 notes and $10 notes
= $63.03 (correct to 2 d.p.)
= 144 × 5 : 96 × 10
= 3 : 4
5. The monthly salaries of Paula and Tom are in the ratio
of 5 : 9. The monthly salary of Tom is $3200 more than
3. A metal bar of mass 3.6 kg is cut into two pieces in the
that of Paula. Find the monthly salary of Paula.
ratio 3 : 5. The length of the shorter piece is 45 cm. Find
(a) the length of the longer piece,
Solution
(b) the length of the original metal bar,
Let $5x and $9x be the monthly salaries of Paula and
(c) its rate of mass per unit length in kg/m,
Tom respectively.
(d) the mass of the shorter piece.
9x – 5x = 3200
4x = 3200
Solution
x = 800
(a) Let x cm be the length of the longer piece.
\ monthly salary of Paula
45 3
= = $800 × 5
x 5
5 = $4000
x = 45 ×
3
= 75 6. Mrs Lim deposits $3800 in a bank at the interest rate of
The length of the longer piece is 75 cm. 3% per annum.
(a) Find the simple interest if the time of deposit is 4
(b) Length of original metal bar = 45 + 75 years.
= 120 cm (b) If the simple interest is $171, find the time of
3.6 kg deposit.
(c) Rate of mass per unit length =
1.2 m
= 3 kg/m Solution
45 (a) The required simple interest
(d) Mass of the shorter piece = 3 ×
100 = $3800 × 3% × 4
= 1.35 kg = $456
1 = $12/l
The time of deposit is 1 years.
2
(c) x : y = 7.5 : 12
= 75 : 120
7. The annual value of a shop is $36 000. If the property
= 5 : 8
tax rate of the shop is 10%, find the amount of property
tax. (d) Peanut oil is cheaper. It is a better buy.
2l
Solution (e) Consumption rate =
16 days
Amount of property tax 1
= $36 000 × 10% = l/day
8
= $3600 1 1
(f ) Required number of days = 2 ÷
2 8
8. On a day, the exchange rate between the Singapore = 20
dollars (SGD) and the US dollars (USD) is 1
1 SGD = 0.816 USD, correct to 3 decimal places. The 10. A man took 4 hours to drive 360 km from Singapore
2
exchange rate between the Australian dollars (AUD) to Kuala Lumpur. He used 37.5 litres of petrol for the
and the US dollars is 1 AUD = 1.073 USD, correct to entire journey.
3 decimal places. (a) Find his average speed.
(a) Find the exchange rate, correct to 3 decimal places, (b) Find the petrol consumption rate in km/l.
between Singapore dollars and Australian dollars (c) He drove at an average speed of 110 km/h on a
in AUD/SGD. highway for 2 hours during his journey. Find his
(b) Mr Lim has a son studying in a university in average speed for the remaining part of his journey.
Sydney. He has to remit 25 000 AUD to him. Find
the required amount of Singapore dollars correct
Solution
to the nearest dollar. 360
(a) Average speed =
41
Solution 2
57
11. A car starts from rest. After travelling 125 m in 10 s, (b) (i) Average speed for the return journey
its speed picks up to 25 m/s. It travels at this speed for 120
=
20 seconds. Then brakes are applied. The car stops in 21
6
6 seconds and the braking distance is 95 m.
(a) Express the speed 25 m/s in km/h. 5
= 55 km/h
(b) Find the average speed of the car during the period 13
at which its speed increases. (ii) Average speed for the whole trip
(c) Find the average speed of the car during the period
120 + 120
the brakes were applied. =
2 + 21
(d) Find the average speed of the car for the whole 6
journey. 6
= 240 ×
25
Solution = 57.6 km/h
3600
(a) 25 m/s = 25 × km/h
1000
= 90 km/h
125
(b) Required average speed =
10
= 12.5 m/s
95
(c) Required average speed =
6
5
= 15 m/s
6
Solution
(a) Let x km/h be the required average speed.
40 40
54 × +x2– = 120
60 60
4
36 + x = 120
3
4
x = 84
3
x = 63
His average speed for the remaining journey is
63 km/h.
Class Activity 1
Objective: To understand the meaning of percentage in some daily life examples and make connections between percentages
and fractions/decimals.
Questions
1. The percentage of oxygen in dry air by volume is about 21%.
(b) Find the volume of air that a normal man inhales in each time.
2. In the 2011/12 NBA season, the basketball player Jeremy Lin had a field goal percentage of 44.6%.
Field goal percentage is the ratio of number of field goals made to the number of field goals attempted.
(b) If the total number of shots attempted by Jeremy was x in the season, what would be the number of goals he made?
Express your answer as a decimal of x.
number of goals made
= 44.6%
number of goals attempted
number of goals made 44.6
=
x 100
number of goals made = 0.446x
3. On 13 April 2012 news, SMRT announced that “To date, about 245 buses or 23 per cent of SMRT's bus fleet were
wheelchair accessible.”
Rephrase the above statement using fraction.
23
To date, about 245 buses or of SMRT’s bus fleet were wheelchair accessible.
100
59
4. A club had 13 800 members in the year 2012. Mr X and Mr Y contested in the chairman election in that year. Total valid
votes polled for Mr X was 7289 and that for Mr Y was 6103. Mr X won by obtaining 54.4% of the votes.
7289
× 100%
7289 + 6103
= 54.4%
Questions
The mass of Tim is 60 kg. The mass of Fred is 5% more than that of Tim.
Mass of Fred
= 105% × 60 kg
105
= × 60 kg
100
= 63 kg
2. By how many percent is the mass of Tim less than that of Fred?
63 – 60
× 100%
63
= 4.76%
3. Discuss whether the statement ‘If A is 5% more than B, then B is 5% less than A’ is correct or not.
The statement is incorrect. The correct statement should be ‘If A is 5% more than B, B is 4.76% less than A.’
4. Illustrate by an example to show whether the statement ‘If A is 20% less than B, then B is 20% more than A’ is correct
or not.
Consider the height of two plants. Plant A is 80 cm tall while plant B is 100 cm tall.
100 – 80
× 100% = 20%
100
Height of plant A is 20% less than height of plant B.
100 – 80
× 100% = 25%
80
Height of plant B is 25% more than height of plant A.
Therefore, the statement ‘If A is 20% less than B, then B is 20% more than A’ is incorrect.
The correct statement should be ‘If A is 20% less than B, then B is 25% more than A’.
61
Class Activity 3
Objective: To work out the calculation of GST, service charge and discount on the receipts.
Questions
1. Refer to Receipt 1 from a restaurant.
The subtotal is the sum of all the prices listed above it.
(b) A 10% service charge (S.C.) is included in the bill. How is the 10% service charge calculated?
The GST is based on the sum of the subtotal amount and the service charge.
i.e. GST = $(26.40 + 2.64) × 7%
= $2.03 (correct to 2 d.p.)
The purpose of the rounded amount is to make the total amount end to the nearest 5 cents.
(e) If you are the manager of the restaurant, how would you explain to a customer who queries how the total amount
of $31.05 is obtained?
The subtotal amount is equal to the sum of the prices of the items above it minus the discount.
i.e. subtotal amount
= $(5.00 + 20.00 + 7.00 + 5.00 + 518.00 – 77.70)
= $477.30
The total amount is the sum of the subtotal amount and the GST, rounded to the nearest 10 cents.
i.e. total amount = $477.30 + $33.41
= $510.71
= $510.70 (correct to the nearest 10 cents)
(a) What is the relationship between the subtotal and total amounts in this case?
The total amount is obtained by rounding the subtotal amount correct to the nearest 10 cents.
i.e total amount = $7.42 – $0.02
= $7.40
(b) There are three figures on a row indicating GST rate, amount before GST and GST amount. Are you able to explain
how they are calculated? Show your working clearly.
63
Try It! 4. 650 people attended a family-day function organised by
a community centre. If 24% of them were men, 32% of
Section 10.1
them were women and the rest were children, find the
1. A company has 50 employees and 34 of them are women. number of
Find the percentage of (a) men,
(a) female employees, (b) women,
(b) male employees. (c) children,
Solution attending the function.
34
(a) Percentage of female employees = × 100%
50
Solution
= 68% (a) Number of men = 650 × 24%
(b) Percentage of male employees = 100% – 68% = 156
= 32% (b) Number of women = 650 × 32%
= 208
2. Alan’s height is 135 cm and Suling’s height is 150 cm.
(c) Number of children = 650 – 156 – 208
Express = 286
(a) Alan’s height as a percentage of Suling’s height,
(b) Suling’s height as a percentage of Alan’s height.
5. Tank A has 20 goldfish and tank B has 25 goldfish. Both
Solution tanks have the same number of goldfish with short tail
Alan’s height fin. If the percentage of goldfish with short tail fin in
(a) The required percentage = × 100%
Suling’s height tank B is 36%, find the percentage of goldfish with short
135 tail fin in tank A.
= × 100%
150
= 90% Solution
Number of short tail fin goldfish in tank B
Suling’s height
(b) The required percentage = × 100% = 36% × 25
Alan’s height
= 9
150
= × 100% Percentage of short tail fin goldfish in tank A
135
1 9
= 111 % = × 100%
9 20
= 45%
3. The following table shows the test results of 3 groups
of students.
Total number Number of Section 10.2
Group
of students students who pass 6. Kumar was fined $187 while driving. If the fine was
A 20 13 110% of his daily wage, find Kumar’s daily wage.
B 24 15 Solution
C 25 16 Let $w be Kumar’s daily wage.
w × 110% = 187
Which group of students has the highest pass percentage? 187
w =
1.1
Solution
Percentage of pass in group A = 170
13 Kumar’s daily wage is $170.
= × 100%
20
= 65% 7. Bob’s mass is 25% more than that of Ada. Carl’s mass is
Percentage of pass in group B 5% less than that of Bob. If the total mass of Ada, Bob
15
= × 100% and Carl is 165 kg, find Bob’s mass.
24
= 62.5% Solution
Percentage of pass in group C Let x be Ada’s mass.
16 Ada’s mass = x kg
= × 100%
25 Bob’s mass = 125% of x kg
= 64% = 1.25x kg
Hence, group A has the highest percentage of students Carl’s mass = 95% of 1.25x kg
who passes. = 1.1875x kg
9. There are 40 students in a class. In term 1, 25% of His height at the beginning of the year was 125 cm.
students got grade A in Mathematics. In term 2, 30% of
students got grade A in Mathematics. Find the percentage 12. The water level in a tank decreased from 30 cm to
increase in the number of students who got grade A in 25.5 cm. Find the percentage decrease in the water level.
Mathematics.
Solution
Solution Decrease in height of water level = 30 – 25.5
Number of students who got grade A in term 1 = 4.5 cm
= 25% × 40
= 10 Percentage decrease in height of water level
4.5
= × 100%
Number of students who got grade A in term 2 30
= 30% × 40 = 15%
= 12
Increase in number of students who got grade A 13. The volume of air in a balloon is 1600 cm3. If 12.5% of
= 12 – 10 air is released, find the new volume of air in the balloon.
= 2
Solution
Percentage increase in the number of students who got
New volume
grade A
= Original volume × (100% – Percentage decrease)
2
= × 100% = 1600 × (100% – 12.5%)
10
87.5
= 20% = 1600 ×
100
= 1400
10. A shop sold 250 CDs last week. The sales of CDs
increased by 16% this week. How many CDs were sold
14. Mrs Li’s mass dropped to 62 kg after her mass decreased
this week?
by 7%. What was her original mass? Give your answer
correct to 3 significant figures.
Solution
Method 1: Solution
Increase
Percentage increase = × 100% Let Mrs Li’s original mass be m kg.
Original value
62 = m × (100% – 7%)
Increase = 250 × 16% 93
= 40 = m ×
100
62
Sales volume for this week = (250 + 40) m = = 66.7 (correct to 3 s.f.)
0.93
= 290 CDs
Mrs Li’s original mass was 66.7 kg.
65
15. The number of girls in a dancing club is twice as many 570
= M × (100% – 24%)
as the number of boys. If the number of girls decreased 76
= M ×
by 20%, by what percentage must the number of boys 100
be increased so that there is an equal number of boys 100
M = 570 ×
and girls in the club? 76
= 750
Solution The marked price of the TV set is $750.
Let x be the number of boys.
(b) The new selling price
Number of girls = 2x
= $750 × (100% – 15%) × (100% – 9%)
Number of girls after 20% decrease
= $750 × 0.85 × 0.91
= 2x × (100% – 20%)
= $580.125
= 1.6x
Increase in the selling price
Required percentage increase in number of boys = $580.125 – $570
1.6x – x = $10.125
= × 100%
5 = $10.13 (correct to 2 d.p.)
= 0.6 × 100%
= 60%
19. A plumber charges $20 for a job excluding GST. If the
GST rate is 7%,
(a) how much is the GST for the service provided?
Section 10.4 (b) how much does a customer have to pay for the
16. During a sale, a watch was sold for $784. If the marked service provided inclusive of GST?
price of the watch is $980, find the percentage discount.
Solution
Solution (a) GST
= Service charge GST rate
Discount
= $980 – $784 = $20 × 7%
= $196 = $1.40
196
Percentage discount = × 100% (b) Total amount = Service charge + GST
980
= $20 + $1.40
= 20%
= $21.40
17. The marked price of a mobile phone is $380. If the phone 20. The marked price, inclusive of GST, of a vacuum cleaner
is sold at a discount of 25%, find in a shop is $214. The GST rate is 7%.
(a) its discount, (b) its selling price. (a) Find its price before GST.
(b) Find the amount of GST levied on it.
Solution (c) If the vacuum cleaner is sold at 10% discount, find
(a) Discount
= $380 × 25% the decrease in the GST levied.
= $95
Solution
(b) Method 1:
(a) Price before GST
Selling price = $380 – $95
214
= $285 =$
1.07
Method 2: = $200
Selling price
= Marked price × (100% – Discount %) (b) GST = Marked price – Price before GST
75 = $214 – $200
= $380 × = $285 = $14
100
(c) Selling price of vacuum cleaner
18. A TV set is sold at $570 after a 24% discount. = $214 × (100% – 10%)
(a) Find the marked price of the TV set. = $192.60
(b) If the TV set was sold at a 15% discount and then GST levied
at a further 9% discount on the discounted price, 192.60
= $192.60 –
1.07
how much more or less would this selling price be?
= $192.60 – $180
Solution = $12.60
(a) Let $M be the marked price of the TV set. Decrease in GST levied
Selling price = $14 – $12.60
= Marked price × (100% – Discount %) = $1.40
67
Level 2 Solution
8. Gopal earns an income of $4000 a month. He spends (a) The required percentage
45
$3360 and saves the rest. = × 100%
40
(a) What percentage of his income is his expenditure?
(b) What percentage of his income is his savings? = 112.5%
(b) The required percentage
Solution 40
3360 = × 100%
45
(a) The required percentage = × 100%
4000
= 88.9% (correct to 3 s.f.)
= 84%
(b) The required percentage = 100% – 84% 12. The length of a rectangle is 160% of its breadth. If the
= 16% breadth of the rectangle is 50 cm, find
(a) the length of the rectangle,
9. There are 80 books on a bookshelf. 28 of them are English (b) the perimeter of the rectangle.
books, 32 of them are Mathematics books and the rest
are Science books. Find the percentage of Solution
(a) English books, (a) Length of rectangle
(b) Mathematics books, = 160% × 50
(c) Science books = 80 cm
on the shelf. (b) Perimeter of rectangle
= 2(50 + 80)
Solution = 260 cm
28
(a) Percentage of English books = × 100%
80
= 35% Level 3
(b) Percentage of Mathematics books =
32
× 100% 13. In the year 2010, the population of Singapore was
80 5 076 700 and the population of Hong Kong was 7 097
= 40% 600. Express
(a) the population of Singapore as a percentage of the
(c) Percentage of Science books
population of Hong Kong,
= 100% – 35% – 40%
(b) the population of Hong Kong as a percentage of
= 25%
the population of Singapore.
Give your answer correct to 3 significant figures.
10. In a 400-ml can of mixed juice, 41% is mango juice,
32.5% is pineapple juice and the rest is water. Find the Solution
volume of (a) Population of Singapore as a percentage of the
(a) mango juice, population of Hong Kong
(b) pineapple juice, 5 076 700
= × 100%
(c) water 7 097 600
in the can. = 71.5% (correct to 3 s.f.)
(b) Population of Hong Kong as a percentage of the
Solution
population of Singapore
(a) Volume of mango juice = 400 ml × 41%
7 097 600
= 164 ml = × 100%
5 076 700
(b) Volume of pineapple juice = 400 ml × 32.5% = 140% (correct to 3 s.f.)
= 130 ml
(c) Volume of water = (400 – 164 – 130) ml 14. At a shooting practice, Peter hit the target 18 times out
= 106 ml of 25 shots and Cliff hit it 15 times out of 20 shots. Who
had a higher percentage of hitting the target during the
practice?
11. Ali’s mass is 45 kg and May’s mass is 40 kg.
(a) Express Ali’s mass as a percentage of May’s mass.
Solution
(b) Express May’s mass as a percentage of Ali’s mass.
Peter’s percentage of shots that hit targets
18
= × 100%
25
= 72%
Number of Number of 18. The number of pages of magazines A and B are 64 and
Line
vases inspected defective vases 96 respectively. The space used for advertisement in
1 30 3 magazine B is twice that in magazine A. If the percentage
2 50 4 of space used for advertisement in magazine B is 40%,
3 21 2 find the percentage of space used for advertisement in
magazine A.
Which production line had the lowest percentage of
defective vases? Solution
Number of pages of advertisement in magazine B
Solution = 40% × 96
Percentage of defective vases from Line 1 = 38.4
3
= × 100% Number of pages of advertisement in magazine A
30
= 38.4 ÷ 2
= 10%
= 19.2
Percentage of defective vases from Line 2
4
Required percentage
= × 100% 19.2
30 = × 100%
64
= 8%
= 15%
Percentage of defective vases from Line 3
2
= × 100% 19. The following table shows the number of different atoms
21
in three compounds.
11
= 9 % Number of atoms
21
Production line 2 has the lowest percentage of defective Compound Carbon Hydrogen Oxygen
vases. (C) (H) (O)
Benzene,
6 6 0
16. The capacity of a large cup is 50% more than that of a C6H6
small cup. By how many per cent is the capacity of the Ethanol,
small cup less than that of the large cup? 2 6 1
C2H5OH
Solution Glucose,
6 12 6
Let x be the capacity of the small cup. C6H12O6
Capacity of big cup
(a) Find the percentage of hydrogen atoms in each
= (100% + 50%) × x
compound.
= 1.5x
(b) Which compound has the highest percentage of
Required percentage hydrogen atoms?
1.5x – x
= × 100%
1.5x Solution
1 (a) Percentage of hydrogen atoms in Benzene
= 33 %
3 6
= × 100%
6+6+0
2
17. The area of a 3-room HDB flat is 60 m . = 50%
(a) The area of the living room is 30% of the area of Percentage of hydrogen atoms in Ethanol
the flat. Find the area of the living room. 6
(b) The area of the kitchen is 9 m2. Express the area = × 100%
2+6+1
of the kitchen as a percentage of the total area of 2
the flat. = 66 %
3
69
Percentage of hydrogen atoms in Glucose 2. 25% of the staff in an office is infected by flu. If
12 the number of staff infected by flu is 6, find the total
= × 100%
6 + 12 + 6 number of staff in the office.
= 50%
Solution
(b) Ethanol has the highest percentage of hydrogen
Let n be the total number of staff.
atoms.
n × 25% = 6
n = 6 ÷ 25%
= 6 × 100/25
Exercise 10.2 = 24
Level 1 The total number of staff is 24.
1. Find the unknown quantity in each case.
(a) 30% of a is 18.
3. A piece of alloy contains 4 kg of copper. If the alloy
(b) 37.5% of $b is $108.
contains 80% copper by mass, find the mass of the piece.
2
(c) 22 % of c kg is 44 kg.
9
Solution
(d) 150% of d cm2 is 126 cm2. Let m kg be the mass of the piece.
(e) 0.5% of e °C is 7 °C. m × 80% = 4
1 m = 4 ÷ 80%
(f ) % of f hours is 12 hours.
3
= 4 × 100/80
= 5
Solution
The piece of alloy is 5 kg.
(a) a × 30% = 18
18
a =
0.3 4. Adam attempts 65% of the questions in a test. If he
= 60 attempts 52 questions, find the total number of questions
The required quantity is 60. in the test.
(b) b × 37.5% = 108
Solution
108
b = Let the number of questions in the test be n.
0.375
n × 65% = 52
= 288 52
The required quantity is $288. n =
0.65
2 = 80
(c) c × 22 % = 44
9 There are 80 questions in the test.
44
c = 200
× 1
9 100
= 44 ×
9 Level 2
2
5. 45% of the members in a council are women. There are
= 198 72 female council members. Find
The required quantity is 198 kg. (a) the total number of council members,
(d) d × 150% = 126 (b) the number of male council members.
126
d = Solution
1.5
= 84 (a) Let n be the total number of council members.
The required quantity is 84 cm2. n × 45% = 72
72
(e) e × 0.5% = 7 n =
0.45
7 = 160
e =
0.005
The total number of council members is 160.
= 1400
The required quantity is 1400 °C. (b) Number of male council members = 160 – 72
= 88
1
(f )
f × % = 12
3
12
f = 6. Last Sunday, 85% of the customers of a supermarket in
1
× 1 a housing estate were residents of the same estate. 2380
3 100
Solution Solution
(a) Let N be the total number of customers. (a) Number of glasses of drinks sold by Stall A
N × 85% = 2380 = 175
2380 Number of glasses of drinks sold by Stall B
N =
0.85 = 220 × 85%
= 2800 = 187
The total number of customers is 2800. Number of glasses of drinks sold by Stall C
= 180
(b) Number of customers who were not residents
Stall B sold the greatest number of glasses of drinks.
= 2800 – 2380
The number of glasses of drinks sold was 187.
= 420
(b) Percentage of glasses of drinks sold by
175
7. John is 10% taller than Mary. If the difference between Stall A = × 100%
200
their heights is 16 cm, find Mary’s height. = 87.5%
Percentage of glasses of drinks sold by
Solution Stall B = 85%
Let h cm be the height of Mary. Percentage of glasses of drinks sold by
Height of John = h × (1 + 10%) Stall C = 80%
= 1.1h Stall A sold the highest percentage of its glasses
1.1h – h = 16 of drinks.
0.1h = 16 This percentage figure was 87.5%.
16
h = (c) Number of glasses of drinks by
0.1
= 160 Stall A = 200
\ The height of Mary is 160 cm. Number of glasses of drinks by
Stall B = 220
Number of glasses of drinks by
8. The price of book A is 15% less than that of book B. Stall C = 180 ÷ 80%
If the total price of book A and book B is $74, find the = 225
price of book B. Stall C prepared the greatest number of glasses of
drinks, which was 225 glasses of drinks.
Solution
Let $p be the price of Book B.
10. In Lihua’s music CD collection, 38% are Chinese music
Price of Book A = $p × (1 – 15%)
CDs, 44% are English music CDs and the rest are
= $0.85p
Japanese music CDs. There are 45 Japanese music CDs.
p + 0.85p = 74
(a) Find the total number of CDs in her collection.
1.85p = 74
74
(b) Find the number of Chinese music CDs.
p = 7
1.85 (c) If 63 %
of the English music CDs are CDs with
11
= 40 pop songs, find the number of CDs with English
\ The price of Book B is $40. pop songs.
Solution
Level 3 (a) Percentage of classical music CDs
9. At a food centre, there are three drink stalls. On a certain = 100% – 38% – 44%
day, Stall A sold 175 glasses out of 200 glasses of sugar = 18%
cane juice, Stall B sold 85% of its 220 glasses of soya Total number of CDs = 45 ÷ 18%
bean drink and Stall C sold 180 glasses of herbal tea = 250
which is 80% of its prepared glasses of herbal tea. (b) Number of Chinese music CDs = 250 × 38%
(a) Which stall sold the greatest number of glasses of = 95
drinks? How many glasses of drinks were sold by
(c) Number of CDs with English pop songs
this stall?
7
(b) Which stall sold the highest percentage of = 250 × 44% × 63 %
11
its prepared glasses of drinks? What was this 700 1
percentage figure? = 250 × 0.44 × ×
11 100
= 70
71
11. The master bedroom in a flat is 20% larger than In general,
the children’s bedroom. The area of living room is x% of y = xy%
200% that of the master bedroom. If the living room is and y% of x = xy%
14 m2 larger than the children’s bedroom, find ∴ x% of y and y% of x are equal.
(a) the area of the children’s bedroom,
(b) x × 60% = 30
(b) the total area of three rooms.
30
x =
0.6
Solution
(a) Let x m2 be the area of the children’s bedroom. = 50
Area of the master bedroom y × 30% = 60
60
= x (1 + 20%) y =
0.3
= 1.2x m2
Area of the living room = 200
= 1.2x × 200% ∴ x y
= 2.4x m2 (c) A club P has 30 members. If 60% of the members
2.4x – x = 14 are male, then there are 18 male members.
1.4x = 14 A club Q has 60 members. If 30% of the members
14 are male, then there are 18 male members.
x =
1.4 The number of male members of these two clubs
= 10 are equal.
The area of the children’s bedroom is 10 m2.
(b) Total area of three rooms
= x + 1.2x + 2.4x Exercise 10.3
= 4.6x Level 1
= 4.6 × 10 1. Copy and complete the following table.
= 46 m2
Original Increased Percentage
Increase
value value increase
12. A piece of architectural bronze, made of an alloy of (a) $25 $28 $3 12%
copper, zinc and lead, contains 57% of copper and 40%
of zinc by mass. If the difference in the mass of copper (b) 16 s 20 s 4s 25%
and that of zinc is 5.1 kg, find (c) 40 g 52 g 12 g 30%
(a) the mass of the piece, (d) 20 cm 23 cm 3 cm 15%
(b) the mass of lead in the piece.
(e) 50 °C 55 °C 5 °C 10%
Solution (f ) 60 copies 69 copies 9 copies 15%
(a) mass of piece
5.1 Solution
= kg
57% – 40% (a) Increase
= $28 – $25
= 30 kg = $3
3
(b) mass of lead in piece Percentage increase = × 100%
25
= (100% – 57% – 40%) × 30 kg = 12%
= 0.9 kg
(b) Increased value = 16 + 4
= 20 s
13. (a) Is 60% of 30 equal to 30% of 60? Explain your 4
answer. Generalise your findings using variables x Percentage increase = × 100%
16
and y.
= 25%
(b) If 60% of x is 30 and 30% of y is 60, is x equal to
y? Explain your answer. (c) Increased value = 40 × (100% + 30%)
(c) Write a real-life problem that involves the = 52 g
mathematics in (a). Increase = 40 × 30%
= 12 g
Solution
(d) Original value = 23 – 3
(a) 60% of 30 = 30 × 60% = 18
= 20 cm
30% of 60 = 60 × 30% = 18 3
∴ 60% of 30 and 30% of 60 are equal. Percentage increase = × 100%
20
= 15%
Solution
2. Copy and complete the following table.
Let $P be the original price of the watch.
Original Decreased Percentage P × (100% + 8%) = 777.60
Decrease
value value decrease P = 777.60 ÷ 1.08
(a) 2 = 720
36 kg 30 kg 6 kg 16 3 %
The original price of the watch was $720.
(b) 2h 1.5 h 0.5 h 25%
(c) 80 m 64.8 m 15.2 m 19% 5. The price of a computer system decreases from $1250
(d) 15 cm3 12 cm3 3 cm3 20%
to $1050. Find the percentage decrease in price.
73
Percentage increase =
2.8
× 100% (b) Number of customers at 2 p.m.
7.2 = 65 × (1 – 20%)
8
= 38 % = 65 × 80%
9
= 52
(c) The required percentage change
8. The regular price of an airline ticket is $960. (52 – 50)
(a) Its price increases by 30% during the peak season. = × 100%
50
Find the price of the ticket during the peak season. = 4%
(b) If the regular price drops to $748.80, find the
percentage decrease in the price.
Level 3
Solution
11. Refer to the news on 1 July 2011 below:
(a) Price of ticket during the peak season
HDB flat prices in the second quarter of 2011 rose by
= $960 × (100% + 30%)
2.9 per cent to a record, according to initial estimates
= $1248
by the Housing Board.
(b) Decrease = $960 – $748.80
The increase is the fastest since the third quarter of 2010.
= $211.20
Resale flat prices in the first quarter had risen by 1.6
211.20
Percentage decrease = × 100% per cent from the fourth quarter of 2010.
960
= 22% Suppose the price of an HDB flat was $400 000 on 1
January 2011. Estimate its price on
(a) 31 March 2011, (b) 30 June 2011.
9. The original height of a plant was 25 cm. One month
later, its height increased by 20%. In the second month, Solution
it increased by 15% of the plant’s height at the end of (a) Price on 31 March 2011
the first month. Find its height after = $400 000 × (1 + 1.6%)
(a) 1 month, = $406 400
(b) 2 months.
(b) Price on 30 June 2011
Solution
= $406 400 × (1 + 2.9%)
(a) Plant’s height after 1 month = $418 185.60
= 25 cm × (100% + 20%)
= 30 cm 12. Refer to the news on 30 June 2011 below:
(b) Plant’s height after 2 months Households can expect to pay an average 6.6 per cent
= 30 cm × (100% + 15%) more for their electricity in the July-September quarter
= 34.5 cm as tariffs go up.
SP Services said higher fuel oil prices have resulted
in higher power generation costs. The average fuel oil
10. There were 65 customers in a cafe at 1 p.m. on one day. price over the last three months has risen by 13.8 per
The number of customers increased by 30% from 12 p.m. cent from US$88.63 to US$100.84 per barrel.
to 1 p.m. and it decreased by 20% from 1 p.m. to 2 p.m. (a) Show how SP Services obtain the figure of 13.8%
(a) Find the number of customers in the cafe at 12 p.m. increase in oil price. Show your workings clearly.
(b) Find the number of customers in the cafe at 2 p.m. (b) Assume that a household consumes the same
(c) Find the percentage change in the number of units of electricity energy in June and July. If its
customers in the café from 12 p.m. to 2 p.m. electricity bill in July is $2.80 more than in June,
find the amount of its electricity bill in June.
Solution
(a) Let n be the number of customers at 12 p.m. Solution
Number of customers at 1 p.m (a) Percentage increase in oil price
= n × (1 + 30%) 100.84
= 1.3n = × 100% – 100%
88.63
1.3n = 65 = 13.8% (correct to 3 s.f.) (shown)
65
n = (b) Amount of electricity bill in June
1.3
n = 50 $2.80
=
The number of customers at 12 p.m. is 50. 6.6%
= $42.42
75
Solution 4. When a pair of shoes is sold at 12.5% discount, the
(a) Discount
= $125 – $100 selling price is $78.75. Find
= $25 (a) the marked price of the pair of shoes,
25 (b) the amount of discount.
Discount % = × 100%
125
= 20% Solution
(a) Let $M be the marked price.
(b) Selling price = $70 – $21 M × (100% – 12.5%) = 78.75
= $49 78.75
21 M =
0.875
Discount % = × 100%
70
= $90
= 30% The marked price is $90.
(c) Selling price = $240 × (100% – 15%) (b) Amount of discount = $(90 – 78.75)
= $204 = $11.25
Discount = $240 × 15%
= $36
5. Copy and complete the following table.
(d) Marked price = $256 + $64
Price before Amount of
= $320 Marked price
GST GST
64
Discount % = × 100% (a) $210 $200 $10
320
= 20% (b) $378 $360 $18
(c) $105 $100 $5
= Marked price × (100% – 22%)
(e) 351
$351 (d) $493.50 $470 $23.50
Marked price =
0.78 (e) $420 $400 $20
= $450 (f) $546 $520 $26
Discount
= $450 – $351
= $99
Solution
(f ) Marked price = $155 ÷ 31% (a) Marked price = $200 × (100% + 5%)
= $500 = $210
Selling price = $500 – $155 GST = $200 × 5%
= $345 = $10
(b) Marked price = $360 × (100% + 5%)
2. A table is sold at a discount of 20% of its marked price. = $378
Find its selling price if its marked price is $980. GST = $360 × 5%
= $18
Solution
(c) Price before GST = $5 ÷ 0.05
Selling price = $980 × (100% – 20%)
= $100
= $784
Marked price = $100 + $5
3. A rice cooker is sold at a discount of 15%. If the discount = $105
is $24, find
(d) Price before GST = $23.50 ÷ 0.05
(a) its marked price,
= $470
(b) its selling price.
Marked price = $470 + $23.50
Solution
= $493.50
(a) Let $M be the marked price. (e) Price before GST = $420 ÷ 1.05
M × 15% = 24 = $400
M =
24 GST = $420 – $400
0.15 = $20
= 160
(f ) Price before GST = $546 ÷ 1.05
The marked price is $160.
= $520
(b) Selling price = $160 – $24 GST = $546 – $520
= $136 = $26
77
(d) the average percentage discount on Mrs Li’s 14. Refer to the receipt below issued by a restaurant in which
purchases. the prices listed are not inclusive of GST. Find
(a) the subtotal amount,
Solution (b) the service charge amount,
(a) Selling price of the hair dryer (c) the GST amount,
= $50 × (100% – 5%) (d) the total amount, which is rounded up to the nearest
= $47.50 5 cents.
(b) Marked price of each skirt
= $120 ÷ (100% – 20%)
= $150
(c) Total amount of discount
= $[(50 – 47.50) + 2 × (150 – 120)]
= $62.50
(d) Original price of all the goods = $(50 + 2 × 150)
= $350
62.50
Average percentage discount = × 100%
350
6
= 17 %
7
Solution
(a) Mass of copper = 25 × 60%
= 15 kg
(b) Percentage of copper in new alloy
15
= × 100%
25 + 5
= 50%
3. 26% of ballpoint pens are red, 38% are blue and the
remaining ones are black. There are 57 blue ballpoint
pens in the box.
(a) Find the total number of ballpoint pens in the box.
(b) How many of them are black?
(c) If 10 more blue ballpoint pens were put into the box,
Solution what would the new percentage of blue ballpoint
(a) Subtotal amount pens be?
= 9.25 + 4.88 + 4.12 + 2.50 + 2.20 + 3.55 + 2.19
= $28.69 Solution
(a) Total number of ballpoint pens = 57 ÷ 38%
(b) Total amount = 150
= $28.69 – $0.04
= $28.65 (b) Percentage of black ballpoint pens
= 100% – 26% – 38%
(c) Rounding adjustment = 36%
= –$0.04 Number of black ballpoint pens = 150 × 36%
(d) Change = 54
= $50.70 – $28.65 (c) New percentage of blue ballpoint pens
= $22.05 57 + 10
= × 100%
(e) Total amount exclusive of GST 150 + 10
= $28.69 ÷ 107% 67
= × 100%
= $26.81 (correct to 2 d.p.) 160
= 41.875%
(f) GST amount
= $28.69 – $26.81
= $1.88 4. 30% of the members of a club are female and 20% of
them own cars. 30 female members in the club own cars.
(a) How many female members are there?
Revision Exercise 10 (b) What is the total number of members?
1. In a particular month, Mr Lin spent $900 which was (c) How many new female members have to be
30% of his monthly salary on food. recruited to increase the percentage of female
(a) Find his monthly salary. members to 37.5%?
(b) If he saved 12% of his monthly salary, find his
monthly savings. Solution
(a) Number of female members = 30 ÷ 20%
Solution = 150
(a) His monthly salary = $900 ÷ 30% (b) Total number of members = 150 ÷ 30%
= $3000 = 500
(b) His monthly savings = $3000 × 12%
= $360
79
(c) Let n be the number of new female members 8. The cost of making a table is $600, of which 40% is the
required. labour cost and the remaining is the material cost.
150 + n = (500 + n) × 37.5% (a) Find the cost of
150 + n = 187.5 + 0.375n (i) labour,
0.625n = 37.5 (ii) material used.
n = 60 (b) If the cost of labour is increased by 15% while
60 new female members have to be recruited. the cost of material is decreased by 20%, find the
percentage change in the cost of making the table.
5. Huaming earned $36 000 in the year 2010 and $38 880
Solution
in the year 2011.
2
(a) Find the percentage increase in his annual income (a) (i) Cost of labour = $600 × = $240
2+3
in the year 2011. 2
(b) If Huaming’s income in the year 2010 was 10% (ii) Cost of material used = $600 ×
2+3
less than that of the previous year, what was his = $360
annual income in the year 2009?
(b) New cost of labour = $240 × (100% + 15%)
Solution = $276
38 880 – 36 000 New cost of material used = $360 × (100% – 20%)
(a) Percentage increase = × 100% = $288
36 000
2880
New cost of making the table = $(276 + 288)
= × 100% = $564
36 000
= 8% Percentage decrease in the cost of making the table
600 – 564
(b) Income in the year 2009 = × 100%
600
= $36 000 ÷ (100% – 10%) 36
= $40 000 = × 100%
600
= 6%
6. Mr Wong’s monthly expenditure was $2125 after he had
reduced it by 15%. The original monthly expenditure was 9. At a clearance sale of a department store, handbags were
80% of his monthly income. Find sold at 40% discount and skirts at 25% discount. Jenny
(a) the original monthly expenditure, bought 2 handbags for $144 each and 3 skirts which
(b) his monthly income. were marked at $60 each. Find
(a) the marked price of each handbag,
Solution (b) the selling price of each skirt,
(a) Original monthly expenditure (c) the total discount on Jenny’s purchases,
= $2125 ÷ (100% – 15%) (d) the overall percentage discount,
= $2500 (e) the total amount of the GST on the handbags and
(b) Monthly income = $2500 ÷ 80% skirts bought, given that the GST rate is 7%.
= $3125
Solution
(a) Marked price for each handbag
7. Mrs Raj’s mass increased by 20% to 66 kg. = $144 ÷ (100% – 40%)
(a) Find her original mass. = $240
(b) If her mass dropped by 15%, find her new mass.
(c) What percentage is her new mass of her original (b) Selling price of each skirt
mass? = $60 × (100% – 25%)
= $45
Solution (c) Total marked price = $(240 × 2 + 60 × 3)
(a) Original mass = 66 ÷ (100% + 20%) = $660
= 55 kg Total selling price = $(144 × 2 + 45 × 3)
(b) New mass = 66 × (100% – 15%) = $423
= 56.1 kg Total discount = $660 – $423
56.1
= $237
(c) Required percentage = × 100% 237
55
(d) Overall percentage discount = × 100%
= 102% 660
10
= 35 %
11
81
(d) the total amount, which is rounded up to the nearest
5 cents.
Solution
(a) Subtotal amount
= 8.80 + 5.00 + 14.80 + 5.80 + 8.00 + 2.40 + 0.80
= $45.60
(b) Service charge amount
= $45.60 × 10%
= $4.56
(c) GST amount
= ($45.60 + $4.56) × 7%
= $3.51 (correct to 2 d.p.)
(d) Total amount
= 45.60 + 4.56 + 3.51
= 53.67
= $53.70 (correct to the nearest 5 cents)
Class Activity 1
Objective: To recognise number patterns.
Questions
1. In each of the following cases, the first four terms of a sequence are given. Find the 5th and 6th terms of each sequence.
Let an be the number of green dots and Tn be the total number of dots in the nth pattern of the diagram.
(a) Copy the following table and write down the terms T1 , T2 , T3 and T4 in your table.
Figure number, n Total number of dots in Figure n, Tn Number of green dots in Figure n, an
1 1 1
2 4 3
3 9 5
4 16 7
T5 = 52
= 25
T6 = 62
= 36
83
(c) What pattern do you observe? Describe it.
T11 = 112
= 121
(e) Write down the terms a1, a2, a3 and a4 in your table above.
a5 = 7 + 2
= 9
a6 = 9 + 2
= 11
A restaurant uses square tables where each of them can seat 4 customers with a chair on each side (see
Figure 1). Tables can be placed together to form a long rectangular table to handle larger groups of customers (see Figure 2
and Figure 3).
Questions
1. Draw a diagram showing the arrangement with chairs when 4 tables are placed together.
Figure 4
5 5 4 + 2 + 2 + 2 + 2 = 12
6 6 4 + 2 + 2 + 2 + 2 + 2 = 14
85
3. What pattern do you observe about Cn in the table above? Explain your answer.
The rule is to add 2 to the previous term to get to the next term.
4. Write down the number of chairs available when 18 square tables are placed together.
C18 = 4 + (18 – 1) × 2
= 38
Cn = 4 + (n – 1) × 2
= 2n + 2
87
5. The diagram below shows a sequence of dot patterns 7. In a multi-storey building, the first floor is 5 m above
where n is the figure number. Let Tn be the number of the ground level and each subsequent floor is 3 m above
dots in the nth pattern. the previous floor.
(a) How high is the 5th floor above the ground level?
The diagram below shows a sequence of dot patterns.
(b) Express the height of the nth floor above the ground
level in terms of n.
(c) When you are 155 m above the ground level, which
floor are you on?
Solution
(a) 17 metres
(b) Height of nth floor = 5 + (n – 1) × 3
= 5 + 3n – 3
Figure number = 3n + 2
(c) Height of nth floor = 155
(a) Write down the number of dots for Figures 4 and
155 = 3n + 2
5 in the table below.
3n = 153
Number of dots in n = 51
Figure number, n
Figure n, Tn I would be at the 51st floor.
1 1
2 1+4=5
3 1+4+4=9
4 1 + 4 + 4 + 4 = 13
5 1 + 4 + 4 + 4 + 4 = 17
Solution
(a) The sequence of Tn is 1, 5, 9, 13, 17, …
The general term Tn = 1 + (n – 1) × 4
= 1 + 4n – 4
= 4n – 3
(b) The 18th term T18 = 4(18) – 3
= 69
Solution
(a) 4, 7, 10, 13, …
The general term Tn = 4 + (n – 1) × 3
= 4 + 3n – 3
= 3n + 1
(b) T15 = 3(15) + 1
= 46
89
Level 2 7. A sequence is formed by 2 × 12, 2 × 22, 2 × 32,
4. Find the 7th term of each sequence. 2 × 42, …
(a) 2, 4, 6, 8, … (a) Write down the first 4 terms of the sequence.
(b) 23, 20, 17, 14, … (b) Find the 8th term of the sequence.
(c) 5, 10, 20, 40, …
(d) 216, –144, 96, – 64, … Solution
(a) 2 × 12, 2 × 22, 2 × 32, 2 × 42, …
Solution T1 = 2 × 12
(a) 2, 4, 6, 8, … = 2
T5 = 8 + 2 = 10 T2 = 2 × 22
T6 = 10 + 2 = 12 = 8
T3 = 2 × 32
T7 = 12 + 2 = 14
= 18
(b) 23, 20, 17, 14, … T4 = 2 × 42
T5 = 14 – 3 = 11 = 32
T6 = 11 – 3 = 8 (b) T8 = 2 × 82
T7 = 8 – 3 = 5 = 128
Solution
T1 = 17
T2 = 17 + 4
= 21
T3 = 17 + 4 + 4
= 25
T4 = 17 + 4 + 4 + 4
= 29
Solution
(a) The 5th figure is as shown below.
Solution
(a) 30 seats
(b) Total number of seats
= 20 + 22 + 24 + 26 + 28 + 30
= 150 (b) By counting the tiles,
T1 = 4
11. A ball is dropped onto a floor from a height of 320 cm. T2 = 6
3 T3 = 8
Each time it falls, it rebounds to of the height from T4 = 10
4
where it fell. Let an be the height of the nth rebound in T5 = 12
centimetre.
(c) P1 = 10 × 2
= 20
P2 = 20 + 4
= 24
P3 = 24 + 4
= 28
P4 = 28 + 4
= 32
P5 = 32 + 4
(a) Find the values of a1, a2 and a3. = 36
(b) Determine the value of a6.
13. The diagram shows a sequence of huts formed by
Solution matchsticks, where n is the figure number.
3
(a) a1 = × 320
4
= 240
2
3
a2 = × 320
4
3
= × 240
4
= 180
91
The matchsticks, of length 3 cm each, are arranged end to (b) The next 3 terms are
end. If Tn is the number of matchsticks in the nth figure T8 = 8 + 13
and Pn is its respective perimeter, find = 21
(a) Tn for n = 1 to 5, (b) T8, T9 = 13 + 21
(c) Pn for n = 1 to 5, (d) P8. = 34
T10 = 21 + 34
Solution = 55
(a) By counting, (c)
T1 = 5
T2 = 9
T3 = 13
T4 = 17
T5 = 21 Ignoring the first term, the difference between
successive terms of the Fibonacci sequence is the
(b) T8 = 21 + 4 + 4 + 4
Fibonacci sequence itself.
= 33
(c) P1 = 5×3
= 15 Exercise 11.2
P2 = 8×3 Level 1
= 24
1. Find the first 3 terms of each sequence from the given
P3 = 11 × 3
general term Tn.
= 33
(a) Tn = 2n + 1 (b) Tn = 7 – 3n
P4 = 14 × 3
n
= 42 (c) Tn = 2(n – 1)2 (d) Tn =
n+2
P5 = 17 × 3
= 51 Solution
(a) Tn = 2n + 1 (b) Tn = 7 – 3n
(d) P8 = 51 + 9 + 9 + 9 T1 = 7 – 3(1)
T1 = 2(1) + 1
= 78 = 4
= 3
T2 = 2(2) + 1 T2 = 7 – 3(2)
14. The sequence 1, 1, 2, 3, 5, 8, 13, … is known as the = 5 = 1
Fibonacci sequence. T3 = 2(3) + 1 T3 = 7 – 3(3)
(a) What is the rule used to obtain the terms in the = 7 = –2
sequence?
n
(b) Write down the next 3 terms of the sequence. (c) Tn = 2(n – 1)2 (d) Tn =
n+2
(c) Complete the following where each term in the T1 = 2(1 – 1)2
lower row is the difference of the two terms just 1
= 0 T1 =
2 1+2
above it. T2 = 2(2 – 1)
= 2 1
=
T3 = 2(3 – 1) 2 3
2
= 8 T2 =
2+2
1
=
Write a paragraph to explain what you have 2
discovered. 3
T3 =
3+2
Note: You may access the website www.starpub.com.sg/
dm/s1e/weblinks_1b.html for more information 3
=
regarding this famous sequence. 5
Solution Solution
Tn = n3 – 1 (a) Tn = 2n
T7 = 73 – 1 (b) (i) The sequence 3, 5, 7, 9, 11, …
= 342 can be written as
n
1 + 2, 1 + 4, 1 + 6, 1 + 8, 1 + 10, …
2 \ Tn = 1 + 2n
4. The general term of a sequence is Tn = 108 × . Find
3
the 3rd term. (ii) The sequence 4, 16, 36, 64, 100, …
can be written as
Solution 4(1)2, 4(2)2, 4(3)2, 4(4)2, 4(5)2, …
n
2 \ Tn = 4n2
Tn = 108 ×
3
3 (iii) The sequence 4, 16, 36, 64, 100, …
2
T3 = 108 × = 32 can be written as
3
4(1 + 1)2, 4(2 + 1)2, 4(3 + 1)2, …
\ Tn = 4(n + 1)2
Level 2
5. The general term of a sequence is Tn = 7n + 4. Find the 9. The general term of a sequence is Tn = 2n2.
sum of its 5th term and 6th term. (a) Write down the first 4 terms of the sequence.
(b) The first 4 terms of another sequence are 5, 11, 21,
Solution 35.
Tn = 7n + 4 (i) Suggest a formula for the general term of the
T5 = 7(5) + 4 = 39 new sequence.
T6 = 7(6) + 4 = 46 (ii) Find the 6th term of the new sequence.
The required sum = T5 + T6 Solution
= 39 + 46 (a) T1 = 2(1)2
= 85 = 2
T2 = 2(2)2
6. Consider the sequence 39, 37, 35, 33, … . = 8
(a) Find its general term Tn in terms of n. T3 = 2(3)2
(b) Hence, find the 18th term. = 18
T4 = 2(4)2
Solution = 32
(a) 39, 37, 35, 33, … (b) The sequence 5, 11, 21, 35, …
Tn = 39 + (n – 1) × (–2) can be written as
= 41 – 2n 2 + 3, 8 + 3, 18 + 3, 32 + 3 , …
(b) T18 = 41 – 2(18) (i) Tn = 2n2 + 3
= 5
(ii) T6 = 2(6)2 + 3
= 75
7. Express the nth term of the sequence 8, 27, 64, 125, ...
in terms of n.
10. Consider the sequence 4, 10, 18, 28, … . Ali and Sumin
Solution observe different number patterns as shown in the table.
The sequence 8, 27, 64, 125, …
can be written as Ali’s Sumin’s
Term, Tn
(1 + 1)3, (2 + 1)3, (3 + 1)3, (4 + 1)3, … pattern pattern
\ Tn = (n + 1)3 1 1×4 12 + 3 × 1
2 2×5 22 + 3 × 2
8. (a) For the sequence 2, 4, 6, 8, 10, … , express the nth
3 3×6 32 + 3 × 3
term in terms of n.
(b) Hence, express the nth term of each of the following 4 4×7 42 + 3 × 4
sequences in terms of n.
(i) 3, 5, 7, 9, 11, ...
93
(a) Find the general term an based on Ali’s number (a) Observe the patterns. Copy and complete the
patterns. following table.
(b) Find the general term Sn based on Sumin’s number
patterns. Figure Number of Total number
(c) Is an = Sn? Explain your answer. number, n red dots of dots
1 1 1×2
Solution 2 1+2 2×3
(a) an = n × (n + 3) = n(n + 3) 3
(b) Sn = n2 + 3 × n = n2 + 3n 4
(c) Since n(n + 3) = n2 + 3n, an = Sn. 5
(b) What is the ratio of the number of red dots to the
11. The diagram below shows a sequence of patterns. total number of dots in each pattern?
(c) Hence, find a formula in terms of n, for the sum
1 + 2 + 3 + … + n.
(d) Evaluate the sum 51 + 52 + 53 + … + 200 using
the result in (c).
Solution
(a) Observe the patterns. What are the missing numbers
(a) Figure Number of Total number
in the brackets?
number, n red dots of dots
1 = 12 1 1 1×2
1 + 3 = ( )2 2 1+2 2×3
1 + 3 + 5 = ( )2 3 1+2+3 3×4
1 + 3 + 5 + 7 = ( )2 4 1+2+3+4 4×5
5 1+2+3+4+5 5×6
(b) Express the sum 1 + 3 + 5 + 7 + 9 as a square
number. Number of red dots 1
(b) = =1:2
(c) Express the sum 1 + 3 + 5 + … + (2n – 1) as a Total number of dots 2
square number, where n is a positive integer. (c) For the nth pattern,
number of red dots = 1 + 2 + 3 + … + n
Solution
total number of dots = n(n + 1).
(a) 1 = 12
From the result in (b), we have
1 + 3 = (2)2
1
1 + 3 + 5 = (3)2 1 + 2 + 3 + … + n = n(n + 1).
2
1 + 3 + 5 + 7 = (4)2
1
(d) 1 + 2 + 3 + … + 200 = × 200 × (200 + 1)
(b) 1 + 3 + 5 + 7 + 9 = 52 2
Level 3
13. Study the rectangles formed by small square tiles in the
diagram.
95
(d) T5 = – 40 × (–2) Solution
= 80 (a) Since
T6 = 80 × (–2) 1 1
19 – 23 = 16 – 19
= –160 2 2
1
= 12 – 16
1 2
2. If the general term Tn of a sequence is Tn = , find
2n + 1 1
the sum of the first 3 terms. = –3 ,
2
1
Solution T5 = 12 – 3
2
Sum of the first 3 terms 1
T1 + T2 + T3 =8
2
1 1 1
= + + 1
2(1) + 1 2(2) + 1 2(3) + 1 T 6 = 9 – 3
2
1 1 1 1
= + + =4
3 5 7 2
35 + 21 + 15
= (b) Since
105
9 – 4
71
= = 14 – 9
105
= 19 – 14
= 5
3. Find the general term of each sequence. the nth term = 4 + (n – 1) × 5
(a) 2, 3, 4, 5, … = 4 + 5n – 5
(b)
1 2 3 4
, , , , … = 5n – 1
4 5 6 7
(c) 1 × 2, 2 × 3, 3 × 4, 4 × 5, …
6. The first 5 terms of a sequence are 3, 9, 27, 81 and 243
respectively.
Solution
(a) Find the next two terms of the sequence.
(a) 2, 3, 4, 5, …
(b) Express the nth term of the sequence in terms of
Tn = n + 1
n.
1 2 3 4 (c) Hence, find the nth term of the sequence
(b) , , , , …
4 5 6 7 5, 11, 29, 83, 245, ... in terms of n.
n
Tn =
n+3 Solution
(a) The sequence
(c) 1 × 2, 2 × 3, 3 × 4, 4 × 5, … 3, 9, 27, 81, 243, …
Tn = n(n + 1) can be written as
3, 32, 33, 34, 35, …
4. For the sequence 3, 8, 15, 24, 35, … , the nth term is \ the 6th term = 36
n(n + 2). Find the 31st term. = 729
the 7th term = 37
Solution
= 2187
T31 = 31(31 + 2)
= 1023 (b) The nth term = 3n
(c) The sequence
5. (a) Find the next two terms of the sequence 5, 11, 29, 83, 245, …
1 1 can be written as
23, 19 , 16, 12 , … .
2 2 3 + 2, 9 + 2, 27 + 2, 81 + 2, 243 + 2, …
(b) Express the nth term of the sequence
\ the nth term = 3n + 2
4, 9, 14, 19, … in terms of n.
Solution
(a) (i) The 5th line is
5 × 9 + 4 = 49
(ii) The nth line is
(a) Copy and complete the following table.
n × (n + 4) + 4 = (n + 2)2
(b) When k(k + 4) = 1085, n 1 2 3 4
k(k + 4) + 4 = 1085 + 4 Tn
= 1089
(b) Express Tn in terms of n.
From (a)(ii), (c) How many matchsticks are required for a 10 × 10
(k+2)2 = 1089 square pattern?
k + 2 = 33
k = 31
Solution
Solution
(a) The diagram below shows the pattern for n = 4.
(i) Gn = n + 6 (c) Find the formula in terms of n for the general term
(i) Tn, (ii) Pn,
(ii) Wn = 2n
97
Solution
(a) The diagram below shows the 4th pattern.
(b) n 1 2 3 4
Tn 1 4 9 16
Pn 7 14 21 28
(c) (i) Tn = n2
(ii) Pn = 7n
Class Activity 1
Objective: To specify and plot points on a two-dimensional Cartesian plane in a battleship game.
Tasks
1. Prepare your mathematical ocean on a graph paper. It is a two-dimensional Cartesian plane where both the values of x
and y range from –5 to 5.
2. You are given four battleships. The names and the lengths of these battleships are as follows:
Aircraft Carrier (denoted by AAAAA) – 5 units Cruiser (denoted by CCCC) – 4 units
Destroyer (denoted by DDD) – 3 units Submarine (denoted by SS) – 2 units
Place these battleships randomly on your ocean with the following conditions:
– The ships must either be in a horizontal or vertical position.
– The ships cannot overlap one another.
An example is shown below.
y Keep track of all the missiles fired
Fired to opponent Fired at me
5
Coordinates Hit Miss Coordinates Hit Miss
A A4 A A A ( , ) ( , )
( , ) ( , )
( , ) ( , )
3 Miss
( , ) ( , )
C ( , ) ( , )
2 Hits ( , ) ( , )
C ( , ) ( , )
1 S S ( , ) ( , )
C ( , ) ( , )
_5 _4 _3 O x ( , ) ( , )
_2 _1 1 2 3 4 5
C ( , ) ( , )
_1 ( , ) ( , )
D ( , ) ( , )
_2 ( , ) ( , )
D ( , ) ( , )
_3 ( , ) ( , )
( , ) ( , )
D
_4 ( , ) ( , )
( , ) ( , )
_5 ( , ) ( , )
3. Play the battleship game with a classmate. You and your classmate will take turns to fire missiles by calling out ordered
pairs. If your opponent calls out an ordered pair that your battleships are located, the missile is a hit. For example, for
the battleships located as shown in the diagram, the ordered pair (2, 1) hits the submarine while (2, 3) is a miss. Every
missile fired at you must be made known to your opponent whether it is a hit or a miss. You can use the given table to
keep track of all the missiles fired.
4. A ship will sink if all the integral coordinates pairs of the ship are being hit. The first person who sinks all opposing
battleships is the winner.
99
Class Activity 2
Objective: To use a function machine to understand the concept of function and represent a function using verbal, tabular,
graphical and algebraic forms.
A function can be considered as a machine. It takes an input, applies a rule to it, and then produces an output. We may view
the process as follows:
5, 10, 15, 20, 25
Input Rule Output Input x
Questions
1. Guess the rule of the function. In other words, how do you get 8 from 5, 13 from 10, 18 from 15, etc? Can you observe
the rule? Express the rule in your own words.
(i) When x = 5, y = 5 + 3 = 8.
From observation, the rule is to add 3 to the input to get the output.
2. We may represent a function by a table. Copy and complete the following table.
Input 5 10 15 20 25
Ouput 8 13 18 23 28
(5, 8), (10, 13), (15, 18), (20, 23), (25, 28) 25
15
10
x
O 5 10 15 20 25
When x = 27,
y = 27 + 3
= 30
y=x+3
5. Try to form your own function machine and represent some pairs of input and output values on a coordinate plane.
101
Class Activity 3
Objective: To recognise how the graph of the linear function y = ax + b varies when either a or b varies.
Tasks
Graph of y = ax + b
6
4
A: (–1.50, 0.00)
B: (0.00, 3.00)
B
Slope AB = 2.00
2
A
15 10 5 5 10 15
y = 2·x + 3 2
2. Mark the points A and B where the graph cuts the x-axis and y-axis respectively.
3. Measure the gradient of the line using the command in the software.
Gradient = 2
4. (a) By keeping b fixed, observe how the graph changes when a varies from –5 to 5.
(b) How does the line slope when a is negative and when a is positive?
When a is positive, the line slope increases upwards when viewed from left to right.
When a is negative, the line slope decreases downwards when viewed from left to right.
(c) By keeping a fixed, observe how the graph changes when b varies from –3 to 3.
(a) the coordinates of the point where the graph cuts the y-axis,
(0, b)
Gradient = a
103
Class Activity 4
Objective: To understand the physical interpretation of the gradient of a linear graph.
Questions
A car drives along a straight road which passes through a city P. Its distance, y km, from city P at time x hours is given by the
function y = 50x + 20 for 0 < x < 4.
1. Copy and complete the following table for the line y = 50x + 20. Then, draw the graph of y = 50x + 20 for
0 < x < 4 on the diagram on the right.
x 0 1 2 3 4
y = 50x + 20 20 70 120 170 220
250
200 y = 50x + 20
150
100
50
x
O 1 2 3 4
y-intercept = 20
The y-intercept is the distance in km of the car from City P at the time x = 0.
3. (a) Describe a way of finding the speed of the car in the first hour from the graph.
The speed of the car can be found by finding the gradient of the graph.
(b) Hence, find the speed of the car during the first hour.
(c) Check if the speed of the car is the same throughout the journey. Explain your answer.
The gradient of the car for each hour is always 50 km/h. Therefore, the speed of the car is the same throughout the journey.
(b) How is the gradient of the graph related to the movement of the car?
P Solution
1
(a) Input x
x
_3 _2 _1 O 1 2 3
_1
Q
6 – 5x
_2
_3
Input y
(a) State the coordinates of the points P and Q in the (b) When x = 4,
above diagram. y = 6 – 5 × 4
(b) State the quadrants in which P and Q lie. = –14
(c) Use a scale of 1 cm to 1 unit on both axes, plot
(c) When y = 6,
the points R(1, 3), S(–2, 3), T(0, –1) and U(2, –2)
6 = 6 – 5x
on a sheet of graph paper.
–5x = 0
x = 0
Solution
y
3. Ali attempted 10 multiple-choice questions in a quiz.
S(–2, 3)
3 R(1, 3)
He got x questions answered correctly and y questions
answered wrongly.
(a) Copy and complete the following table.
2
P
x 2 4 6 8 10
1
y 8
x
_3 _2 _1 O 1 2 3 (b) Plot the ordered pair (x, y) on the coordinate plane
as shown below.
_1 T(0, –1)
Q y
_2
U(2, –2) 10
_3
8
6
(a) Coordinates of P = (3, 1),
Coordinates of Q = (–2, –1)
4
(b) P is in the first quadrant.
Q is in the third quadrant.
2
(c) Points R, S, T, U are plotted on the above diagram.
x
O 2 4 6 8 10
105
(c) What can you say about the points plotted? (c) y
(d) Write down an equation that represents the
relationship between x and y. 300
Solution
250
(a)
x 2 4 6 8 10
200
y 8 6 4 2 0
(b) y 150
10 100
8 50
6 x
O 50 100 150 200 250
4
(d) The points form a straight line. They are collinear.
2
x Section 12.3
O 2 4 6 8 10 5. (a) Using a scale of 1 cm to 1 unit on both axes, draw
1
the graph of y = – x – 1 for values of x from –4
2
(c) The points form a straight line. They are collinear. to 4.
(d) The required equation is y = 10 – x. (b) Does the point B(1, –1) lie on the graph?
Solution
4. A restaurant has 10% service charge. Let $y be the amount
including service charge when the food ordered is $x. (a) x – 4 –2 0 2 4
(a) Express y as a function of x in the form of an y=– x–1
1
1 0 –1 –2 –3
equation. 2
(b) Copy and complete the following table. y
x 50 100 150 200 250 4
y
y = _ 12 x _ 1 2
30
x h = 40 _ 5t
–3 –2 –1 O 1 2 3
20
–2
10
t
O 2 4 6 8
y = 10 – 2x Solution
4 Gradient of L3 is undefined, gradient of L4 is 0.
2 10. The vertices of PQR are P(–2, –1), Q(3, –1) and
R(1, 2). Find the gradients of the sides of PQR.
x
O 1 2 3 4 5
Solution
y
(c) She would receive $4 change.
R(1, 2)
2
8. Water is being drained from a tank. The water level h
cm at time t minutes is given by h = 40 – 5t. 1
(a) Using a scale of 1 cm to 1 unit on the t-axis and
1 cm to 5 units on the h-axis, draw the graph of x
_2 _1 O 1 2 3
h = 40 – 5t from t = 0 to t = 8.
(b) Use the graph to find the time when the water level _ Q(3, _ 1)
P(_ 2, _ 1) 1
is 15 cm.
107
Since PQ is horizontal, (b) When t = 0,
y = –2 × 0 + 30
gradient of PQ = 0,
= 30
–3
gradient of QR = \ 30 is the initial volume of water in cm3 in the
2
filter.
3
gradient of PR =
3 (c) –2 is the rate of change of the volume of water in
= 1. the filter in cm3/min.
A x
_2 O 2 4 6
_2
y = 3x _ 6
_4
_6
B
Solution
(a) A(2,0)
(b) B(0,-6)
(c) Gradient = 3
Solution
(a) y = –2t + 30
When t = 10,
y = –2 × 10 + 30
= 10
When t = 11,
y = –2 × 11 + 30
= 8
\ Volume of water drained
= 10 – 8
= 2 cm2
4 A(0, 4)
1
B(5, 3)
A 3 C(1, 3)
x
_5 _4 _3 _2 _1 O 1 2 3 4 5 E(_3, 2)
_1 H
2
_2
1
J F(_2, 0)
x
_3 _5 _4 _3 _2 _1 O 1 2 3 4 5
F I
_1
G _4
_2
G(_5, _2)
_3
Solution
(a) The coordinates of the points are: _4 H(3, _4)
109
6. (a) Plot the points P(1, –2) and Q(– 4, –2) on a Cartesian 8. (a) Plot the points A(–3, 1), B(0, –1), C(1, –1) and
plane. D(4, –1) on a Cartesian plane.
(b) State the quadrant in which P lies. (b) Which of the above points are in the fourth
(c) State the quadrant in which Q lies. quadrant?
(d) Join OP and OQ. Then measure ∠POQ. (c) If a point is in the fourth quadrant, what is the sign
(e) Find the coordinates of a point R such that the of its
x-axis is the perpendicular bisector of the line (i) x-coordinate?
segment QR. (ii) y-coordinate?
Solution Solution
(a) y (a) y
R(_ 4, 2)
2 A(_ 3, 1)
1
1 x
_3 _2 _1 O 1 2 3 4
_4 _3 _2 _1 O x _1
1 2 D(4, _ 1)
B(0, _ 1) C(1, _ 1)
_1
_2
Q(_ 4, _ 2) P(1, _ 2) (b) C(1, –1) and D(4, –1) are in the fourth quadrant.
(c) (i) The sign of its x-coordinate is positive.
(b) P(1, –2) lies in the fourth quadrant. (ii) The sign of its y-coordinate is negative.
(c) Q(– 4, –2) lies in the third quadrant.
(d) ∠POQ = 90° Level 3
(e) Coordinates of R = (– 4, 2) 9. (a) On a sheet of graph paper, draw a Cartesian plane
using a scale of 1 cm to represent 2 units on both
axes and then plot the following points.
7. (a) Plot the points S(–5, 0) and T(0, 2) on a Cartesian
plane. A(6, 0), B(3, 1), C(2, 6), D(0, 2),
(b) Join ST. Find the area of SOT. E(– 4, 2), F(–2, 0), G(– 6, – 6), H(–1, –3),
(c) State the coordinates of the midpoint M of S and I(4, – 6), J(2, –2)
T. (b) Join the points successively from A to J with straight
(d) State the relationship between the lengths of MO, line segments. Join the points J and A. What design
MS and MT. do you get?
Solution Solution
(a) y (a) and (b)
2 T(0, 2) y
M C
1 6
S(_ 5, 0)
_5 _4 _3 _2 _1 O x 4
_1 E 2 D
B
F A
_6 _4 _2 O x
2 4 6
_2
(b) Area of SOT =
1
× OS × OT J
2
H
_4
1
= ×5×2
2 _6
= 5 units2 G I
1
(c) Coordinates of M = –2 , 1
5
(d) MO = MS = MT
(d) y = 2x + 3
(iii) Punggol
(iv) Queenstown
(b) Name the places located at the following Output y Output y
coordinates.
(i) (0, 1) Solution
(ii) (–6, 0) (a) When x = 3,
(iii) (–2, 4) y = x + 7
(iv) (4, –1) = 3 + 7
(c) Find the units of distance between = 10
(i) Jurong and Bukit Timah on the map,
(b) When x = 3,
(ii) Sembawang and Queenstown on the map.
y = 5 – 2x
= 5 – 2 × 3
Solution
= –1
(a) (i) Changi International Airport is at (6, 0).
(ii) Yishun is at (0, 3).
3. A function y of x is given by y = x + 1.
(ii) Punggol is at (3, 2). (a) Copy and complete the following table.
(ii) Queenstown is at (–2, –2). x 0 1 2 3 4
(b) (i) (0, 1) is Ang Mo Kio. y
(ii) (–6, 0) is Jurong.
(b) Plot the ordered pairs (x, y) of the above table on
(iii) (–2, 4) is Sembawang. a copy of the coordinate plane shown below.
(iv) (4, –1) is Bedok. y
(c) (i) The required distance
= –2 – (–6) 5
= 4 units
(ii) The required distance 4
= 4 – (–2)
= 6 units 3
2
Exercise 12.2
Level 1
1
1. Express y as a function of x in the form of an equation
in each of the following.
x
(a) y is 7 less than x. O 1 2 3 4
(b) y is three times of x.
(c) y is one-half of x.
(d) y is 3 more than twice of x.
111
Solution (b) y
(a) y = x + 1
6
x 0 1 2 3 4
y 1 2 3 4 5
4
(b) y
2
5
x
_4 _2 O 2 4
4
3
Level 2
2
5. (–3, 3), (–2, 2), (1, –1) and (4, – 4) are four ordered pairs
of a function y of x.
1 (a) Determine which of the following can be the
function.
x (i) y = –x
O 1 2 3 4 (ii) y = x + 6
(b) Find the value of y when x = 3.
Solution
1
4. A function y of x is given by y = 4 – x. (a) All the given points satisfy the equation y = –x.
2
When x = 1,
(a) Copy and complete the following table. y = x + 6
x –4 –2 0 2 4 = 1 + 6
= 7
y \ (1, –1) does not satisfy the equation y = x – 6.
Hence, the required function is y = –x.
(b) Plot the ordered pairs (x, y) of the above table on
a copy of the coordinate plane shown below. (b) When x = 3,
y = –x
y = –3
6
6. A function y of x has values of x and y as shown in the
table below.
4
x 1 2 3 4 5
2 y 3 4 5 6 7
10 _2
8
(c) The plotted points lie on a straight line.
6
x x
_4 _2 O 2 4 _4 _2
O 2 4
_2 _2
(c) What can you say about the points plotted? (a) If (–2, a) and (b, 2) are points on the graph, find
the values of a and b.
Solution (b) Write down the equation of the function.
(a) y = 1 – 2x
(–2, p) is a point on the function. Solution
p = 1 – 2(–2) (a) From the graph,
\ p = 5 When x = –2, y = –1
(q, –1) is a point on the function. \ a = –1
–1 = 1 – 2q When y = 2, x = 4
2q = 2 \ b = 4
1
\ q = 1 (b) The equation of the function is y = x.
2
Level 3
9. Let y be the total number of wheels of x bicycles.
(a) Copy and complete the following table.
x 1 2 3 4
y
(b) Using a scale of 1 cm to 1 unit on both the x-axis
and the y-axis, plot the ordered pairs (x, y) of the
above table on the coordinate plane.
(c) Write down the equation connecting x and y.
113
Solution (b) y
(a) x 1 2 3 4
11
y 2 4 6 8
(b) y 10
8 9
7 8
6 7
5 6
4 5
3 4
2 3
1 2
x 1
O 1 2 3 4
x
O 1 2 3 4
(c) The required equation is y = 2x.
10. A lift is at the 3rd floor. It rises 2 floor levels per second. (c) The required equation is y = 2x + 3.
Let y be the floor level of the lift after x seconds. (d) When y = 15,
(a) Copy and complete the following table. 15 = 2x + 3
x 0 1 2 3 4 2x = 12
x = 6
y 3 \ The time taken is 6 seconds.
(b) Using a scale of 1 cm to 1 unit on both the x-axis
and the y-axis, plot the ordered pairs (x, y) of the 11. Patrick has $8 in his pocket. Let $y be the amount left
above table on the coordinate plane. after he spends $x on his lunch.
(c) Write down the equation connecting x and y. (a) Copy and complete the following table.
(d) Hence, find the time taken for the lift to reach the
x 2 3 4 5 6
15th floor.
y
Solution
(a) (b) Using a scale of 1 cm to 1 unit on both the x-axis
x 0 1 2 3 4
and the y-axis, plot the ordered pairs (x, y) of the
y 3 5 7 9 11 above table on the coordinate plane.
(c) What can you say about the points plotted in (b)?
(d) Write down the equation connecting x and y.
(e) Hence, find the value of x when y = 7.
Exercise 12.3
3
Level 1
For Question 1 to 7, use a scale of 1 cm to 1 unit on both the
2
x-axis and the y-axis to draw the graphs.
1
1. (a) Copy and complete the following table.
x –4 –2 0 2 4
x y=x–2
O 1 2 3 4 5 6
(b) Draw the graph of y = x – 2 for values of x from – 4
to 4.
(c) The plotted points lie on a straight line.
Solution
(d) The required equation is y = 8 – x.
(a) x –4 –2 0 2 4
(c) When y = 7,
y=x–2 –6 –4 –2 0 2
7 =8–x
x = 8 – 7 (b) y
x = 1
2 y=x_2
115
Solution 4. (a) Draw the graph of y = 3(x – 1) for values of x from
(a) –3 to 3.
x –4 –2 0 2 4
(b) Find the point at which the graph cuts the x-axis.
y = –2x + 1 9 5 1 –3 –7
Solution
(b) (a) x –3 0 3
y
y = 3(x – 1) –12 –3 6
8
y
6
6
4
4
2
2 y = 3(x _ 1)
x
_4 _2 O 2 4
_2 O x
_2 y = _ 2x + 1 _4 2 4
_2
_4
_4
_6
1 _8
3. (a) Draw the graph of y = x + 3 from x = – 4 to
2
x = 4. _ 10
(b) Does the point A(1, 3) lie on the graph?
_ 12
Solution
(a) x – 4 0 4
1
y= x +3 1 3 5 (b) The graph cuts the x-axis at (1, 0).
2
(b) When x = 1,
_1 x
1 O 1 2
y = (1) + 3
2
1 _1
= 3
2
= 3.5 _2
The point A(1, 3) does not lie on the graph.
_3 (b) y = _3
_2 y
20
y = 12 _ 3x
(b) All the lines in (a) pass through the point (0, 1).
10 (1, b)
(c) The constant term of each equation is 1.
(c, 5)
7. (a) Draw the following lines on the same diagram for (a, 0)
_2 O x
values of x from –3 to 3. 2 4 6
(i) y = –x
(ii) y = –x + 1
(iii) y = –x – 2 (b) When y = 0, the graph cuts the x-axis at (4, 0).
(iv) y = –x + 3 ∴ a = 4.
(b) What is the common property of the lines in (a)? The line x = 1 meets the graph at (1, 9).
(c) What is the common property of the equations of ∴ b = 9.
the lines in (a)?
The line y = 5 meets the graph at (2.3, 5).
Solution ∴ c = 2.3.
(a) x –3 0 3
y = –x 3 0 –3
y = –x + 1 4 1 –2
y = –x – 2 1 –2 –5
y = –x + 3 6 3 0
117
Level 3 (c)
9. The volume V cm3 of a gas, at 1 atmosphere pressure, y
formed in a chemical reaction at time t minutes is given
by V = 5t. 100
(a) Using a scale of 1 cm to 1 unit on the t-axis and
1 cm to 5 units on the V-axis, draw the graph of 80
V = 5t from t = 0 to t = 6. y = 20 + 10t
(b) Use your graph to find the time taken to produce 60
20 cm3 of the gas.
40
Solution
(a) 20
x 0 3 6
y 0 15 30 O
t
1 2 3 4 5 6 7 8
V
30 V = 5t (d) Draw the horizontal line y = 70 on the diagram in
(c) to meet the line. From the graph, the time is 5
20 minutes.
10 (e) When water reaches the temperature 100°C, it will
boil and the temperature will stay at 100°C until
t all water becomes vapour.
O 1 2 3 4 5 6
\ We cannot use the equation to find the
temperature of water after 10 minutes.
(b) Draw the horizontal line V = 20 on the diagram in
(a) to meet the line. From the graph, the time is
11. Water is drained from a burette. The height, H cm,
100 minutes.
of water at time t minutes is given by the function
H = 36 – 4t for 0 t 9.
10. The initial temperature of water in a kettle is 20 °C. On (a) Using a scale of 1 cm to 1 unit on the t-axis and
heating, the temperature of water increases by 10 °C 1 cm to 10 units on the H-axis, draw the graph of
per minute. Let y °C be the temperature of water after t H against t.
minutes. (b) Using the graph, find the time that the height of
(a) Copy and complete the following table of values water is 14 cm.
of t and y.
Solution
x 0 2 4 6 8 (a) t 0 5 9
y
H 36 16 0
(b) Write down the equation of the function
connecting t and y. H
(c) Using a scale of 1 cm to 1 unit on the t-axis and
40
1 cm to 20 units on the y-axis, draw the graph of
y against t. H = 36 _ 4t
30
(d) Using the graph, find the time that the temperature
of water is 70 °C. 20
(e) Can we use the equation in (b) to find the
temperature of water after 10 minutes? Explain 10
briefly.
O t
1 2 3 4 5 6 7 8 9
Solution
(a) x 0 2 4 6 8
y 20 40 60 80 100 (b) Draw the horizontal line H = 14 on the diagram
in (a) to meet the line. From the graph, the time is
(b) y = 20 + 10t 5.5 minutes.
1000
Solution S = 300 + 100t
900
(a) t 0 1 2 3
800
d = 180 – 60t 180 120 60 0
700
d 600
200 500
300
100
200
30 100
t
O 1 2 3 O t
1 2 3 4 5 6 7 8 9 10 11 12
(b) From the graph, when d = 30, t = 2.5.
Soo Meng is 30 km from the city after 2.5 hours.
Exercise 12.4
13. Describe a real-life example where 2 variables are in Level 1
a linear relationship and draw a graph to represent the 1. Find the gradients of the lines L1, L2 and L3 in the
relationship. following diagram.
y
L2
Solution
3 L1
The following graph shows the equation over 12 months
L3
where S represents savings in dollars in a savings account
2
and t represents time in months. From the graph, we can
see that the savings account opened with a balance of
1
$300 and has a monthly saving of $100.
x
_3 _2 _1 O 1 2 3
_1
Solution
3
Gradient of L1 =
3
=1
3
Gradient of L2 =
1
=3
–2
Gradient of L3 =
3
2
= –
3
119
2. Find the gradients of the lines L4, L5 and L6 in the Solution
following diagram. (a)
y x –2 –1 0 1 2
L4 y 7 5 3 1 –1
3 L5
2 x
_5 _4 _3 _2 _1 O 1 2 3
A 1 _1
C
x K(_ 3, _ 2) _2
_2 _1 O 1 2 3
_3 G(1, _ 3)
Solution
The vertices of ABC are A(–2, 1), B(2.5, 3) and 2
C(2.5, 1). (a) Gradient of OA = =1
2
2 4
Gradient of AB = = (b) Gradient of BC =
1
4.5 9
2
Since BC is vertical, gradient of BC is undefined.
–2 1
Since AC is horizontal, gradient of AC = 0. (c) Gradient of DE = =–
4 2
–5 5
4. (a) Copy and complete the table for the line (d) Gradient of FG = =–
6 6
y = –2x + 3.
(e) Gradient of HI = 0
x –2 –1 0 1 2 (f ) Since the line JK is vertical, gradient of JK is
y undefined.
–2 2 3 1
Gradient of AB = =– (b) Gradient of PQ = =
5 5 9 3
–4 4 3
Gradient of BC = =– Gradient of QR = =3
3 3 1
2 3 1
Gradient of CA = =1 Gradient of RS = =
2 9 3
3
Gradient of SP = =3
1
7. (a) Draw the quadrilateral ABCD whose vertices are
A(–1, –1), B(7, 3), C(4, 6) and D(0, 4). (c) The gradients of the opposite sides of a parallelogram
(b) Hence, find the gradients of the sides of ABCD. are equal.
(c) What type of quadrilateral is ABCD?
9. Find the gradients and y-intercepts of the lines L1, L2 and
Solution
L3 in the diagram below.
(a) y
C(4, 6) y
6
3
4 D(0, 4)
2
2 B(7, 3)
L3
1
_2 O x
2 4 6
A(_ 1, __1) _2 _1 O x
2 1 2 3
_1
L2
4 1 _2
(b) Gradient of AB = =
8 2
–3 _3
Gradient of BC = = –1 L1
3
2 1
Gradient of CD = =
4 2
5 Solution
Gradient of DA = =5
1 Gradient of L1
vertical change
(c) ABCD is a trapezium. =
horizontal change
–2
=
1
= –2
121
Gradient of L2 11. The distance y metres, of a boy at time t seconds from
vertical change a fixed point F is given by
=
horizontal change
y = 3 + 0.5t for 0 t 10.
2
=
2 (a) Draw the graph of y = 3 + 0.5t for 0 t 10.
= 1 (b) Find the gradient of the graph.
(c) What is the physical meaning of the gradient in
Gradient of L3 this case?
vertical change
=
horizontal change
Solution
2
= (a) t 0 2 4 6 8 10
3
y = 3 + 0.5t 3 4 5 6 7 8
Level 3 y
10. Paul is cycling along a horizontal road. His distance,
8
y km, from the starting point after t hours is given by y = 3 + 0.5t
the function y = 16t, for 0 , t , 3. 6
(a) Using a scale of 2 cm to 1 unit on the t-axis and
1 cm to 10 units on the y-axis, draw the graph of 4
y = 16t.
(b) Find the distance travelled by Paul in 2
(i) the first hour,
t
(ii) the second hour. O 2 4 6 8 10
(c) Interpret the physical meaning of the gradient of
the graph.
5
(b) Gradient of the graph = = 0.5
10
Solution
(a) t 0 1 2 3 (c) The gradient is the speed of the boy in m/s. That
is, the boy’s speed away from the fixed point F is
y = 16t 0 16 32 48
0.5 m/s.
y
12. The volume V cm3 of a piece of melting ice at time
50 t minutes is given by
V = 24 – 3t for 0 t 8.
40
(a) Using a scale of 1 cm to 1 unit on the t-axis and
y = 16t 2 cm to 5 units on the V-axis, draw the graph of
30 V = 24 – 3t for 0 t 8.
(b) Find the gradient of the graph.
20 (c) What is the physical meaning of the gradient in
this case?
10
(d) Interpret the physical meaning of the constant term
24.
t
O 1 2 3
123
Revision Exercise 12 (a) What is the gradient of the line?
In this exercise, use a scale of 1 cm to 1 unit for both the axes (b) Write down the coordinates of the point at which
unless specified otherwise. the line cuts the x-axis.
(c) Write down the coordinates of the point at which
1. Four points A, B, C and D are given as shown in the the line cuts the y-axis.
diagram below.
Solution
(a) State the coordinates of A, B, C and D.
–7
(b) Which point is in the second quadrant? (a) Gradient of the line AB =
7
(c) Find the gradient of
= –1
(i) CD, (ii) BC,
y (b) The line cuts the x-axis at (2, 0).
6
(c) The line cuts the y-axis at (0, 2).
C
2
Solution
(a) The coordinates are x
_4 _2 O 2 4 6
A(4, 0), B(–1, 0), C(–1, 5), D(– 4, –3).
_2
(b) Point C is in the second quadrant.
(c) (i) Gradient of AB = 0
(b) Hence, find the gradients of the sides of ABC.
(ii) Since BC is vertical, gradient of BC is
undefined.
Solution
8
(iii) Gradient of CD = (a) y
3
A(1, 6)
(d) (i) The equation of AB is y = 0. 6
2
2. The graph shows the line joining A(–3, 5) and
B(4, –2). x
_4 _2 O 2 4 6
y
_2
C(5, _2)
6
A(_3, 5)
4 3
(b) Gradient of AB =
5
2 –5
Gradient of BC =
9
x
_4 _2 O 2 4 6 5
=–
9
_2
B(4, _2) –8
Gradient of CA =
4
= –2
1
y= x+2 0 2 4 4000
3
y 3000
L
4 (2, 3) 2000
y= 3x
1
+2
2 1000
_6 _4 _2 O x x
2 4 6 O 1000 2000 3000 4000
125
(d) Draw the horizontal line y = 5000 on the diagram (c) When will the temperature of the meat be 0 °C?
in (c) to meet the line. (d) What is the physical meaning of 20 in the equation
The monthly salary of Joanne is $2400 (correct to y = 20 – 4t ?
the nearest hundred).
Solution
t
O 2 4 6 8
(b) Draw the horizontal line V = 5 on the diagram in
(a) to meet the line.
The required time is 4 minutes.
(c) When t = 0, V = 2.
The constant term is the initial volume of the crystal.
3
(d) The gradient is the rate of increase of the volume
4
of the crystal in cm3/min.
127
(b) the nth term = 17 + (n – 1) × 4 (b) Gradient of the line
= 17 + 4n – 4 –5 – 2
=
= 4n + 13 3+4
= –1
8. (a) The nth term of a sequence is given by n(n + 3). (c) The line cuts the x-axis at (–2, 0).
Find the values of the first 4 terms.
(b) Hence, express the nth term of the sequence 10. The price, $y, for x bowls of fish ball noodle is given by
3, 9, 17, 27, … in terms of n. y = 3x.
(a) Find the price of 4 bowls of fish ball noodle.
Solution
(b) Draw the graph of y = 3x for 0 < x < 4, using the
(a) the nth term = n(n + 3)
scale of 1 cm to 1 unit on both axes.
\ the 1st term = 1 × (1 + 3) (c) Find the gradient of the graph in (b).
=4
Solution
the 2nd term = 2 × (2 + 3)
(a) When x = 4,
= 10
y = 3 × 4
the 3rd term = 3 × (3 + 3) = 12
= 18 The required price is $12.
the 4th term = 4 × (4 + 3) (b) x 0 2 4
= 28
y = 3x 0 6 12
(b) The sequence
3, 9, 17, 27, … y
can be written as 12
4 – 1, 10 – 1, 18 – 1, 28 – 1 , …
\ the nth term = n(n + 3) – 1 10
8
9. (a) Using the scale of 1 cm to 1 unit on both axes, plot y = 3x
the points A(–4, 2) and B(3, –5) on an x-y plane 6
and draw the line connecting the points.
(b) Find the gradient of the line. 4
(c) Write down the coordinates of the point where the
line cuts the x-axis. 2
Solution x
O 1 2 3 4
(a) The graph shows the line connecting
A(–4, 2) and B(3, –5).
12
y (c) Gradient
=
4
A
2
=3
Review Exercise 3
128
Total number of votes cast 13. The following is a sequence of patterns formed by
= 468 ÷ 39% identical square tiles.
= 1200
(b) Number of votes received by Bill
= 1200 × 32%
= 384
(c) The required percentage n=1 n=2 n=3 n=4
468 – 384
= × 100% (a) Draw the pattern for n = 5.
384
7 (b) Let Tn be the number of tiles in the nth pattern.
= 21 %
8 (i) Write down the values of Tn for n = 1 to 5.
(ii) Express Tn in terms of n.
12. (a) When a piece of meat is taken out from a freezer,
Solution
its temperature is –15 °C. It is then placed in a
(a) The pattern for n = 5 is shown below.
microwave oven for defrosting. Its temperature
becomes –3 °C after heating for 90 seconds. If the
temperature of the meat rises at a constant rate, find
(i) the rate of increase of the temperature of the
meat in °C/s,
(ii) the temperature of the meat after heating for
2 minutes.
(b) In a 100-metre dash, the average speed of
John in the first 60 m was 8 m/s and he finished the n=5
last 40 m in 4.5 seconds. Find his average speed for (b) (i) T1 = 3
(i) the last 40 m, T2 = 8
(ii) the 100-metre dash. T3 = 15
T4 = 24
Solution T5 = 35
(a) (i) Rate of increase
(ii) If we add a square tile to the nth pattern, it
–3 – (–15)
= becomes a square with n + 1 tiles on a side.
90
2 \ Tn = (n + 1)2 – 1
= °C/s
15
(iii) If the top row of the nth pattern is cut and
(ii) Increase in temperature after attached to its right-hand side, the pattern will
Heating for 2 minutes become a rectangle of n by n + 2 tiles.
2
= × 2 × 60 \ Tn = n(n + 2)
15
= 16 °C
14. Jenny was 3 m from a lamppost. She began to run at a
Temperature after 2 minutes constant speed away from the lamppost. Her distance,
= –15 °C + 16 °C y m, from the lamppost after t seconds is given by the
= 1 °C function y = 2t + 3. The following diagram shows the
(b) (i) Average speed for the last 40 m graph of the function for 0 < t < 10.
40 y
=
4.5
= 8 m/s 25
129
(a) Find the time that she was 15 m from the lamppost.
(b) Find the gradient of the graph.
(c) What is the physical interpretation of the gradient
of the graph.
Solution
(a) When y = 15,
15 = 2t + 3
2t = 12
t = 6
(b) Gradient = 2
(d) The gradient is the speed of Jenny in m/s.
Solution
(a) Let 5x and 2x be the number of red ants and black
ants respectively.
Decreased number of red ants
= 5x × (1 – 30%)
= 3.5x
Percentage increase in black ants
3.5x – 2x
= × 100%
2x
= 75%
(b) 5x + 2x = 420
7x = 420
x = 60
Number of red ants after the change
= 3.5 × 60
= 210
Review Exercise 3
130
13 Simple Inequalities
Class Activity 1
Objective: To recognise the multiplication properties of inequalities.
Tasks
Fill in each blank with an inequality sign ‘.’ or ‘ ,’.
1. (a) 3 × 2 , 7 × 2 2. (a) –5 × 3 , –1 × 3
(b) 3 × 5 , 7 × 5 (b) –5 × 6 , –1 × 6
Questions
1. Observe the results above, what do you realise?
2. When both sides of an inequality are multiplied by a positive number, say 2, what would be the resulting inequality? Will
the inequality sign change?
a , b
a × 2 , b × 2
2a , 2b
When both sides of an inequality are multiplied by a positive number, the inequality sign will remain the same.
3. When both sides of an inequality are multiplied by a negative number, say , what would be the resulting inequality? Will
the inequality sign change?
a , b
a × (–3) . b × (–3)
–3a . – 3b
When both sides of an inequality are multiplied by a negative number, the inequality sign will change.
131
Try It (b) 14x 21
1 1
Section 13.1 × 14x × 21
14 14
1. Use an inequality to represent a relationship involving 3
the given variable in each of following statements. x
2
(a) The pH value, x, of a face cleanser is less than 7.
(b) The number of passengers, n, of a taxi should not The solution is represented on a number line as
exceed 4. follows:
(c) The area, y m2, of a flat is greater than 89 m2. 3
x<
(d) The volume, V cm3, of juice in a can is at least 2
375 cm3.
(e) The mass, m kg, of a car is more than 1100 kg. – 1 0 1 2 3
(f) The height, H cm, of a basketball player is at least
190 cm.
4. Solve each of the following inequalities.
Solution (a) –3x . 15 (b) –10x – 4
(a) x , 7 (b) n 4
(c) y . 89 (d) V 375 Solution
(e) m . 1100 (f) H 190 (a) –3x . 15
(–1) × (–3x) , (–1) × 15
2. Represent each of the following inequalities using a 3x , –15
number line and name three solutions for x. 1 1
× 3x , × (–15)
4 3 3
(a) x , –2 (b) x – x , –5
7
Solution
Let x be the possible number of cartons.
5x 40
1 1
(5x) × 40
5 5
x 8
The possible number of cartons is 1, 2, 3, 4, 5, 6, 7, 8.
Solution
Let x be the number of bowls of congee that the man
should eat per day.
70x 130
1 1
(70x) × 130
70 70
6
x 1
7
Solution
Let n be the number of members required.
Total monthly fees = $70 × n
= $70n
We require 70n 12000
12000
n
70
3
n 171
7
The minimum number of members is 172.
133
Exercise 13.1 Solution
Level 1 (a) 3 6
∴ x = 3 is a solution of x 6.
1. Use an inequality to represent a relationship involving
the given variable in each of the following statements. (b) 2 4
(a) The noon temperature, T °C, is more than 30 °C. ∴ x = 2 is NOT a solution of x 4.
(b) The height, H m, of a basketball player is at least (c) –1 –5
1.8 m. ∴ x = –1 is a solution of x –5.
(c) The combined length, width and height, C cm, of
an airline hand luggage should not exceed 115 cm. (d) –3 = –3
(d) The passing mark of a test is 50 marks. Larry got ∴ x = –3 is a solution of x –3.
x marks and he failed in the test.
(e) The time taken, t minutes, to travel from the Changi 4. Solve each of the following inequalities and represent
Airport to Jurong is more than 45 minutes. the solution on a number line.
(f) The daily sugar intake, m g, of a patient should not (a) 2x , 10 (b) 3x . 18
be more than 36 grams. (c) 4x 0 (d) 5x –20
(e) –6x . 9 (f) –8x , 12
Solution (g) –14x –35 (h) –16x –88
(a) T . 30
(b) H 1.8 Solution
(c) C 115 (a) 2x , 10
(d) x , 50 1 1
(e) t . 45 × 2x , × 10
2 2
(f) m 36 x , 5
135
(d)
–13x –52 Solution
13x –52 (a) The amount Jinlan gets = $15 × x
x 4 = $15x
The smallest integer x is 4. (b) 15x 75
1
x × 75
15
9. (a) Find the smallest prime number which satisfies the x 5
inequality 3x . 64. She works for less than 5 hours.
(b) Find the largest prime number which satisfies the
inequality – 4x – 49.
2. The price of a concert ticket is $50.
Solution (a) Find the total price for x tickets in terms of x.
(a) 3x . 64 (b) Henry has $200 to spend on tickets. Find the
64 possible number of concert tickets he can buy.
x .
3
1 Solution
x . 21
3 (a) The total price for x tickets = $50 × x
The smallest prime number x is 23. = $50x
– 4x – 49
(b) (b) 50x 200
–49 1
x < x
× 200
–4 50
1
x < 12 x 4
4
The largest prime number x is 11. The possible numbers of concert tickets that he can
buy are 1, 2, 3 and 4.
10. Write down an inequality of the form ax b, where 3. A pack of instant noodles contains 800 mg of sodium.
a ≠ 1, such that its solution is x 7. (a) Express the amount of sodium in x packs of instant
noodle in terms of x.
Solution (b) If the recommended maximum of daily intake of
One possible inequality is sodium is 2300 mg, find the maximum number of
5x 35. packs of instant noodles that you should eat on a
The solution is given by day so that the sodium intake is less than 2300 mg.
1
x × 35. Solution
5
(a) The amount of sodium
i.e. x 7.
= 800x mg
(b) 800x 2300
11. Write down an inequality of the form ax , b, where 23000
a ≠ 1, such that its solution is x . –3. x
800
27
Solution x
8
One possible inequality is –5x < 15. The maximum number of packs is 2.
The solution is given by
15
x > 4. The mass of a book is 400 g.
–5
i.e. x > –3. (a) Find the total mass of x books in terms of x.
(b) If the mass of a pile of such books is more than
7 kg, find the minimum number of books in the pile.
6. The time taken by a carpenter to make a chair is 3 hours. 10. A school needs to raise $45 000 to equip a multimedia
Find the maximum number of chairs he can make if he room. The school has 800 students.
has 17 hours to make them. (a) If each student donates the same amount, how
much would each student need to donate to meet
Solution the target?
Let x be the number of chairs made. (b) If each set of computer costs $1700, find the possible
3x 17 numbers of sets of computers that can be bought
17
x with $45 000.
3
52
x Solution
3
(a) Let $x be the amount that each student needs to
The maximum number of chairs made is 5. donate.
800x = 45 000
7. A car park in the city area charges $4 per hour and part 45 000
x =
thereof. If you have $25, find the maximum number of 800
hours that you can park your car in this car park. x = 56.25
Each student needs to donate $56.25.
Solution
Let x be the number of hours of parking. (b) Let y be the possible numbers of sets of computers
4x < 25 that can be bought.
25 1700y 45 000
x <
4 1
61
y × 45 000
1700
x <
4
450
The maximum number of hours of parking is 6. y
17
8
y 26
8. The selling price of a bouquet of flowers is $25. Find the 17
minimum number of bouquets of flowers a florist must The possible numbers of sets of computers that can
sell if she wants her total sales to be more than $360. be bought are 1, 2, 3, ..., 26.
Solution
11. The length of a rectangle is twice its width. Let the width
Let x be the number of bouquets of flowers sold.
be x cm.
25x 360
(a) Express the perimeter of the rectangle in terms of
1
x × 360 x.
25
72
(b) If the perimeter of the rectangle is 102 cm, find its
x width.
5
2 (c) If the perimeter of the rectangle is less than
x 14 120 cm, find the range of possible values of the
5
The minimum number of bouquets of flowers she must width.
sell is 15.
Solution
(a) Length of the rectangle = 2x cm
Level 3 Perimeter of the rectangle = 2(2x + x)
= 6x cm
9. A fashion boutique offers 20% discount for its dresses.
If your budget for a dress is $250, find the range of the
marked prices of dresses that fits your budget.
137
(b) 6x = 102 Solution
102 Let x years old be the age of De Ming.
x =
6 His father’s age = 4x years.
x = 17 4x + x 50
5x 50
The width of the rectangle is 17 cm.
50
x
(c) 6x 120 5
1 x 10
x × 120
6 Since De Ming’s father should be at least 18 years old
x 20 when De Ming was born, the possible ages of De Ming
are 6, 7, 8, 9 or 10 years old.
The range of the width is 0 width 20.
(b) 5x 75
y cm
1
x × 75
5
Solution x 15
3 x > 15
(a) y x
2
(b) Total length 13 14 15 16 17 18
= 2x + 4y
3
= 2x + 4 x (c)
–7x < 21
2
7x < 21
= 8x cm x < –3
(c) 8x 720 x < –3
720
x
8
– 6 –5 – 4 –3 –2 –1
x 90
Maximum value of x = 90 (d)
–9x . –8
Hence, maximum width 9x , –8
3 9
= × 90 x
2 8
= 135 cm 1
x 1
8
1
13. The age of De Ming’s father is 4 times that of De Ming. x,1
8
The sum of the ages of De Ming and his father is not
more than 50 years. Find two possible ages for De Ming.
– 2 –1 0 1 2 3
2 5 7 77
7 11
∴ 5390 = 2 × 5 × 72 × 11
(b) 7x 105
105
x
7
x 15
The required factors are 1, 2, 5, 7, 10, 11, 14.
139
V Solution
(a) 4x 60
30 60
x
4
x 15
25 The maximum percentage discount is 15%.
V = 5t
(b) (i) Selling price of the lamp
20 = $200 × (100% – 15%)
= $170
15 (ii) Marked price of the rack
= $51 ÷ (100% – 15%)
10 = $60
Class Activity 1
Objective: To make the connection between the area of a parallelogram and that of a rectangle.
D C D C D C
height
A B A B N M
N MN AB
base
A parallelogram is a quadrilateral with parallel opposite sides. The diagram on the left above shows a parallelogram ABCD
in which AB // DC and AD // BC. The perpendicular DN from D to AB is called the height of the parallelogram corresponding
to the base AB. Imagine that we cut the parallelogram along DN as shown in the middle diagram and then place ADM to the
other side to form the quadrilateral NMCD as shown on the right.
Questions
1. Name the quadrilateral NMCD.
NMCD is a rectangle.
Area of NMCD = NM × DN
AB = NM
4. What is the relationship between the area of ABCD and the area of NMCD?
5. If you are tasked to suggest a formula for the area of the parallelogram ABCD, what would that be?
6. If a parallelogram PQRS is given as shown on the right, how could you cut and S R
assemble it into a rectangle with PQ as a side?
P Q
Draw a perpendicular PN from P to SR. Cut along PN and place ∆PSM to the other side to form quadrilateral PQMN.
141
Class Activity 2
Objective: To identify the height corresponding to any given side of a triangle or a parallelogram that is taken as the base.
Questions
1. Identify the height corresponding to the side AB in each of the following triangles.
2. Draw the height corresponding to the base PQ in each of the following triangles.
(a) (b) (c) Q
P P
R
P
Q R
R
Q
3. Identify the height corresponding to the side AB in each of the following parallelograms.
D E
A B A
E
E
4. Draw the height corresponding to the base PQ in each of the following parallelograms.
(a) (b) Q
(c)
R Q Q
P
R P R
S P S
S
A a B a a B, G b H
E F A
h h h h
D C H G D E
b b b C, F a
trapezium trapezium
In the diagram above, ABCD and EFGH are two identical trapeziums in which AB = EF = a, DC = HG = b and
the height of each trapezium is h. Let us place these two trapeziums together to form the quadrilateral AHED.
Questions
1. Can you name the quadrilateral AHED?
AHED is a parallelogram.
(a + b)
143
Class Activity 4
Objective: To identify the height corresponding to the parallel sides of a trapezium.
Questions
1. Identify the height corresponding to the side AB in each of the following trapeziums.
A E
D
A E F B
B E
B
A
C
2. Draw the height corresponding to the side PQ in each of the following trapeziums.
P
R R
P Q
Q
S P
Tasks
You are given 4 identical table tops, each in the shape of a trapezium as shown below. The dimensions are in centimetres. These
table tops are arranged to form a larger table for a class activity.
D 40 C
50 40
A 70 B
1. What shapes can be formed? Sketch the shapes that you can make.
Suggested Answer:
Shape Perimeter
110
110
110
110
145
Try It! Solution
(a) Let r cm be the radius of the circle.
Section 14.1
2rr = 12r
1. The length and breadth of a rectangle are 40 cm and r = 6
10 cm respectively. A square of side y cm has the same The radius is 6 cm.
area as the rectangle. Find
(a) the area of the rectangle, (b) Area of the circle = r × 62
(b) the value of y, = 36r cm2
(c) the perimeter of the square.
4. In the figure, PQRS is a parallelogram, RS = 12 cm,
Solution PS = 6 cm and SM = 5 cm. It is given that RN is
(a) Area of the rectangle = 40 × 10 perpendicular to PS produced and PQ SM.
= 400 cm2
(b) Area of the square = Area of the rectangle
y2 = 400
y = 400
= 20
(c) Perimeter of the square = 4y cm
= 4 × 20 cm
= 80 cm (a) Identify the height corresponding to the base PS
in the parallelogram PQRS.
(b) Find the area of the parallelogram.
2. In the figure, AB = 5 cm, BC = 4 cm and AC = 3 cm.
(c) Find the length of RN.
Solution
(a) Height = NR
(b) Area of PQRS = RS × SM
= 12 × 5
= 60 cm2
(c) PS × RN = Area of PQRS
Find 6 × RN = 60
(a) the area of ABC, RN = 10 cm
(b) the length of CD.
5. In the figure, ABCD is a square of side 24 cm. M and N
Solution are points on AB and DC respectively such that MBND
1
(a) Area of ABC = × BC × CA is a parallelogram and MB = 12 cm.
2
(a) Find the area of MBND.
1 (b) Find the ratio of the area of MBND to that of ABCD.
= ×4×3
2
(c) How is the ratio in (b) related to the ratio of MB
= 6 cm2 to AB?
1 D N C
(b) × AB × CD = 6
2
1
× 5 × CD = 6
2
12
CD = 24
5
= 2.4 cm
Area of ABE =
1
Area of ABCD 9. The side wall ABCD of a shed is in the shape of a
5 trapezium. Given that AB = 2.5 m, BC = 3 m and the
1
× BE × h =
1
× 12 × h area of ABCD is 5.5 m2, find the length of AD.
2 5
24 E
BE = D C
5
= 4.8 cm 36
A B
50
Section 14.2
7. In the figure, PQRS is a trapezium in which PQ = 14 cm, Solution
SR = 8 cm and PS = 10 cm. Find the area of PQRS. Area of ABCD =
1
× (AD + 3) × 2.5
2
1
× (AD + 3) × 2.5 = 5.5
2
AD + 3 = 4.4
AD = 1.4 m
Solution
Section 14.3
10. In the figure, STP is a semicircle R
1
Area of PQRS = × (8 + 14) × 10 of diameter 30 cm, PQRS is a S
2 U
parallelogram, PQ = 16 cm and 15
= 110 cm2
SU = 15 cm. Take r to be 3.142
and giving your answers correct T 30
8. In the figure, the trapezium ABCD is a flowerbed in which to 3 significant figures, find
AB = 3 m and DC = 5 m. If the area of the flowerbed is (a) the perimeter of the figure, Q
10 m2, find B C (b) the area of the figure. 16
P
5
A
147
Solution 12. In the figure, ABCD and ABEF are two equal
(a) Perimeter of the figure parallelograms with common side. AB = 20 cm and the
= semicircle STP + PQ + QR + RS height of each parallelogram with respect to the base AB
1 is 30 cm. G, the intersection of AD and BE, is 16 cm
= × r × 30 + 16 + 30 + 16
2 from AB. Find
= 15(3.142) + 62
= 109.13 F E D C
= 109 cm (correct to 3 s.f.)
G
(b) Area of the figure 30
= Area of semicircle STP 16
+ Area of parallelogram PQRS
A 20 B
1
= × r × 152 + 30 × 15
2
= 803.475
(a) the area of ABCD,
= 803 cm2 (correct to 3 s.f.)
(b) the area of ABG,
(c) the area of the figure.
11. The figure shows the shape of a cross-section of a stool.
AD = 40 cm, EH = 30 cm, BC = 24 cm, FG = 16 cm, Solution
the distances of BC and FG from the ground are 25 cm (a) Area of ABCD
and 15 cm respectively. Find the area of the figure. = 20 × 30
= 600 cm2
B 24 C
(b) Area of ∆ABG
F 16 G 1
25
= × 20 × 16
2
15 = 160 cm2
A E H D (c) Area of DCBG
30 = Area of ABCD – Area of ABG
40 = 600 – 160
= 440 cm2
Solution Area of figure
Area of ABCD =
1
× (24 + 40) × 25 = Area of DCBG + Area of FEGA + Area of ABG
2 = 440 + 440 + 160
= 800 cm2 = 1040 cm2
1
Area of EHGF = × (16 + 30) × 15
2
= 345 cm2
Area of the figure = (800 – 345)
= 455 cm2
4 9 6
N
P Q A 4 B E 6 F
A E B
5
(a) Find the ratio of the area of EFG to that of ABCD.
(b) If the area of EFG is 24 cm2, find the area of
(c) (d)
ABCD.
K H W 12 T
Solution
7 X (a) Let h cm be the height from D to AB.
Z
3
Area of EFG : Area of ABCD
F 7 G 1
Y = ×6×h:4×h
2
Solution =3:4
(a) Area of ABCD = 5 × 4 4
= 20 cm2 (b) Area of ABCD = 24 ×
3
(b) Area of PQRS = 6 × 9 = 32 cm2
= 54 cm2
(c) Area of FGHK = 7 × 7 4. In the figure, ABCD is a parallelogram and AD = 18 cm.
= 49 cm2 E is a point on AD produced such that the area of ABE
5
(d) Area of WXYZ = 3 × 12 is that of ABCD.
6
= 36 cm2
E
Solution
AB DE BC DF Area of ABCD
(a) Let h cm be the height from B to AD.
(a) 10 6 8
5
(b) 18 9 12 Area of ABE = × Area of ABCD
6
(c) 15 8 10 1
× (18 + DE) × h =
5
× 18 × h
2 6
(d) 9 6 72
18 + DE = 30
DE = 12 cm
Solution
(b) 18h = 450
AB DE BC DF Area of ABCD h = 25
(a) 10 6 8 7.5 60 The height is 25 cm.
(b) 18 6 9 12 108
(c) 15 8 12 10 120
(d) 8 9 6 12 72
149
5. In the figure, ABCD is a rhombus, AD = 13 cm, 7. The picture below shows the national flag of the
AC = 24 cm and BD = 10 cm. Republic of the Congo. ABCD is a rectangle and APCQ
is a parallelogram. The ratio of AB to BC is 3 : 2 and
D
1
AP = AB. If AB = 48 cm, find
3
A C
E
D Q C
B
The height is 9
3
cm. = 16 cm
13
(b) Area of APCQ = AP × BC
= 16 × 32
Level 3 = 512 cm2
512
6. In the figure, ABCD is a vertical wall 2 m high. It casts (c) The required percentage = × 100%
48 × 32
a shadow ABEF on the ground. ABEF is a parallelogram
1 1
in which EN is 1 times the height of the wall. = 33 %
2 3
D C
2 8. In the figure, ABCDEF is a signboard in which
A B ABCF and FCDE are parallelograms, FN = 10 cm and
N
DM = 6 cm.
F E E D
6
(a) Find the length of EN. F C
M
(b) If the area of the wall is 7.6 m2, find the area of
the shadow. 10
A N B
Solution
1
(a) The height EN = 2 × 1 (a) Find the ratio of the area of ABCF to that of FCDE.
2
=3m (b) If the area of the signboard is 320 cm2, find
(i) the area of ABCF,
7.6
(b) AB = (ii) the length of FC.
2
= 3.8 m Solution
Area of the shadow = 3.8 × 3 (a) Area of ABCF : Area of FCDE
= 11.4 m2 = FC × FN : FC × DM
= FN : DM
= 10 : 6
=5:3
A B A B
Exercise 14.2
1 cm Level 1
1. Find the area of each trapezium.
1 cm (a) D (b)
10 C K 8 H
Solution
D F C E 9 6
E
A B F 5 G
14
R (c) (d)
A B R
S Z 5 Y
S
Q 3
7 9
4
P T
P
W 11 X T
Q
In the above diagram, ABCD, ABEF and PQRS are
parallelograms of areas 6 cm2 each. Solution
1
(a) Area of ABCD = (10 + 14) × 9
10. The figure shows one way to divide a parallelogram into 2
two identical parts by a line segment. Find as many ways = 108 cm2
as possible to divide the parallelogram into two identical (b) Area of FGHK = (5 + 8) × 6
1
parts. 2
= 39 cm2
D C 1
(c) Area of PQRS = (3 + 7) × 4
2
= 20 cm2
1
(d) Area of WXYZ = (5 + 11) × 9
2
A B = 72 cm2
A E B
b
151
a b h Area of ABCD (b) Let h cm be the height from A to DC.
10h = 60
(a) 7 cm 10 cm 8 cm h = 6
(b) 5 cm 9 cm 42 cm2 1
Area of AMCD = (5 + 10) × h
(c) 13 cm 14 cm 231 cm2 2
1
(d) 11 cm 10 cm 85 m2 = (5 + 10) × 6
2
= 45 cm2
Solution
5
= 15 : 21
=5:7
5
A B
(c) Area of ANMD = 200 ×
3 E 5 5+7
1
Solution = 83 cm2
3
(a) DC = AB (opp. sides of rectangle)
6 + CF = 3 + 5
∴ CF = 2 cm 6. In the figure, ABCD is a trapezium, D 10 C
1
(b) Area of AECF = (3 + 2) × 5 AB = 18 cm and CD = 10 cm. The
2
area of ACD = 75 cm2.
= 12.5 cm2
Find the area of ABCD.
A B
4. In the figure, ABCD is a parallelogram, M is the midpoint 18
of AB and CD = 10 cm. Solution
D 10
C Area of nACD :
DC : AB = Area of nABC
75 10
=
Area of ABC 18
A M B
18
(a) Find the length of AM.
∴ Area of ABC = 75 ×
10
(b) If the area of ABCD is 60 cm2, find the area of = 135 cm2
AMCD.
Area of ABCD = Area of ACD + Area of ABC
Solution = 75 + 135
(a) AB = DC (opp. sides of //gram) = 210 cm2
2AM = 10
AM = 5 cm
D 1
Area of ABMG = × (54 + 27) × 18
0.75 m 2
= 729 cm2
0.45 m
(c) Red area = Area of ABMG
= 729 cm2
A 0.4 m B 729
The required percentage = 54 × 36
× 100%
(a) Find the perimeter of the frame. = 37.5%
(b) Find the area enclosed by the frame.
(c) If the cost of the frame is $30 per metre and the
cost of the glass panel used to fit the window is 9. In the figure, AB, CD and 30
$150 per m2, find the cost of making the window. EF are three parallel rungs E F
of a ladder which are of
h
Solution equal distance, h cm, apart.
(a) Perimeter of the frame AB = 50 cm, CD = 40 cm and 40
C D
= 0.4 + 0.75 + 0.5 + 0.45 EF = 30 cm. The area of ABDC
= 2.1 m is 1260 cm2. Find h
(a) the value of h, 50
(b) Area enclosed by the frame (b) the area of CDFE, A B
1
= (0.45 + 0.75) × 0.4 (c) the ratio of the area of
2
CDFE to that of ABDC.
= 0.24 m2
(c) The cost of making the window
= $(30 × 2.1 + 150 × 0.24) Solution
= $99 (a) Area of ABDC = 1260
1
× (40 + 50) × h = 1260
2
8. The figure shows the national flag of the Czech Republic. 45h = 1260
G is the point of intersection of the diagonals of the h = 28
rectangle ABCD and M is the midpoint of BC.
1
The ratio of the length of AB to that of BC is 3 : 2. If (b) Area of CDFE = × (40 + 30) × 28
2
BC = 36 cm, find = 980 cm2
A B (c) Area of CDFE : Area of ABDC
= 980 : 1260
G = 7 : 9
M
153
Solution (b) Join C and E.
(a) EF = 20 – x – x
D
= (20 – 2x) cm
3 15 8
(b) (i) Area of CDEF = × Area of ABCD
4
=
3
× 20 × 15 E C
4
= 225 cm2 6 6
1 A 17 B
(ii) (20 + 20 – 2x) × 15 = 225
2
40 – 2x = 30 Perimeter of the figure
2x = 10 = 17 + 6 + 8 + 15 + 6
x = 5 = 52 cm
Area of the figure
= Area of ABCE + Area of CDE
Exercise 14.3 1
= 17 × 6 + × 8 × 15
Level 1 2
1. Find the perimeter and area of each figure, where the = 162 cm2
unit of length is centimetre. (c) Perimeter of the figure
(a) (b) = 27 + 23 + 5 + 23 + 15 + 13
1 D = 106 cm
D C
1 15 8
Area of the figure
F 2
= Area of ABEF + Area of BCDE
E C
E 1
1 = (15 + 27) × 5 + 5 × 20 cm2
6 6 2
A 3 B
A 17 B = 205 cm2
FE = 3 – 1
= 2 cm (c) (d)
Leave your answers in
Perimeter of the figure terms of r.
= 3 + 2 + 1 + 1 + 2 + 1 F1 E G
G 15 F
= 10 cm 3 2.8
C
Area of the figure 2
6 D C 13 12 13
= Area of AGEF + Area of GBCD
3 2.8
= 2 × 1 + 2 × 1
A B
A 5 B D 5 E
= 4 cm2 5
155
(b) Height from G to EF Solution
1 Area of the wall
= BC
2 = Area of square ABFG + Area of parallelogram BCEF
1 + Area of semicircle CDE
= (7 + 10 + 7) 2
2 1 1
= 1 × 1 + 1 × 4 + ×r×
= 12 cm 2 2
1 = 5.39 m2 (correct to 3 s.f.)
Area of GEF = × 10 × 12
2
= 60 cm2 8. The figure shows a cross-section of an iron rail. The top
Area of the shape part consists of a rectangle and two semicircles at the
= Area of ABCD – 4 × Area of GEF ends. The bottom part is a trapezium. These two parts
= 24 × 24 – 4 × 60 are joined by a rectangle in the middle. Taking r to be
= 336 cm2 3.142, find
(a) the perimeter,
6. In the figure, ABC is a right-angled triangle and ABEF (b) the area of the cross-section.
is a parallelogram. AC and BE intersect at D. The unit 20
of length is centimetre. Find
10 10
(a) the area of ABC,
(b) the area of ABD,
(c) the area of the parallelogram ABEF, 15
(d) the area of the figure. 10 10 10
C
13 13
F E
12
D 40
18
14
10
Solution
(a) Perimeter of the cross-section
A
15
B = 40 + 13 + 10 + 15 + 5 + 2r × 5 + 20 + 5 + 15
+ 10 + 13
= (146 + 10r) cm
Solution
= 177 cm (correct to 3 s.f.)
1
(a) Area of ABC = × 18 × 15
2 (b) Area of the cross-section
= 135 cm2 1
= (40 + 30) × 12 + 10 × 15 + 20 × 10 + r × 52
1 2
(b) Area of ABD = × 15 × 10
2 = 770 + 25r
2
= 75 cm = 849 cm2 (correct to 3 s.f.)
(c) Area of parallelogram ABEF
= 15 × 14 9. The figure shows the cross-section of a funnel. ABFG
= 210 cm2 is a trapezium of height 5 cm. BCEF is a rectangle with
CE = 10 cm and EF = 6 cm. CDE is a triangle such that
(d) Area of the figure
the perpendicular distance from D to CE is 4 cm. Find
= Area of ABC + Area of ABEF – Area of ∆ABD
(a) the area of BCDEF,
= 135 + 210 – 75
(b) the length of AG if the area of ABFG is equal to
= 270 cm2
the area of BCDEF.
A G
Level 3 5
B F
7. The figure shows the wall of a staircase. ABFG is a
6
square of side 1 m. BCEF is a parallelogram in which
10
the perpendicular distance from C to BF is 4 m. CDE is C E
4
a semicircle. Taking r to be 3.142, find the area of the
D
wall, giving your answer correct to 3 significant figures.
Solution
E
(a) Area of BCDEF
G F D = Area of BCEF + Area of ∆CDE
1 1
C = 10 × 6 + × 10 × 4
2
A 1 B 2
4 = 80 cm
D 6 6 6 Ch
F E
5
A P Q B
Solution
(a) Area of ABCD
= AD × DC (ii)
= AD × (DF + FE + EC)
= 5 × (6 + 6 + 6)
= 5 × 18
= 90 cm2
(b) Area of PQEF
= DA × FE
=5×6 (b) The perimeter of each figure = 5 × 12
= 30 cm2 = 60 cm
(c) Area of FERS = Area of PQRS – Area of PQEF Area of figure (i) = 15 × 15
= 6 × h – 30 = 225 cm2
= 6h – 30 Area of figure (ii) = 20 × 5 + 10 × 5
= 150 cm2
Area of figure = Area of ABCD + Area of FERS
90 + 6h – 30 = 150 (c) From (b), the perimeter will remain the same but
6h = 90 the area will differ.
h = 15 cm
11. Describe two items used in our daily life which are
composite plane figures.
Solution
• An oval sports field is composed of a rectangle and
two semicircles.
• A direction arrow on a lane of a road is composed of
a triangle and a rectangle.
157
Revision Exercise 14 (c) Length of a side of the square
1. In the figure, ABCD is a D F 15 C = 5
rectangle, AB = 32 cm, 8
BE = 13 cm, EC = 8 cm and = 2.24 cm (correct to 3 s.f.)
E
CF = 15 cm. Find 13
(a) the perimeter of ABCD, 3. In the figure, the shaded region is formed by three
A B
(b) the area of ABCD, 32 semicircles, AB = 12 cm and BC = 6 cm. Find, in terms
(c) the area of AEF. of r,
(a) the perimeter of the shaded region,
Solution (b) the area of the shaded region.
(a) AD = 8 + 13
= 21 cm
Perimeter of ABCD = 2(32 + 21)
= 106 cm
A 12 B 6 C
(b) Area of ABCD = 32 × 21
= 672 cm2 Solution
(c) DF = 32 – 15 (a) The radii of the semicircles are 9 cm, 6 cm and
= 17 cm 3 cm.
Area of AEF Perimeter of the shaded region
= Area of ABCD – Area of ABE =r×9+r×6+r×3
– Area of CEF – Area of ADF = 18r cm
1 1
= 672 – × 32 × 13 – × 8 × 15 (b) Area of the shaded region
2 2
1 1 1
–
1
× 21 × 17 = r × 92 – r × 62 – r × 32
2 2 2
2
= 672 – 208 – 60 – 178.5 = 18r cm2
= 225.5 cm2
4. The figure shows the national flag of Kuwait. The ratio
of its width to its length is 1 : 2. The height of the black
2. The figure shows a ‘T’ shape with right-angled 1
vertices. CD = DE = FG = GH = 1 cm, AH = 2 cm and trapezium is of the length of the national flag. The
4
EF = 3 cm. horizontal bands, coloured in green, white and red, are
(a) Find the perimeter of the figure. of equal width. It is given that the width of the flag is
(b) Find the area of the figure. 36 cm. Find
(c) If a square is equal in area to the figure, find the (a) the area of the black trapezium,
length of a side of the square. Give your answer (b) the area of the green trapezium,
correct to 3 significant figures. (c) the area of the green trapezium as a percentage of
the area of the flag.
F 3 E
1 1
H C
G 1 1 D
A B
Solution
Solution (a) Length of the flag = 36 × 2
(a) AB = 3 – 1 – 1 = 72 cm
= 1 cm Height of the black trapezium
Perimeter of the figure 1
= 1 + 2 + 1 + 1 + 3 + 1 + 1 + 2 = × 72
4
= 12 cm = 18 cm
(b) Area of the figure Width of each horizontal band
= Area of ABCH + Area of DEFG 1
=2×1+1×3 = 36 ×
3
= 5 cm2 = 12 cm
159
Solution (b) Let h cm be the height of the trapezium AMNC
(a) Perimeter of ABCD = 2(30 + 20) 1
× (17 + 34) × h = 180
= 100 cm 2
51
h = 180
(b) Area of ABCD = 20 × 27 2
2
= 540 cm2 h = 180 ×
51
(c) Let h be the height from A to CD. 1
h = 7
17
Area of ADE + Area of ABCE = 540 cm2
1
1
× 10 × h + (30 + 20) × h = 540
1 The height is 7 cm.
17
2 2
5h + 25h = 540
540 10. The figure shows a logo which is formed by two identical
h =
30 parallelograms ABCG and AEFG. The parallelograms
= 18 have a common side AG of 20 cm and a common height
30 cm. CG and AE intersect at D. The area of the logo
1
Area of ABCE = (30 + 20) × 18 is 1020 cm2. Find the perpendicular distance from D to
2
AG.
= 450 cm2
A 20 G
(d) Let h cm be the height from A to CD.
Area of ADE : Area of ABCE
1 1
= × 10 × h : (30 + 20) × h 30
2 2
D
= 10 : 50
=1:5 B C E F
Alternatively,
Area of ADE : Area of ABCE Solution
= 90 : 450 Let h cm be the perpendicular distance from D to AG.
=1:5 Area of AEFG = 20 × 30
1 = 600 cm2
Note: Area of ADE = × 10 × h
2
2 × Area of AEFG – Area of ADG = Area of the logo
1
= × 10 × 18 1
2 2 × 600 – × 20 × h = 1020
2
2
= 90 cm 1200 – 10h = 1020
10h = 180
9. The figure shows a right-angled ABC, M and N h = 18
are points on the sides AB and BC respectively. The required distance is 18 cm.
AM = MB = 8 cm, BN = NC = 15 cm, MN = 17 cm,
AC = 34 cm and MN // AC. Find
(a) the area of the trapezium AMNC,
(b) the height of the trapezium AMNC.
A
8
34
M
17
8
B N C
15 15
Solution
(a) Area of trapezium AMNC
= Area of ∆ABC – Area of ∆MBN
1 1
= × (8 + 8) × (15 + 15) – × 8 × 15
2 2
= 180 cm2
Class Activity 1
Objective: To visualise and draw sketches of 3D shapes from different views.
Task
Your teachers will place some large 3D objects, such as cuboids and prisms, in front of the class.
1. Look at the objects from your seat and sketch a view of each object. You may use a camera device to take pictures of the
objects and help you sketch the views.
Cube
Cuboid
Triangular prism
Cylinder
161
2. Identify those right angles on your own sketches.
3. What do you notice when you compare your sketches with a classmate who is seated at a different place in the classroom?
The sketches are different. The view from another orientation is different.
Class Activity 2
Objective: To visualise and draw the nets of cubes, cuboids, prisms and cylinders.
Tasks
2. Cut up a container so that you can lay it flat. Draw the net of the container.
Cube
3. How else would you cut up the same container to get a different net? Draw this net.
Cube
4. What do you observe when you compare your nets with those of your classmates for the containers of the same shape?
top D E
C
8
6
N
A B
14
Solution
1
Area of ABC = × 14 × 6
2
front
= 42 cm2
Solution
Volume of the prism = 42 × 8
(a) = 336 cm3
4. The figure shows a swimming pool with a rectangular
(b) surface ABCD (not drawn to scale). It is 25 m long, 18
m wide, 3 m deep at the deep end and 2 m deep at the
shallow end. Find the volume of the swimming pool.
D C
18
A 25 B
2. The figure shows a prism whose end faces are H
G
2
parallelograms, where the unit of length is centimetre. 3
Draw a net of the prism.
F
E
2.5 Solution
Base area = Area of ABFE
1
2 = × (3 + 2) × 25
2
= 62.5 m2
1
Volume of the prism = 62.5 × 18
= 1125 m3
Solution
163
6. A rectangular block of copper is 10 cm by 9 cm by (b) Perimeter of the base = Perimeter of ABC
4 cm. = 5 + 12 + 13
(a) Find the volume of the copper block. = 30 cm
(b) Find the total surface area of the copper block. Total surface area of the prism
(c) If the copper block is melted and recast into a cube, = 30 × 14 + 2 × 30
find the length of a side of the cube. Give your = 480 cm2
answer correct to 3 significant figures.
Solution
13. The external and internal diameters of a cylindrical water
pipe are 4 cm and 3.2 cm respectively and the length of (a) (i) Internal total surface area
the pipe is 5 cm. = 2(5.90 × 2.35 + 5.90 × 2.39
(a) Find the volume of material used in making the + 2.35 × 2.39)
pipe. = 67.165
(b) Find the total surface area of the pipe. = 67.2 m2 (correct to 3 s.f.)
(c) If water flows through the pipe at 10 cm/s, find the (ii) Internal total surface area
volume of water delivered in 30 seconds. = 67.165 × 10 000 cm2
Give your answers correct to 3 significant figures. = 671 650 cm2
= 672 000 cm2 (correct to 3 s.f.)
4 cm (b) (i) Internal volume
= 5.90 × 2.35 × 2.39
= 33.137 35
3.2 cm = 33.1 m3 (correct to 3 s.f.)
5 cm (ii) Internal volume
= 33.137 35 × 1 000 000 cm3
= 33 137 350 cm3
= 33 100 000 cm3 (correct to 3 s.f.)
165
15. A cylindrical drum 25 cm high has a base radius of (b) Area of the cross-section
30 cm. = 13 × 18 +
1
× r × 132
(a) Find its curved surface area 4
(i) in cm2, = 366.73
(ii) in m2. = 367 cm2 (correct to 3 s.f.)
(b) Find its volume
(c) Total surface area of the bread
(i) in cm3,
= 82.42 × 40 + 2 × 366.73
(ii) in m3.
= 4030 cm2 (correct to 3 s.f.)
Give your answers correct to 3 significant figures.
(d) Volume of the bread
Solution = 366.73 × 40
(a) (i) Its curved surface area = 14 700 cm3 (correct to 3 s.f.)
= 2r × 30 × 25
= 4712.39
17. The figure shows a prism whose cross-section is formed
= 4710 cm2 (correct to 3 s.f.)
by cutting out CDF from the parallelogram ABFE.
(ii) Its curved surface area AB = 25 cm, DE = 15 cm and BH = 30 cm. The heights
1 from C to EF and AB are 8 cm and 10 cm respectively.
= 4712.39 × m2
10 000 Find the volume of the prism.
= 0.471 239 m2
K J L
= 0.471 m2 (correct to 3 s.f.)
8
(b) (i) Volume of the drum I
= r × 302 × 25 10
= 70 685.8 D F H
E
15
= 70 700 cm3 (correct to 3 s.f.)
C 30
(ii) Volume of the drum
1
= 70 685.8 × m3 A B
1000 000 25
3
= 0.070 7 m (correct to 3 s.f.)
Solution
Area of the cross-section of the prism
16. The figure shows a loaf of bread with a uniform cross- 1
section. ABDE is a rectangle and BCD is a quarter of = 25 × 18 – × 10 × 8
2
a circle with centre B. It is given that AB = 18 cm, = 410 cm 2
AE = 13 cm and
CH = 40 cm. Find Volume of the prism
(a) the perimeter of the cross-section, = 410 × 30
(b) the area of the cross-section, = 12 300 cm3
(c) the total surface area of the bread,
(d) the volume of the bread.
Give your answers correct to 3 significant figures.
J I
H
F G
D 40
E
13
A C
18 B
Solution
(a) Perimeter of the cross-section
1
= 13 + 18 × 2 + 13 + × 2r × 13
4
= 82.42
= 82.4 cm (correct to 3 s.f.)
Solution
(a) (b) (a)
Solution (b)
(a) (b)
(a) (b)
4. Draw the views from the top and from the right-hand
side of each of the following shapes.
(a) (b)
2. Draw the views from the front and from the top of each
of the following shapes. Solution
(a) (b) (a)
Solution
(a) It is not the net of cube.
(b) It is a net of a cube.
167
6. Draw a sketch of the shape that can be formed by each (a) (b)
of the following nets.
(a)
Solution
(a) Yes, the figure is the top view of the solid.
(b) Yes, the figure is the right-hand side view of the
solid.
9. Sketch the views from the front and from the top of the
solid below.
Solution
(a)
Solution
(b)
Solution
Solution
(a) Yes, the figure can be the top view of a cube.
(a)
(b) Yes, the figure can be the side view of a cuboid.
1.5 1.5
(c) Yes, the figure can be the side view of a cylinder.
1 1
8. Determine whether the given figure can be a view of the 1 2 1
following solids.
1 1 1 1
2
1 1
1 1
1 1
1.5 1.5
1.5 1.5
1 1
2 r
5 5 5
Solution
2 r
Level 3
11. Sketch two different views of the given bottle.
Solution
xercise 15.2
E
Level 1
Give your answers correct to 3 significant figures where
applicable.
1. (a) Draw a net of a cube whose sides are 3 cm.
Top view Front view
(b) Find the volume and total surface area of a cube
whose sides are
(i) 3 cm,
12. The figures are two different views of a solid. Sketch (ii) 11 cm.
the shape of a possible solid.
Solution
(a) Diagram is not drawn to exact scale.
Solution
169
(b) (i) Volume of the cube = 33 Solution
= 27 cm3 (a)
Total surface area of the cube = 6 × 32
3.5
= 54 cm2
(ii) Volume of the cube = 113 5 3.5
= 1331 cm3
Total surface area of the cube = 6 × 112
= 726 cm2
5 4 4
2. A cuboid has dimensions 4 cm by 3 cm by 2 cm.
(a) Draw a net of the cuboid. 3.5
(b) Find the volume of the cuboid. 3 3.5
(c) Find the total surface area of the cuboid.
Solution
(a) 3.5 3.5
5 4
(c) H G
E F
6 C 10
D
4 5
A B
9
Chapter 15 Volume and Surface Area of Solids
170
Solution 7. Copy and complete the following table for cuboids.
(a) Volume of the prism = 3 × 2 × 5
= 30 cm3 Total surface
Length Breadth Height Volume
area
Total surface area of the prism
= 2 × (3 × 2 + 3 × 5 + 2 × 5) (a) 9 cm 6 cm 270 cm3
= 62 cm2 (b) 8 cm 5 cm 440 cm3
1 (c) 14 cm 6 cm 608 cm2
(b) Area of ABC = × 10 × 12
2
= 60 cm2 Solution
Volume of the prism = 60 × 25
= 1500 cm3 Total surface
Length Breadth Height Volume
area
Total surface area of the prism
= (13 + 13 + 10) × 25 + 2 × 60 (a) 9 cm 6 cm 5 cm 270 cm3 258 cm2
171
9. Find the volume and total surface area of each prism, (c) Let the drop in water level be h cm.
where the unit of length is centimetre. 50 × 30 × h = 6000
(a) (b) 6000
h =
1500
I H
L K
D 8 =4
C The drop in water level is 4 cm.
J 12 I
6
F
G 5
J
E H G
B 11. A rectangular tin plate measures 40 cm by 30 cm. A
E 5
15
C
25 small square of side 8 cm is cut out from each corner as
3
D 5 F shown in the figure. The plate is then folded along the
A 20 B 7
A dotted lines to form an open tray. Find
(a) the external surface area of the tray,
Solution
(b) the volume of the tray.
(a) Area of the cross-section ABCDEF
= 20 × 5 + 10 × 8 8
= 180 cm2 8
2
15. The figure shows a swimming pool with a rectangular
4 surface ABCD (not drawn to scale). CD = 20 m,
5 AE = 1.6 m, BF = 1 m and the area of ABCD is
240 m2. Find
(a) Find the volume of the space inside the room.
(a) the length of AD,
(b) Find the total surface area of the walls of the room.
(b) the volume of the pool.
(c) The owner is going to put wallpaper onto the walls.
Each roll of wallpaper is 10 m long and 0.53 m wide. D 20 C
How many rolls of wallpaper should be ordered?
Solution G
A
(a) Volume of space = 5 × 4 × 3 H
B
1
= 60 m3
1.6
F
(b) Surface area of walls E
= 2(5 × 3 + 4 × 3) – 1 × 2 – 2(1 × 1.5)
= 49 m2
Solution
(c) Area covered by each roll = 10 × 0.53 240
(a) Length of AD =
20
= 5.3 m2
49 = 12 m
Number of rolls = (b) Area of the cross-section ABFE
5.3
1
= 9.245 = × (1.6 + 1) × 20
2
10 rolls should be ordered. = 26 m2
Volume of the pool = 26 × 12
= 312 m3
Level 3
14. The figure shows a camping tent. The tent is 16. The figure shows a tray whose end faces are trapeziums.
closed at both ends and at the bottom. Given that If AB = 15 cm, BC = AD = 12.5 cm, CD = 22 cm,
AB = AC = 1.7 m, BC = 1.6 m, AN = 1.5 m and BF = 45 cm and AN = 12 cm, find
CF = 2.1 m, find (a) the area of ABCD,
(a) the space inside the tent, (b) the volume of the tray,
(b) the surface area of the tent, (c) the external surface area of the tray.
(c) the total cost of the material used in making the
H 22 G
tent if the material of the tent costs $30 per m2.
12.5
D E F
N C 45
D
12.5 12
A
A 15 B
E F
1.7 Solution
1.5 1.7
2.1 1
(a) Area of ABCD = × (15 + 22) × 12
N
2
B 1.6 C = 222 cm2
(b) Volume of the tray = 222 × 45
Solution = 9990 cm3
1
(a) Area of ABC = × 1.6 × 1.5 (c) External surface area of the tray
2
= 1.2 m2 = (12.5 + 15 + 12.5) × 45 + 2 × 222
= 2244 cm2
173
17. The diagram shows a piece of chocolate which is in 19. A manager wants to design a rectangular box of capacity
the form of a prism with a parallelogram base ABCD. 900 cm3. The box should have a square base of side a cm
AB = 4 cm, AD = 3 cm, DN = 2.5 cm and the volume and a height of h cm, where a and h are integers greater
of the piece is 15 cm3. Find than 1.
(a) the area of ABCD,
(b) the length of BF,
(c) the total surface area of the piece.
H G h
D
C
3 2.5
a
E F a
A B
N
4
(a) Find two possible sets of dimensions for the box.
Solution
(b) Suggest the dimensions for a design that uses the
(a) Area of ABCD = 4 × 2.5
least material to make the box.
= 10 cm2
(b) Volume = 15
10 × BF = 15 Solution
BF = 1.5 cm (a) a2h = 900
When a = 5, 52 × h = 900
(c) Total surface area = (3 + 4 + 3 + 4) × 1.5 + 2 × 10
h = 36
= 41 cm2
When a = 10, 102 × h = 900
h = 9
18. The diagram shows a marble chess box which is Two possible sets of dimensions are:
in the form of a hexagonal prism. The length of 5 cm × 5 cm × 36 cm, and
each side of the hexagon is x cm and the height of 10 cm × 10 cm × 9 cm.
the box is 2.5 cm. It is known that the base area of the
3
3 (b) Total surface area of the box = 2a2 + 4ah
box is x2 cm2 and the volume of the box is 375 3 cm3.
2 900
Find = 2a2 + 4a ×
a2
(a) the value of x, = 2a2 +
3600
(b) the total surface area of the box. a
Setting up a table or using a spreadsheet program,
we have
2.5
a Total surface area
5 770
6 672
x
7 612.3
Solution 8 578
(a) Base Area × Height = Volume
9 562
3
3 10 560
x2 × 2.5 = 375 3
2
2 11 569.3
x2 = 375 3 × 3 12 588
7.5
x2 = 100 13 614.9
x = 100
Hence the material used is minimum when
x = 10 a = 10. The optimal dimensions are
10 cm × 10 cm × 9 cm.
3
3
(b) Total surface area = 6x × 2.5 + 2 × x2
2
3
= 6 × 10 × 2.5 + 3 × 102
= 670 cm2 (correct to 3 s.f.)
1 Solution
Volume of the disc = r × 72 × 3
= 147r cm3
2. Find the volume and the total surface area of each cylinder
below, where the unit of measurement is in centimetre. Level 2
(a) (b) 6. Find the height of a cylinder if its
(a) volume = 63r cm3, base radius = 3 cm,
(b) volume = 100 cm3, base radius = 2 cm.
13 18
Solution
(a) Let the height of the cylinder be h cm.
6 30 r × 32 × h = 63r
h = 7
The height of the cylinder is 7 cm.
Solution
(b) Let H cm be the height of the cylinder.
(a) Volume of the cylinder = r × 62 × 13
r × 22 × H = 100
= 468r cm3
25
Total surface area of the cylinder H =
r
= 2r × 6 × 13 + 2 × r × 62
= 7.96 (correct to 3 s.f.)
= 228r cm2
The height of the cylinder is 7.96 cm.
(b) Volume of the cylinder
= r × 92 × 30
= 2430r cm3
Total surface area of the cylinder
= 2r × 9 × 30 + 2 × r × 92
= 702r cm2
175
7. Find the base radius of a cylinder if its Solution
(a) volume = 150r cm3, height = 6 cm, Let the depth of water in the jar be h cm.
(b) volume = 400 cm3, height = 8 cm. r × 52 × h = 15 × 10 × 4
24
h =
Solution r
(a) Let the base radius of the cylinder be r cm. = 7.64 (correct to 3 s.f.)
r × r2 × 6 = 150r The depth of water in the jar is 7.64 cm.
r 2 = 25
r = 5
11. The figure shows a solid half cylinder of base diameter
The base radius of the cylinder is 5 cm.
2 cm and height 2.5 cm.
(b) Let the base radius of the cylinder be R cm. (a) Find its volume. D
r × R2 × 8 = 400 (b) Draw a net of the solid. D
C
50 (c) Find its total surface area.
R = 2.5
r
= 3.99 (correct to 3 s.f.)
A 2 B
The base radius of the cylinder is 3.99 cm.
Solution
8. Find the circumference of a solid cylinder if its 1
(a) Volume of the solid = × r × 12 × 2.5
(a) curved surface area = 660 cm2, height = 10 cm, 2
(b) curved surface area = 1200 cm2, height = 15 cm. = 1.25r cm3
(b) The net of the solid is shown below.
Solution
(a) Circumference × height = curved surface area
660
Circumference of the cylinder =
10 D C D'
= 66 cm
1200
(b) Circumference of the cylinder = 2.5
15
= 80 cm
A 2 B A'
3.14
9. A metal cylinder of base radius 6 cm and height 5 cm
is melted and recast into a cylindrical metal bar of base
radius 2 cm. Taking r to be 3.124, find
(a) the length of the bar formed, (c) Total surface area of the solid
(b) the ratio of the total surface area of the cylinder to = (2 + r) × 2.5 + r × 12
that of the bar. = 16.0 cm2 (correct to 3 s.f.)
Solution
(a) Let y cm be the length of the bar formed. Level 3
r × 22 × y = r × 62 × 62 12. A cylindrical can of curry chicken 3.5
4y = 180 has a base radius of 3.5 cm and a
The length of the bar is 45 cm. height of 10 cm. Find
(b)
Total surface area of the original cylinder (a) the volume of the can,
(b) the area of the wrapper on the 10
= 2r × 6 × 5 + 2 × r × 62
= 132r cm2 curved surface of the can.
Total surface area of the bar
= 2r × 2 × 45 + 2 × r × 22
= 188r cm2
The required ratio = 132r : 188r Solution
= 33 : 47 (a) Volume of can = r × (3.5)2 × 10
= 385 cm3 (correct to 3 s.f.)
10. A rectangular tray of dimensions 15 cm by 10 cm by (b) Area pressed by roller after 3 revolutions
4 cm is full of water. The water is poured into an empty = 2 × r × (3.5) × 10
cylindrical jar of internal radius 5 cm. Taking r to be = 220 cm2 (correct to 3 s.f.)
3.124, find the depth of water in the jar.
Solution
(a) Volume of the bucket = r × 102 × 18
3 = 1800r cm3
(b) Volume of the tank = 60 × 45 × 36
= 97 200 cm3
5 5 (c) Let the rise in the water level be h cm.
Find 60 × 45 × h = 1800r × 7
(a) the volume of the stone, 14r
h =
(b) the increase in the contact area between the water 3
and the measuring cylinder. = 14.7 (correct to 3 s.f.)
The rise in the water level is 14.7 cm.
Solution
(d) Let n be the number of full buckets required.
(a) Volume of the stone
1800r × n 97 200
= r × 2.52 × 3
97 200
= 18.75r cm3 n
1800r
(b) Increase in water contact area n 17.19
= 2r × 2.5 × 3 The required number of full buckets is 18.
= 15r cm2
17. A piece of cylindrical pipe is 8 cm long. Its internal base
15. A stack of 10 $1 coins forms a cylinder of base diameter diameter is 4 cm and its wall is 0.5 cm thick.
2.24 cm and height 2.4 cm. (a) Find the external base radius of the pipe.
(a) Find the volume of the stack. (b) Find the volume of material used in making the
(b) Find the total surface area of the stack. pipe.
(c) If six more $1 coins are added to the stack, find (c) Find the total surface area of the pipe.
the increase in the total surface area of the stack. (d) Water flows through the pipe at a rate of 15 cm/s.
(Take r to be 3.142) Find the volume of water delivered in 20 seconds.
Solution
(a) External base radius of the pipe
2.4
= 4 ÷ 2 + 0.5
= 2.5 cm
(b) Cross-sectional area of the pipe
2.24
= r × 2.52 – r × 22
Solution
= 2.25r cm2
(a) Volume of stack = r × (1.12)2 × 2.4
Volume of material used in making the pipe
= 9.46 cm3 (correct to 3 s.f.)
= 2.25r × 8
(b) Total surface area = 18r cm3
= 2 × r × 1.12 × 2.4 + 2 × r × (1.12)2
= 24.8 cm2 (correct to 3 s.f.)
177
(c) Internal lateral surface area of the pipe A sheet of rectangular paper measures 20 cm by 15 cm. It
= 2r × 2 × 8 can be rolled up along either side to form two cylinders,
= 32r cm2 A and B as shown.
External lateral surface area of the pipe (a) Which cylinder has a greater volume?
= 2r × 2.5 × 8 cm2 (b) Suppose the sheet is divided into two equal parts,
= 40r cm2 X and Y, as shown and then rolled up to form two
Total surface area of the pipe cylinders of height 15 cm each. Would the sum of
= 32r + 40r + 2 × 2.25r the volumes of these two small cylinders be greater
= 76.5r cm2 than the volume of cylinder B? Explain how you
arrived at your conclusion.
(d) Volume of water delivered in 1 second
= r × 22 × 15 Solution
= 60r cm3 (a) Let a cm and b cm be the base radii of cylinder A
Volume of water delivered in 20 seconds and cylinder B respectively.
= 60r × 20 2ra = 15
= 1200r cm3 15
a =
2r 2
15
18. A cylindrical cake has a base radius of 12 cm and height Volume of cylinder A = r × × 20
2r
8 cm. It is cut into two equal halves as shown. Taking r 1125
= cm3
to be 3.142, find r
(a) the volume of each half, 2rb = 20
10
(b) the total surface area of each half, b =
r
(c) the percentage increase in the total surface area of 2
the cake due to the cutting. Volume of cylinder B = r ×
10
× 15
r
1500
= cm3
12
r
Cylinder B has a greater volume.
Solution
15 (a) 3 m2 = 3 × 10 000 cm2
= 30 000 cm2
(b) 13.6 m2 = 13.6 × 10 000 cm2
= 136 000 cm2
179
Level 2 Solution
7. A playground is built on a rectangular plot of land 35 m (b) Area of the cross-section ABCDEFGH
long and 24 m wide. Find the area of the land 1
= × (4 + 16) × 8 – 4 × 4
2
(a) in m2, (b) in cm2.
= 64 cm2
Solution Volume of the prism = 64 × 20
(a) Area of the playground = 35 × 24 = 1280 cm3
= 840 m2 Total surface area of the prism
(b) Area of the playground = 840 × 10 000 cm2 = (10 + 4 + 10 + 6 + 4 + 4 + 4 + 6) × 20
= 8 400 000 cm2 + 2 × 64
= 1088 cm2
9. Find the total surface area and volume of the each prism, 7
H
Solution
D (a) Volume of the solid
17 17
8 = 10 × 7 × 12 – r × 22 × 12
E C G = 840 – 48r
12 12 = 689 cm3 (correct to 3 s.f.)
27
A 30 B (b) Total surface area of the solid
Solution = 2 × (10 × 7 + 10 × 12 + 12 × 7) – 2 × r × 22
1
Volume = × 30 × 8 × 27 + 12 × 30 × 27 + 2r × 2 × 12
2 = 548 + 40r
= 12 960 cm3 = 674 cm2 (correct to 3 s.f.)
Total surface area
1 12. The figure shows a solid formed by stacking two cubes
= × 30 × 8 × 2 + 12 × 30 × 2 + 12 × 27 × 2 + 17 ×
2 of sides 3 cm and 5 cm respectively. Find
27 × 2 + 30 × 27 (a) the volume of the solid,
= 3336 cm2 (b) the total surface area of the solid.
10. Find the total surface area and volume of the prism,
where the unit of length is centimetre. 3
I P M L
A 6 6
H E D 5
4 4
K
10 G 4 F 10
4 20
B C Solution
(a) Volume = 33 + 53
= 152 cm3
(b) Total surface area = 5 × 32 + 5 × 52 + 52 – 32
= 186 cm2
Solution
9 (a) (i) Volume of the tank
Solution = r × 0.82 × 2
(a) Volume of the solid = 1.28r
= r × 32 × 9 + r × 92 × 6 = 4.02 m3 (correct to 3 s.f.)
= 567r (ii) Volume of the tank
= 1780 cm3 (correct to 3 s.f.) = 4.02 × 1 000 000 cm3
(b) Total surface area of the solid = 4 020 000 cm3 (correct to 3 s.f.)
= 2r × 3 × 9 + 2r × 9 × 6 + 2 × r × 92 (b) (i) Volume of a can
= 324r = r × 102 × 25
= 1020 cm2 (correct to 3 s.f.) = 2500r
= 7850 cm3 (correct to 3 s.f.)
14. F 40 E (ii) Volume of a can
G 7850
13
= m3
12 12 13 1 000 000
= 0.007 850 m3
A D
M B C N
20 (c) Let n be the number of cans that can be filled with
5 5
petrol.
The figure shows the uniform cross-section of a 2500r × n 1.28r × 1 000 000
solid. ADEF is a trapezium and BGC is a semicircle. n 512
AM = DN = 5 cm, AF = DE = 13 cm, EN = FM = 12 cm, It can fill 512 cans with petrol.
BC = 20 cm, EF = 40 cm and the thickness of the solid
is 8 cm. Taking r to be 3.142, find 16. The figure shows a bolt which has a hexagonal head
(a) the volume of the solid, and a cylindrical body. The hexagonal head is a prism
(b) the total surface area of the solid. 3
3
of base area cm2 and height 0.5 cm.
2
Solution The length of each side of the hexagon is 1 cm. The
(a) AD = 5 + 40 + 5 cylindrical body has a diameter of 1 cm and a length of
= 50 cm 4 cm. Taking r to be 3.142, find
Area of the cross-section (a) the volume of the bolt,
1 1 (b) the total surface area of the bolt.
= × (40 + 50) × 12 – × r × 102
2 2
= (540 – 50r) cm2 0.5
Volume of the solid
= (540 – 50r) × 8
= 3060 cm3 (correct to 3 s.f.) 1
(b) Perimeter of the cross-section 1
= 40 + 13 + 30 + 13 + r × 10
= 96 + 10 r 4
Total surface area of the solid
= (96 + 10r) × 8 + 2 × (540 – 50r)
= 1848 – 20r
= 1790 cm2 (correct to 3 s.f.)
181
Solution (d) Volume of the sofa
(a) Volume of the bolt = 3218.12 × 180 cm3
3
3 1 2 = 579 261.6 cm3
= × 0.5 + ×r×4 579 261.6
2 2 = m3
3 1 000 000
= 4.44 cm (correct to 3 s.f.)
= 0.579 m3 (correct to 3 s.f.)
(b) Total surface are of the bolt
3
1 2 3 1 2
= 2 × r × 0.5 × 4 + r × + ×2–r× 18. In the figure, 6 cylindrical cans of soft drink are packed
2 2 2
+ 6 × 0.5 × 1 tightly in a rectangular carton which is open at the top.
= 20.8 cm2 (correct to 3 s.f.) Each can is of diameter 6.4 cm and height 12 cm. Taking
r to be 3.142, find
(a) the external surface area of the carton,
17. The figure shows the uniform cross-section of a (b) the total volume of the cans,
sofa. EFG is a semicircle, AB = 72 cm, BC = 20 cm, (c) the percentage of the volume of the carton which
CD = 40 cm, AG = 63 cm and the length of the sofa is is empty space.
1.8 m. Taking r to be 3.142, find
(a) the perimeter of the cross-section in centimetre,
(b) the area of the cross-section in cm2,
(c) the total surface area of the sofa in m2,
(d) the volume of the sofa in m3.
F
G E Solution
(a) External surface area of carton
= 6.4 × 2 × 12 × 2 + 6.4 × 3 × 12 × 2 + 6.4 × 2 ×
63 6.4 × 3
40 C = 12.8 × 12 × 2 + 19.2 × 12 × 2 + 12.8 × 19.2
D
20 = 1010 cm2 (correct to 3 s.f.)
A 72 B (b) Total volume of the cans
6.4 2
=r × 12 × 6
Solution 2
(a) GE = 72 – 40 = 2316.53376
= 32 cm = 2320 (correct to 3 s.f.)
Perimeter of the cross-section (c) Percentage of empty space
1
= 72 + 20 + 40 + 43 + 63 + × 2r × 16 12.8 × 19.2 × 12 – 2316.53376
2 = × 100%
12.8 × 19.2 × 12
= 238 + 16r
= 288.27 = 21.5% (correct to 3 s.f.)
= 288 cm (correct to 3 s.f.)
19. (a) State and draw two items in your home that are
(b) Area of the cross-section
1 made up of basic solids.
= 20 × 40 + 63 × 32 + × r × 162 (b) Measure their dimensions and calculate their
2
= 2816 + 128r volumes and total surface areas.
= 3218.12
= 3220 cm2 (correct to 3 s.f.) Solution
For (a) and (b), the answers vary.
(c) Total surface area of the sofa Examples of items that are made up of basic solids are
= (20 + 40 + 43 + 63 + 72 + 16r) × 180 as follows:
+ 2 × 3218.12 A table may be in the form of a cuboid with 4 cylindrical
= 288.27 × 180 + 2 × 3218.12 legs.
= 58 324.84 cm2 A candleholder may be a cube with a cylindrical hole.
58 324.84
= m2
10 000
= 5.832 484 m2
= 5.83 m2 (correct to 3 s.f.)
A B N C D
(b) Total surface area of the cube = 6 × 92 2
= 486 cm2
2.5
183
(a) Construct the quadrilateral ABCD using a protractor, (f) Volume of the prism = 4.56 × 2.5 = 11.4 cm3
a pair of compasses and a ruler. (g) Total surface area of the prism
(b) Measure the lengths of AD and CD. = 9.2 × 2.5 + 2 × 4.56
(c) Find the perimeter of ABCD. = 32.12 cm2
(d) Find the area of ABCD.
(e) Draw a net of the prism.
(f ) Find the volume of the prism. 7. The base radius and height of a cylinder are 6 cm and
(g) Find its total surface area. 10 cm respectively.
(a) Find, in terms of r, the volume of the cylinder.
Solution (b) If the base radius increases by 20% and the height
(a) D decreases by 10%, taking r to be 3.142, find
(i) the new base radius,
(ii) the new height,
(iii) the percentage change in the volume of the
3.0 C cylinder.
1.6
Solution
A 1.8 B
(a) Volume of cylinder
Construction Steps: = r × 62 × 10
1. Draw a line segment AB 1.8 cm long. = 360 cm3
2. Draw a ray from A and perpendicular to AB. (b) (i) New base radius = 120% × 6
3. With B as centre and radius 3.0 cm, draw an = 7.2 cm
arc to cut the ray at D. (ii) New height = 90% × 10
4. Draw the line segment BD. = 9 cm
5. Draw BC 1.6 cm long and perpendicular to BD.
(iii) New volume of cylinder
6. Draw the line segment CD.
= r × (7.2)2 × 9
7. Then ABCD is the required quadrilateral.
= 466.56r cm3
(b) Length of AD = 2.4 cm
Length of CD = 3.4 cm Percentage change in volume
(c) Perimeter of ABCD = 1.8 + 1.6 + 3.4 + 2.4 466.56r – 360r
= × 100%
= 9.2 cm 360r
(d) Area of ABCD = 29.6%
1 1 There is a 29.6% increase in volume.
= × 1.8 × 2.4 + × 3.0 × 1.6
2 2
2
= 4.56 cm
8. The figure shows an arrow sign which is in the form of a
(e) 2.4 prism. It is given that DK = 2.5 cm, AB = AG = 10 cm,
AN = 6 cm, BG = 16 cm, CD = 15 cm, DE = 8
cm and BC = GF.
1.8 (a) Sketch the view of the sign when viewing from the
3.4 top.
(b) Find the total surface area of the sign.
1.6 (c) Find the volume of the sign.
1.6
N
G
M
L
10 E
F
3.4 N
16 A 6 8
K
10 C 2.5
15 D
I
3.4
B
2.4
1.6
1.8 2.5
G H
90
30
G 20 F 30 E 20
A D
40
A 60 B
Find B C
(a) the perimeter of the cross-section,
(b) the area of the cross-section, Solution
(c) the surface area of the cabinet (a) Area of the cross-section
(i) in cm2, 1
= 30 × 20 + × r × 202
(ii) in m2. 2
(d) the volume of the cabinet = 600 + 200r
(i) in cm3, = 1228 cm2
(ii) in m3. Length of the curve ABCD
Give your answers correct to 3 significant figures.
1
= 30 + × 2r × 20
2
Solution = 30 + 20r
(a) Perimeter of the cross-section = 92.83 cm
1
= 60 + 90 + 90 + × 2r × 30 External surface area of the tray
2
= 240 + 30r = 92.83 × 30 + 2 × 1228
= 334.25 = 5240.9
= 334 cm (correct to 3 s.f.) = 5240 cm2 (correct to 3 s.f.)
185
(b) Volume of the tray 12. A circular pipe is 30 cm long. Its internal and external
= 1228 × 30 diameters are 3 cm and 4 cm respectively.
= 36 840 (a) Find the volume of the material used in making
= 36 800 cm3 (correct to 3 s.f.) the pipe.
(b) Find the total surface area of the pipe.
(c) Let the base radius of the bin be r cm.
(c) If water flows through the pipe at a rate of 5 cm/s
rr 2 × 40 = 36 840
into an empty rectangular tank, find the volume of
r = 921 water delivered in 20 seconds.
r (d) If the rectangular tank is 28 cm long and 20 cm
= 17.1 (correct to 3 s.f.) wide, find the depth of water in the tank after 20
The required base radius is 17.1 cm. seconds.
(Take r to be 3.142)
11. The figure shows an open wooden box. Its external Solution
length, breadth and height are 25 cm, 22 cm and 20 cm (a) Area of the cross-section of the pipe
respectively. The thickness of the wood is 2 cm. Find = r × 22 – r × 1.52
(a) the external volume of the box, = 1.75r cm2
(b) the internal volume of the box, Volume of material used
(c) the volume of wood used in making the box, = 1.75r × 30
(d) the total surface area of the box. = 52.5r
= 165 cm3 (correct to 3 s.f.)
(b) Total surface area of the pipe
2 = 2r × 2 × 30 + 2r × 1.5 × 30 + 2 × 1.75r
20 = 213.5r
= 671 cm2 (correct to 3 s.f.)
(c) Volume of water delivered in 20 s
22 = r × 1.52 × 5 × 20
= 225r
25 = 707 cm3 (correct to 3 s.f.)
Class Activity 1
Objective: To select an appropriate data collection method.
Task
Discuss and suggest an appropriate way (taking measurements, observing outcomes, conducting surveys, reading publications)
to collect data to inform the following statements.
conducting a survey
taking measurements
conducting surveys
conducting surveys
taking measurements
reading publications
187
Class Activity 2
Objective: To discuss the advantages and disadvantages of using pictogram and bar graph.
1. The following table shows the distribution of different types of trees in a farm.
You are the owner of the farm and would like to present the data by a graph to the visitors.
If the data is to be represented by a pictogram or bar graph, which graph
(a) is easier to draw?
It is easier to read the number of trees from the bar graph.
Age (years) 12 13 14 15 16
Number of male students 16 23 16 19 8
Number of female students 12 17 20 15 13
Between pictogram and bar graph, which graph would you think is more appropriate to represent the data? Explain your
answer.
The data cannot be represented by a pictogram. An icon cannot represent both male and female students.
Questions
1. The table shows the revenue of a new company in its first 4 quarters.
(b) Which diagram would meet each of the following objectives of the chairman’s presentation?
The pie chart is more appealing to the public. This is because each sector in a pie chart can be represented by different colours.
(ii) easier for the audience to read the difference between the revenues in the third quarter and the
second quarter
The line graph would be preferred. This is because the reading of the revenue in each quarter can be read from the scales on the vertical
axis.
(iii) can show the proportion of the first quarter revenue in the whole year revenue
The pie chart can show the proportion of the first quarter revenue in the whole year revenue. This proportion is the proportion of the area of
the sector representing the first quarter revenue in the whole circle.
The line graph can show the trend of the revenue in these 4 quarters. The trend can be observed as the general movement of the line graph.
2. The table shows the time that Eric spent on four home assignments in one day.
Assignment 1 2 3 4
Time (minutes) 45 30 60 45
Between line graph and pie chart, which graph is more appropriate to represent the data? Explain your answer.
A pie chart is more appropriate as the data does not involve a time series.
189
3. The table shows the price index of the private residential property in 6 consecutive quarters in Singapore.
Suppose the aim of drawing a graph is to predict the index in the third quarter of 2011. Given a choice between the line
graph and pie chart, which graph is more appropriate? Explain your answer.
It is meaningless to draw a pie chart to represent the data. There is no meaning for both the sum of the given figures and each figure as a portion of
the sum. To predict the index in the third quarter of 2011, extend the line graph to 2011 Q3 to get an expected index of 206.
Class Activity 4
Objective: To work collaboratively on a survey to collect and classify data, present data using appropriate graphs, and analyse
data. Use the data to make some decisions.
Notes to teacher
This activity wants students to apply the statistical techniques they have learnt to conduct a survey. They should be guided at
various stages. Teachers may check and discuss their questionnaire before their data collection. Students should be reminded
to use appropriate graphs to represent their data. When analysing data, students should first set the facts they want to reflect
in the survey.
Tasks
1. Before collecting data, students must design a questionnaire. They should be aware the following points:
• The questionnaire should have a brief description of the objective of the survey.
• The questionnaire should not be too long.
• Questions should be related to the objective.
• Questions should be in simple and clear language, and appropriate format.
• They should first ask a few classmates to attempt the draft questionnaire to examine whether there are any problems
in the questionnaire design.
Students have to decide the target interviewees, the sample size, the way, the place and the time to collect data. They
should make sure that they are in a safe environment when collecting data.
2. The data collected in each question can be organised into tables. Each set of data may have different way of presenting
them.
3. Students may enter their data into a spreadsheet program and generate graphs using the Graph Wizard of the program.
4. In analyzing data, students may use percentage and ratio. For instance, calculate the percentage of students who are satisfied
with the cleanliness of the canteen or the ratio of rubbish that is recyclable. Teachers may need to set some guidelines for
the presentation of the report to students.
Questions
1. The table shows the number of residential broadband subscriptions in Singapore in recent years.
If you are going to present this set of data and show the trend of the data to your classmates, which type of statistical
graph is most appropriate? Give your reasons.
The line graph is the most appropriate to represent the data. The data forms a time series and we can visualize the trend of movement of the data from the
line graph.
2. The table shows the average monthly incomes of employees from four different industries in Singapore in the year 2010.
If you are going to present this set of data to some school leavers, which type of graph is most appropriate? Give your
reasons.
The bar graph is most appropriate to represent the data. It is easy to read the difference between the average monthly incomes of two industries
3. The table shows the percentage of resident non-students in Singapore aged 25 – 39 years having various education
attainments in the year 2010.
The pie chart is most appropriate to represent the data as it is visually appealing to the public. The public can easily observe the proportion of residents in
191
4. The table shows the number of rainy days in each of the first 6 months in the year 2011 in Singapore.
If you are going to present this set of data and show the trend of the data to your classmates, which type of statistical
graph is most appropriate? Give your reasons.
The pictogram is most appropriate to represent the data. This is because a pictogram is visually interesting and appealing to the public. At a glance, the
5. Work in groups with your classmates and write down the advantages and disadvantages of the following types of statistical
graphs.
(a) Pictogram (b) Bar graph
(c) Line graph (d) Pie chart
Questions
Two branch managers present the following line graphs of annual sales of their stores to the board of directors.
54 100
Sales ($ million)
Sales ($ million)
52 80
50 60
48 40
2010 2011 2012 2010 2011 2012
Year Year
he manager of branch A claimed that his branch had a higher percentage increase if compared with branch B from 2010 to
T
2012. Do you think his claim is correct? Explain your answer.
For branch A,
sales in 2010 = $50 million
sales in 2012 = $54 million
54 – 50
\ percentage increase in sales from 2010 to 2012 = 50 × 100%
= 8%
For branch B,
sales in 2010 = $60 million
sales in 2012 = $80 million
80 – 60
\ percentage increase in sales from 2010 to 2012 = 60 × 100%
1
= 33 3 %
Manager A’s claim is incorrect. Both graphs use different scales to represent their sales. The scale used for branch A misleads readers into thinking branch A
193
Try It! Section 16.3
Section 16.2 3. The pictogram below shows the number of plants in a
1. The following list shows the fitness grades ranging from flowerbed.
Number of plants in a flowerbed
A to E of 36 students.
Daffodil
B B A C D C A B E
C A D B C E D C B Marigold
D E E A D B C B A
E C B C E C D C D Orchid
Represent the data using a frequency table. Tulip
Solution
represents 5 plants
Fitness Grade Tally Frequency
A //// 5 (a) Which type of plant is the most available in the
flowerbed?
B //// /// 8
(b) Find the difference in the numbers of marigold and
C //// //// 10 tulip plants in the flowerbed.
D //// // 7
E //// / 6 Solution
(a) The row of orchid has the most number of symbols.
Total 36
Therefore, orchid is most available in the flowerbed.
(b) Difference = 32 – 18
2. The following list shows the prices (in dollars) of = 14
24 books. number of tulips – number of marigold
(b) Percentage of books with prices higher than $30 10-16 July
3+8
= × 100%
24 17-23 July
11
= × 100%
24
24-30 July
= 45.8% (correct to 3 s.f.)
represents 5 plants
10 rate
6. The bar graph shows the sales of coffee and tea at a stall
in 4 hours on a certain morning. 20
coffee
tea
40
Crude birth rate (thousands)
Number of cups
30
20
10 10
O
08 00 – 08 59 09 00 – 09 59 10 00 –10 59 11 00 –11 59
Time
195
8. The line graph below shows the Consumer Price Index Solution
of Country X from 2004 to 2010, where the index in Expenses
2009 is set to 100. Category Angle at Centre
($ million)
420
Consumer Price Index of Country X Transport 420 × 360° = 79.0°
1914
673
110 Trade & Industry 673 × 360° = 126.6°
1914
746
Consumer price index
100 Info-communications 75
technology and 75 × 360° = 14.1°
1914
media development
Total 1914 360.0°
90
Trade &
O Manpower Industry
2004 2005 2006 2007 2008 2009 2010
Year 140.3° 126.6°
(a) Present the data by a pictogram and a line graph 12. The bar graph shows the times taken by 3 students to
respectively. complete a mathematics assignment. By observing the
(b) Which of the diagrams drawn in (a) is more bar graph, can we claim that Ling took three times as
appealing to the public? long as Majid to complete the mathematics assignment?
(c) Which of the diagrams drawn in (a) can reveal the Explain your answer.
trend of the sales?
(d) Which of the diagrams drawn in (a) can indicate Time taken for a mathematics assignment
the month with the lowest sales?
Solution 45
Time taken (min)
(a)
Sales of cars of a company in four months
40
January
35
February
30
Ali Ling Majid
March
Student
April
Solution
represents 5 cases Ratio of time taken by Majid to Ling
 = 35 : 45
= 7 : 9
Therefore, we cannot claim that Ling took three times
as long as Majid.
197
13. The following diagram is released by Singapore Police 14. The line graph shows the energy level of a person after
Force. It shows the fatal and injury rate of traffic accidents drinking Bull tea.
in Singapore from 2007 to 2011. (a) What is the message that the graph wants to convey
to readers?
Fatal and Injury Accident Rate (2007 – 2011) (b) The advertisement claims that ‘You will be full of
1,000,000 120.00 energy 30 minutes after drinking Bull tea.’ Would
950,000
you believe it? Explain your answer.
110.00
900,000 100.98
98.17
Energy level of a person after drinking Bull Tea
100.00
850,000 93.41
91.96 90.00
800,000
83.31
80.00
Energy level
750,000
700,000 70.00
2007 2008 2009 2010 2011
Vehicle Population Fatal and Injury Accident Rate per 10,000 Vehicle Population
199
(c) Describe some of its good features. Solution
(d) Suggest some ways to improve it.
Number of pets Tally Frequency
Solution 0 //// // 7
(a) Advise students that each survey must have its 1 //// //// 10
objective or goal. 2 //// // 7
(b) Questions may be 3 //// 4
(i) two-state, 4 // 2
(ii) multiple choice, and Total 30
Solution
(b) Percentage of passengers who covered more than
7+5 (a) Number of
6 km = × 100% Tally Frequency
30 defective vases
= 40% 0 //// /// 8
1 //// //// 10
Level 3 2 //// 5
6. The following are the scores of 32 students in a science 3 //// 4
test.
4 /// 3
7.2 2.8 4.2 6.0 6.5 3.3 5.3 9.0 7.1 0.8 Total 30
4.1 3.7 8.1 3.6 4.7 6.4 8.4 1.5 5.5 7.9
9.3 5.8 6.9 5.7 1.6 2.3 4.8 4.0 9.2 6.5 (b) Percentage of cartons that contain at most one
8 + 10
defective vase = × 100%
30
= 60%
201
8. The scores in Round 1 of the first 30 golfers in the 2011 Solution
US Masters Tournament were as follows:
(a) Time (x h) Tally Frequency
0 x 3 //// //// /// 13
69 72 72 71 69 72 71 73 67 71
73 72 72 74 69 69 71 69 65 71 3 x 6 //// // 7
68 67 74 69 72 72 72 74 74 71 //// 4
6 x 9
9 x 12 /// 3
(a) Construct a frequency table for the data.
Total 27
(b) What is the winning score in Round 1 if the game
was won by the one with lowest score?
(b) Percentage of people who surfed the Internet
(c) If the even-par score is 72, find the percentage of
in hours in the range 3 x 9
golfers who completed the round with an even-par
7+4
score. = × 100%
27
Solution
= 40.7% (correct to 3 s.f.)
(a) scores Tally Frequency
10. The following data are the monthly utilities bills (in $)
65 / 1 of 35 families in August 2011.
67 // 2
68 / 1 236.49 315.37 256.88 104.56 418.30 176.40 198.95
69 //// / 6 367.90 187.80 54.73 467.32 238.89 356.60 245.71
287.59 314.90 267.80 378.24 480.43 69.78 135.60
71 //// / 6
78.40 178.10 245.67 314.90 83.20 472.10 174.30
72 //// /// 8 267.80 69.30 134.70 314.60 180.90 245.60 430.15
73 // 2
74 //// 4 (a) Construct a frequency table of uniform intervals
Total 32 for the data, using the intervals 0 , x 100,
100 , x 200 and so on.
(b) The winning score is 65. (b) Find the percentage of the number of families whose
(c) Percentage of golfers who completed the round monthly utilities bills are more than $200 but less
with an even-par score than or equal to $400.
8
= × 100% Solution
30
= 26.7% (correct to 3 s.f.) (a) Test scores (x) Tally Frequency
0 x 100 //// 5
9. A survey investigates the number of hours that a person //// //// 9
100 x 200
surfs the Internet per day. The following list shows the
survey results of 27 people who surfed the Internet. 200 x 300 //// //// 9
300 x 400 //// // 7
1.5 3.2 2.1 3.0 0.8 4.2 11.6 2.7 6.6 400 x 500 //// 5
4.1 1.6 6.3 2.5 2.0 5.8 2.4 8.8 5.4 Total 35
0.4 9.2 2.9 5.7 7.8 1.0 3.9 10.3 2.9
(b) Number of families whose monthly electricity bills
(a) Construct a frequency table of uniform intervals, are more than $200 but less than or equal to $400
using the intervals 0 x 3, 3 x 6 and so = 9 + 7
on. = 16
(b) Find the percentage of people in the survey who Required percentage
spend more than 3 hours but fewer than or equal 16
to 9 hours per day surfing the Internet. = × 100%
35
= 45.7% (correct to 3 s.f.)
203
3. 5. The bar graph below shows the number of in-coming
Evergreen telephone calls received by an office in a working week.
5
30
= 19
The average number of calls received in a week
20
is 19.
10
6. The bar graph shows the numbers of rainy days in city
XYZ over a period of four months.
0
Badminton Cooking Dancing Sewing (a) Which month had the least number of rainy days?
Club (b) How many more rainy days were there in July than
in June?
(c) Find the total number of rainy days in these four
Solution months.
(a) The dancing club has the greatest number of
members.
(b) Total number of members = 29 + 19 + 35 + 24
= 107
0 Solution
May Jun Jul Aug
Month (a) Ahmad’s monthly income increased steadily from
January to March.
Solution (b) In January, Ahmad’s income was less than his
(a) May had the least number of rainy days. expenditure.
The difference = $2800 – $2300
(b) Number of rainy days in July = 8
= $500
Number of rainy days in June = 5
Difference = 8 – 5 (c) Total income in 3 months
=3 = $(2300 + 2500 + 3000)
There are 3 more rainy days in July than in June. = $7800
Total expenditure in 3 months
= $(2800 + 2500 + 2000)
Level 2
= $7300
7. The table shows the number of different types Hence his total income was greater than his total
of shops in a shopping mall. expenditure.
Rating Frequency
Appliances 8 Level 3
Clothing 19 9. The following table shows the number of medals was
Food 20 won by the top 3 countries in the 2008 Beijing Summer
Shoes 10 Olympics.
Others 15 Country Gold Silver Bronze Total
A pictogram is used to represent the data. It is given that China 51 21 28 100
the number of symbols in each row should not exceed 10.
What is the least number of shops that can be represented USA 36 38 36 110
by each symbol?
Russia 23 21 28 72
Solution
(a) Represent the data using a bar graph.
As the highest frequency is 20, the least number of shops
20
(b) Express the total number of medals won by USA
is = 2. as a percentage of the total number of medals won
10
by China.
8. The bar graph shows Ahmad’s income and expenditure (c) Express the number of gold medals won by USA
for each month from January to March. as a percentage of the number of gold medals won
by China, correct to 3 significant figures.
Income and expenditure of Ahmad
Income
Expenditure
3000
Amount ($)
2000
1000
0
Jan Feb Mar
Month
205
Solution Solution
(a) Represent the data using a bar graph. It is not a fair comparison chart.
1. The scale on the vertical axis does not start from
Gold zero.
2. The DD Brand bar is highlighted (it is different in
Silver
55 colour).
Bronze 3. The width of the DD Brand bar is double the widths
50 of the other brands.
All of the above tricks give an illusion that the annual
45 sales of DD Brand cars is a lot more than the other brands.
40
11. Visit a shopping mall near your home. Count the number
Number of medals
20
Exercise 16.4
15 Level 1
1. The line graph shows the changes in the body temperature
10 (°C) of a patient from 6 a.m. to 12 a.m. on a certain
day.
5 Body temperature of a patient
0 China USA Russia
Countries 41
Temperature (°C)
39
110
(b) Required percentage = × 100%
100
37
= 110%
36 35
(c) Required percentage = × 100%
51
= 70.6% (correct to 3 s.f.) 0
6 a.m. 9 a.m. 12 p.m. 3 p.m. 6 p.m. 9 p.m. 12 a.m.
Time
10. The bar graph shows the annual sales of four models
of cars. It was used in an advertisement by DD Brand (a) What was the highest body temperature of the
Company. Is it a fair comparison chart? Explain your patient? When did the patient have it?
answer. (b) Find the number of readings for temperatures above
37 °C.
24
The patient had it at 9 p.m.
23 (b) The number of readings for temperatures above
37 °C is 4.
22
21
20
AA BB CC DD
Brand
35% 10%
Geography
20
Favourite subjects of 180 students
19
(a) Which subject do most students like?
O (b) Find the angle at the centre of the sector representing
Mon Tues Wed Thurs Fri Science.
Day (c) Find the number of students whose favourite subject
is English.
(a) On which day is the closing price the lowest?
(b) Describe the trend of the price of the stock. Solution
(a) Most students like Science.
Solution (b) Angle representing Science
(a) The closing price is the lowest on Friday. = 35° × 360°
(b) The price of the stock rose slightly on Tuesday and = 126°
then it dropped on the next three days. (c) Number of students whose favourite subject is
English
3. The pie chart shows how some = 25° × 180
students travel from home to school. = 45
On foot
(a) By which mode do most of Bus
the students travel from home 105°
135°
to school? Others Level 2
(b) Find the percentage of students MRT 5. The line graph below shows the mass (kg) of a baby in
who go to school on foot. the first 12 months.
(c) Find the angle of the sector Modes of travel Mass of a baby
representing ‘Others’. of students from
home to school
Solution 10
(a) Most of the students travel by bus.
8
(b) Percentage of students who go to school on foot
105
Mass (kg)
= × 100% 6
360
= 29.2% (correct to 3 s.f.)
4
(c) Angle representing 'others'
= 360° – 135° – 105° – 90° 2
= 30°
O 2 4 6 8 10 12
Month
(a) What was the mass of the baby at birth?
(b) How many months after birth was the mass of the
baby exceeding 8 kg?
207
(c) Find the average rate of increase in the mass of the 7. A shop sells three grades of coffee
baby, in kg/month, which are Platinum, Gold and Platinum Silver
(i) during the first two months, Silver. The pie chart represents
5x°
(ii) between the 10th and 12th months, the mass (in kg) of each grade
4x°
(iii) in the first year. of coffee sold on a particular day.
Gold
The total mass of coffee sold was
Solution 14.4 kg.
(a) The mass of the baby at birth is 3.6 kg. Sales of coffee
(b) The mass of the baby was exceeding 8 kg after Find
6.6 months. (a) the value of x,
(b) the percentage of Platinum coffee in the total mass,
(c) (i) The required rate
4.8 – 3.6 (c) the mass of Gold coffee sold.
= 2
= 0.6 kg/month Solution
(a) 4x° + 5x° + 90° = 360° (∠s at a point)
(ii) The required rate 9x = 270
10 – 9.6 x = 30
= 2
= 0.2 kg/month 90
(b) Percentage of platinum coffee = × 100%
360
(iii) The required rate = 25%
10 – 3.6
= 12 4 × 30
(c) Mass of Gold coffee sold = 14.4 × kg
= 0.533 kg/month (correct to 3 s.f.) 360
= 4.8 kg
13 years
400
Ages of students
200
(a) the value of x,
(b) the total number of students in the class,
O
Feb Mar Apr May Jun Jul (c) the number of students who are 13 years old.
Month
Solution
(a) In which months was the balance below $500? (a) x° + 30° + 230° = 360° (∠s at a point)
(b) Can you predict the balance in July? Explain briefly. x = 100
230
Solution (b) Total number of students = 23 ÷
360
(a) The balance was below $500 in the months of = 36
February and March.
(c) Number of students who are 13 years old
100
(b) The balance in July cannot be predicted. This is = 36 ×
360
because the balances in the previous months were = 10
fluctuating and there was no trend.
O
2006 2007 2008 2009 2010 2011
Year 103
Profits ($’000)
102
(a) Describe the trend of mobile phone subscriptions
between 2006 and 2010. 101
(b) Based on the graph, estimate the number of 100
mobile phone subscribers in Singapore in the
year 2011. 0
1 2 3 4
Year
Solution
Fig. 1
(a) The line graph indicates that the mobile subsciptions
increased steadily from 2006 to 2010.
(b) The estimated number of mobile phone subscriptions 115
in the year 2011 is 7.8 million.
Profits ($’000)
110
10. The line graph below shows the blood alcohol 105
concentration (BAC) of Jack over time after having a
glass of wine. 100
Solution
0.02
(a) The profits can be read more accurately in Fig. 1
than in Fig. 2. The variation of the profits seems
O 1 2 3 4 to be greater in Fig. 1.
Time (hours)
(b) A manager would most likely use Fig. 1 to show
the profits. This is because it gives an impression
(a) How many hours after drinking did the BAC reach that the profits grow very fast.
the highest level? What was its value?
(b) What would be the expected BAC 4 hours after
drinking a glass of wine?
209
12. The pie charts below show the proportions of education Solution
expenses in the government operating expenses of Hong (a) No, we cannot conclude that Singapore spent
Kong in 2009/10 and Singapore in 2009/10 respectively. more money on education than Hong Kong. This
is because the area on each chart just indicates the
proportion of expenses on education, not the actual
Education amount spent on education.
x 58.2
(b) (i) x = × 360°
289.0
= 72.5° (correct to 1 d.p.)
8.5174
y = × 360°
32.7584
= 93.6° (correct to 1 d.p.)
Government operating expenses
of Hong Kong in 2009/10 (ii) Education expenses of Hong Kong
= 58.2 billion HKD
58.2
= billion SGD
6.40
Education = 9.09375 billion SGD
y Education expenses of Singapore
= 8.5174 billion SGD
Hong Kong spent more money on education.
(ii) Amount per student in Hong Kong
9.09735 billion SGD
Government operating expenses =
1294500
of Singapore in 2009/10
= 7024.91 billion SGD (correct to 2 d.p.)
(a) Based on the pie charts above, can you conclude Amount per student in Singapore
that Singapore spent more money on education 8.5174 billion SGD
=
than Hong Kong? Explain briefly. 698544
(b) The following table shows some relevant figures. = 12193.08 billion SGD (correct to 2 d.p.)
Hence Singapore spent more money per student on
Hong Kong in Singapore in
education.
2009/10 2009/10
Government
289.0 billion 32.7584 billion 13. The line graph shows the numbers of Singapore residents
operating who were at least 70 years old in the years 1995, 2000,
HKD SGD
expenses 2005 and 2010.
Education 58.2 billion 8.5174 billion
Number of Singapore residents who are at least 70 years old
Expenses HKD SGD
Student
1 294 500 698 544
Enrolment
300
Assume that the exchange rate between Hong Kong
dollars (HKD) and Singapore dollars (SGD) is 1
250
SGD = $6.40 HKD.
Number (thousand)
211
Solution (b) Name a suitable type of statistical graph which can
(a) Betty was the winner of the election. show the proportion of the company staff in each
department.
(b) The symbol used to represent Betty's votes is bigger
(c) Amy wants to represent the data by a line graph.
than those of Alfred and Carmen. It seems that
Do you think it is appropriate to do so? Why?
Betty had much more votes than the others.
Solution
5. The bar graph shows the average monthly incomes of
(a) bar graph
employees in 4 competitive companies.
(b) pie chart
Average monthly incomes of employees
(c) The data does not relate to time. Therefore, it cannot
be represented by a line graph.
5000
Monthly income ($)
2000
A B C D
Number of responds
Company 30
20
(a) Find the ratio of the average monthly income in
company A to that in company B.
10
(b) Find the ratio of the length of the bar representing
the average monthly income in company A to that
in company B. O Excellent Very Satisfactory Not Poor
(c) Ajib said the average monthly income of employees Good Satisfactory
Comments
in Company D is twice that in Company A. Do you
agree with his statement? Explain your answer.
(a) Find the percentage of comments which are
‘Satisfactory’ or better.
Solution
(b) The manager of the hotel said that it is more
(a) The required ratio = $ 3400 : $ 4200
appropriate to represent the data by a pie chart.
= 17: 21
Do you agree with him? Explain your answer.
(b) The required ratio = 7 : 11
Solution
(c) The vertical scale does not start from zero. The
(a) Total number of responses
readers may use the heights of the bars to compare
= 16 + 30 + 20 + 10 + 4
the actual average monthly incomes of different
= 80
companies.
Number of comments which are ‘satisfactory’ or
better
Level 2 = 16 + 30 + 20
6. The table shows the number of staff in different = 66
departments of a company. = 82.5%
(b) Yes, it is better to represent the data by a pie chart.
Department Number of staff We will then be able to compare the proportion of
each comment to the whole survey.
Accounting 4
General 10
Marketing 5
Production 17
42 Solution
(a) The maximum temperature was 33.5 °C and it
40 occurred in February.
(b) The wettest month was July. The rainfall in July
38 was 300 mm.
2007 2008 2009 2010 2011
Year
(c) The graph should have units of measurements on
the two vertical scales. The scales can be read more
(a) Find the percentage increase in the profits from accurately if graph paper is used.
2007 to 2011.
(d) It shows 3 sets of data on one graph. We can
(b) By what percentage is the height of the point
see whether there is any relationship between
representing the profits in 2011 higher than that
temperature and rainfall.
in 2007?
(c) By observing the graphs, Laura said that the profits
in 2011 was three times that in 2007. Do you agree 10. Design a simple survey on the Internet usage of your
with her? Explain your answer. classmates. Present your survey in a report which should
include your questionnaire, the methods used for collecting
Solution and organising data, the statistical diagrams to illustrate
(a) Percentage increase your results, the summary of your findings and the
44 – 40 conclusion.
= × 100%
40
= 10% Solution
Advise students that each survey must have its objective
(b) Required percentage
or goal. Questions may be
3–1
= × 100%
1 (i) two-state
= 200%
(ii) multiple choice, and
(c) Yes, the line graph misleads the readers. The vertical
scale does not start from zero. It seems that the (iii) open-ended types.
profit in 2011 is three times of that in 2007.
Level 3
9. The diagram shows monthly statistics of the maximum
and minimum temperatures, and rainfall recorded at the
climate station located in Changi in the year 2010.
2010 Monthly Statistics (Climate Station)
40 350
38
300
36
34 250
32
200
30
150
28
26 100
24
50
22
20 0
Jan Feb Mar Apr May Jun Jul Aug Sept Oct Nov Dec
Source: h ttp://app2.nea.gov.sg/data/cmsresource/
20110524444301448687.pdf
213
Revision Exercise 16 2. Students in a class were asked about their favourite games.
1. The table shows the number of hybrid vehicles in The results of the survey are shown in the bar graph.
Singapore from 2008 to 2010.
Favourite games of students
Number of students
vehicles
2008 Game
215
7. The line graph shows the exchange rate between (a) The girl’s height was 143 cm when she was
Singapore dollars (SGD) and the US dollars (USD) in 11 years old. Her height increased rapidly in the
SGD/USD at the end of each year from 2002 to 2010. next two years. Then from 13 to 14 years old, she
grew by another 2 cm. At 15 years old, her height
Exchange rate between Singapore and US dollars
was 157 cm and it remained the same for the next
2.0 two years.
1.6
(c) m = 45 + 0.9(h – 150)
1.4
When h = 157,
1.2 m = 45 + 0.9 × (157 – 150)
1 = 51.3
O 2002 2003 2004 2005 2006 2007 2008 2009 2010 2011
Year 9. The bar graph shows the number of friends of each of 4
students in a social networking website.
(a) Describe the general trend of the exchange rate.
Number of friends in a social networking website
(b) Find, correct to the nearest integer, the percentage
change in the exchange rate from the end of 2002
to the end of 2010. 120
(c) Based on the graph, estimate the exchange rate at
the end of the year 2011.
Number of friends
100
Solution
80
(a) The general trend is decreasing. This means the US
dollar is weakening with respect to the Singapore
60
dollar.
(b) Percentage change 40
Alan Bowie Carol Doris
1.3 – 1.75
= × 100% Student
1.75
= –25.7% (correct to 3 s.f.)
(a) Find the ratio of the number of friends of Alan to
(c) The estimated exchange rate at the end of 2011 is that of Doris.
1.24 SGD/USD. (b) Find the ratio of the height of the bar representing
the number of friends of Alan to that of Doris.
8. The line graph shows the heights of a girl at different (c) By observing the graph, Doris said that her number
ages. of friends in the social networking website was
(a) Describe the growth in height of the girl. twice as many as Alan. Do you agree with her?
(b) Based on the graph, estimate the girl’s height when Explain your answer.
she is 18 years old.
(c) A doctor proposed that the ideal mass, m kg, of a Solution
girl with height, h cm, where h 150, is given by (a) Required ratio
the formula m = 45 + 0.9(h – 150). Find the ideal = 80 : 120
mass of the girl when she is 17 years old. =2:3
(b) Required ratio
Solution = 2 : 4
= 1 : 2
Heights of a girl at different ages
(c) Yes, the graph misleads the readers. The vertical
160
scale does not start from zero. The height of the
rectangle is not proportional to the number of
Height (cm)
friends.
150
140
0
11 12 13 14 15 16 17 18
Age (Year)
x > – × 20
1 (b) Area of ABCD = 5 × 10 000 cm2
4 = 50 000 cm2
x > –5
The required negative integers are
–1, –2, –3, –4 and –5. 5. In the figure, the shape is formed by 3 semicircles, where
AB = BC = CD = t cm.
A 4 E B Solution
9 (a) Area of the shape
2 2
3t t 1
Solution = r× + rt 2 – r × ×
2 2 2
(a) AECF is a parallelogram.
r 9t 2 t2
Area of AECF = 4 × 5 = × + t2 –
2 4 4
= 20 cm2 3r
= t 2 cm2
(b) Let h cm be the perpendicular distance from 2
F to EC. 3r
(b) (i) t 2 = 20
5 2 × h = 20 2
1 40
h = 20 × t2 = 3r
5 2
4 t= 40
= 3r
2
= 2.83 (correct to 3 s.f.) t = 2.06 (correct to 3 s.f.)
\ The required distance is 2.83 cm.
217
(ii) Perimeter of the shape (c) Let r cm be the new radius.
3t t rr 2 × 12 = 72r
=r× +r×t+r× 2
2 r 2 = 6
= 3rt r = 6
= 3r × 40
= 2.45 (correct to 3 s.f.)
3r
= 19.4 cm (correct to 3 s.f.) \ The new radius is 2.45 cm.
6. The figure shows a L-shaped prism. AB = 10 cm, 8. The pictogram shows the mean sulphur dioxide level in
BC = 3 cm, CD = 6 cm, FA = 8 cm and BG = 13 cm. the air in Singapore from March 2011 to June 2011.
Find
(a) the total surface area of the prism, March 2011
(b) the volume of the prism.
K J April 2011
May 2011
I H
F E June 2011
G
8 6 C represents 2 ng/m3
D 13
3
A 10 B (a) State the mean sulphur dioxide level in April
2011.
(b) In which month was the air most polluted by sulphur
Solution
dioxide during these four months?
(a) Area of the cross-section ABCDEF
=8×4+6×3 Solution
= 50 cm2 (a) Mean sulphur dioxide level in April 2011
Perimeter of ABCDEF 1
= 10 + 3 + 6 + 5 + 4 + 8 =2×4
2
= 36 cm = 9 µg/m3
\ Total surface area of the prism
= 36 × 13 + 2 × 50 (b) It is most polluted in May.
= 568 cm2
(b) Volume of the prism 9. The bar graph shows the sales of magazines A, B and C
= 50 × 13 in a bookstall on a day.
= 650 cm3 Sales of magazines on a day
Review Exercise 4
218
Solution 11. In the figure, ABCD is a trapezium and AECD is
(a) Total number of magazines sold a parallelogram. The ratio of the area of AECD to
= 9 + 15 + 12 the area of ABCD is 6 : 11. The area of BCE is
= 36 40 cm2.
The required percentage (a) Find the area of ABCD.
15 + 12 (b) If EB = 10 cm, find the height from D to AB and
= × 100%
36 the length of AE.
= 75%
(b) The required angle D C
9
= 360° ×
36
= 90°
10. The line graph shows the total international call minutes A E B
of Singapore from 2006 to 2010.
Solution
Total international call minutes of Singapore (a) Let 11x cm2 be the area of ABCD.
Then the area of AECD = 6x cm2
11x – 6x = 40
14 000 5x = 40
x = 8
12 000 \ Area of ABCD = 11 × 8
Total international call minutes (million)
= 88 cm2
10 000 (b) (i) Let h cm be the height from D to AB.
1
× 10 × h = 40
2
8000
5h = 40
h = 8
6000 \ The height is 8 cm.
(ii) Let y cm be the length of AE.
4000 By considering the area of AECD,
y × 8 = 88 – 40
2000 8y = 48
y = 6
\ AE = 6 cm.
O 2006 2007 2008 2009 2010 2011
Year 12. The figure shows the cross-section of a prism of length
15 cm. It is a semicircle of diameter 20 cm surmounted
(a) Describe the general trend of the total international on a rectangle of width 20 cm and height h cm. Given
call minutes from Singapore from 2006 to 2010. that h is the smallest integer such that the area of the
(b) Find the total international call minutes from rectangle is larger than the area of the semicircle, find,
Singapore in 2009 from the graph. in terms of r,
(c) Estimate the total international call minutes from (a) the value of h,
Singapore in 2011 from the graph. (b) the total surface area of the prism,
(c) the volume of the prism.
Solution
(a) The general trend is increasing from 2006 to 2010.
(b) The reading = 10 000 million minutes.
(c) The estimated total international call from Singapore
in 2011 is 14 400 million minutes.
h
20
219
Solution (b) The time taken
(a) Area of the rectangle = Area of the semicircle = 37500r ÷ 2000
1 = 58.9 s (correct to 3 s.f.)
20 × h = × r × 102
2
(c) Let h cm be the length of the trough.
20h = 50r
Area of PQRS
5r
\ h = 1
2 = × (60 + 18) × 20
2
(b) Area of the cross-section = 780 cm2
= r × 102 780 × h = 37500r
= 100r cm2 h = 151.0
Perimeter of the cross-section \ h = 151 (correct to 3 s.f.)
5r \ The length of the trough is 151 cm.
= 20 + 2 × + r × 10
2
(d) External surface area of the trough
= (20 + 15r) cm
= (18 + 29 + 29) × 151.0 + 2 × 780
\ Total surface area = 13 000 cm2 (correct to 3 s.f.)
= (20 + 15r) × 15 + 2 × 100r
= 300 + 225r + 200r
14. The line graphs below show the profits of two companies,
= (300 + 425r) cm2
A and B, from 2010 to 2012.
(c) Volume of prism
= 100r × 15 Profits of company A
= 1500r cm3
3.9
60
60 P S 0 2010 2011 2012
29 20 29
25 Year
Q 18 R
Figure A Figure B
Profits of company B
The tank is full of water. The water is then drained from
the tank to the trough at the rate of 2000 cm3 per second
and it just completely fills the trough.
7
(a) Express the initial volume of water in the tank in
Profits (million $)
Review Exercise 4
220
(b) Find the percentage increase in the profits of (b) The area of the HDB flat increases as the household
company B from 2010 to 2012. size increases.
(c) Explain why the line graphs give the impression (c) The average household size decreases for all
that the financial performance of company A is different types of dwelling from the year 2000 to
better than that of company B. 2010.
Solution
(a) (i) Percentage change
3.96 – 4.0
= × 100%
4.0
= –1%
(ii) Percentage change
4.2 – 4.0
= × 100%
4.0
= 5%
(b) Percentage increase
6–5
= × 100%
5
= 20%
(c) The vertical scales of two graphs are different. The
graph of company A exaggerates the change.
15. The bar graph shows the average household size by the
type of dwelling in the years 2000 and 2010.
Average household size by type of dwelling
2000
5 4.7
2010
4.4
Household size (persons)
4.1 4.0
4 3.9
3.7 3.7
3.4
3.1
3 2.8
2.2 2.1
2
Solution
(a) Percentage change
2.8 – 3.1
= × 100%
3.1
= –9.86% (correct to 3 s.f.)
221
Problems in Real-world Contexts
A. Rate and Discount in a Supermarket
The advertisement on the right shows the price of fresh tiger jumbo prawns in a promotion.
1. What is the promotional price of the tiger prawns in dollars per kg?
2. Find the percentage discount in this promotion, giving your answer correct to 3
significant figures.
3. Sally bought 1.5 kg of the prawns. How much did she save from the usual price?
Solution
1. Promotional price in dollars per kg
= $3.68 per 100 g
= $36.80 per 1000 g
= $36.80 per kg
2. Percentage discount
4.29 – 3.68
= × 100%
4.29
= 14.2% (correct to 3 s.f.)
3. Amount of savings
= $1.5(4.29 – 3.68)
= $0.92 (correct to 3 s.f.)
B. Exchange Rate
The following table shows the currency exchange rates at 5:08 p.m. on 9th April 2012.
Source: http://sg.finance.yahoo.com/currencies/investing.html#cross-rates
1. Write down two statements about the exchange rate between Singapore dollars (SGD) and Euros (EUR) from the
table.
2. A cell of the table, as shown above with a , has been accidentally masked by a correction tape. Could you
find out its value? Give your answer correct to 4 decimal places.
3. Su Ling went for a tour in Australia. If she exchanged 3000 Singapore dollars, how many Australian dollars would
she get?
4. Ramesh exchanges 1000 Singapore dollars to a currency A, then to another currency B, and finally exchanging the
money back to Singapore dollars according to the exchange rates in the table above. He thinks that he will get back
1000 Singapore dollars. Do you agree with him? Justify your answer.
Solution
1. 1 EUR = 1.6488 SGD
1 SGD = 0.6065 EUR
2. 1 USD = 1.2610 SGD
1 SGD = 0.7930 (correct to 4 d.p.)
The value in the cell is 0.7930.
1. At the end of 2012, what is the number of completed shoebox units in the suburban area? Calculate this as a
percentage of the total number of completed units in the same year.
2. At the end of which year do you see the largest increase in the number of completed shoebox units?
3. (a) Describe the trend of the number of completed shoebox units from the end of 2011 to the end of 2015.
(b) Could you predict the number of completed shoebox units at the end of year 2016? Explain how you arrive
at your prediction.
Solution
1. Number of completed shoebox units at the end of 2012 = 900
900
Required percentage = × 100%
4100
= 22.0% (correct to 3 s.f.)
223
2.
End 2012 End 2013 End 2014 End 2015
Increase 4100 – 2400 = 1700 5400 – 4100 = 1300 6800 – 5400 = 1400 8200 – 6800 = 1400
1700 1300 1400 1400
Percentage × 100% × 100% × 100% × 100%
2400 4100 5400 6800
Increase
= 70.8% = 31.7% = 25.9% = 20.6%
The largest increase occurred at the end of 2012.
3. (a) There is an increase in the number of shoebox units. The largest increase was at the end of 2012 and from the
end of 2013 onwards, the number of shoebox units decreased.
(b) Increment from end 2013 to end 2014 = 1400
Increment from end 2014 to end 2015 = 1400
We may estimate that the increment from end 2015 to end 2016 to be 1400.
\ estimated number of completed shoebox units at the end of 2016 = 9600
D. Paper Clip
The diagram shows a paper clip which is made from a circular wire of diameter 1 mm. The dimensions are in centimetres.
The clip consists of 3 semicircles and 4 straight edges.
Suppose you are the manufacturer of the paper clip and you would like to know more about the material required to make
the clip.
1. Calculate the total length of the wire needed to make a clip. State the assumptions you have to make in your
calculation.
2. What is the volume (in cm3) of the material used for making each clip?
3. The clips are packed in a box of 200 pieces. It is known that the density of the material is 7.6 grams/cm3. Find the
total mass of the clips in a box.
4. The wire of the clip is then bent into the following shapes:
(i) a square
(ii) a rectangle
(iii) an equilateral triangle
(iv) a circle
(a) Find a possible set of dimension(s) for each shape.
(b) Find the area of each shape.
(c) Given the same perimeter (in this case, the length of clip is fixed), which shape gives the largest area?
Solution
1. Assume that all 3 semicircles are of diameter 1 cm.
Total length of wire = 3 × r × 0.5 + 4 + 3 + 3 + 2
= 16.71 (correct to 4 s.f.)
= 16.7 cm (correct to 3 s.f.)
2. Volume = r × 0.052 × 16.71
= 0.131 cm3 (correct to 4 s.f.)
= 0.13 cm3 (correct to 3 s.f.)
3. Total mass = 7.6 × 0.13 × 200
= 197.6 g (correct to 3 s.f.)
(c) From the table above, the circle gives the largest area.
1. Represent the data above using a graph, including the gold, silver and bronze medals won by the countries.
Hint: You may use a spreadsheet program to help you choose the appropriate graph.
2. How would you compare the results?
3. Besides ranking by gold medals, what other ways would you suggest ranking the countries? Will the ranking be
different? Which way of ranking do you think is most reasonable? Explain your answer.
Solution
50
40
30
20
10
0
ina tes ssi
a in any ali
a rea an ly nce
Ch Sta Ru rita rm str Ko Jap Ita Fra
i t ed ea tB Ge Au u th
Un Gr So
225
2. The countries may be ranked by the number of gold medals won.
3. Since countries may rank differently for each type of medal, ranking countries by the total number of medals is most
reasonable.
Local calls will be charged at 16 cents per minute during day time and 8 cents per minute during
Company A
night time.
Local calls will be charged at 22 cents for the first minute and 8 cents per minute thereafter for the
Company B
whole day.
Your friend from the United States is visiting Singapore for 7 days. He would like you to suggest a phone plan to purchase
during his visit.
1. What would you need to consider when helping your friend decide on which phone plan to purchase?
2. Assume that he makes 10 phone calls in the day and 5 phone calls at night for each day, and each call lasts about
3 minutes. Calculate his daily call charges per day for each plan.
3. In general, he makes x phone calls in the day and y phone calls at night for each day, and each phone call lasts about
t minutes. Express the daily charges in terms of x, y and t for each plan.
4. Use the two formulae obtained in Question 3 to decide on which phone plan to purchase. Justify your decision made.
Solution
1. Company A Company B
Daily call charge = $ 0.16 × 10 × 3 + $0.08 × 5 × 3 Daily call charge = $0.22 × 15 + $0.08 × 15 × 2
= $4.80 + $1.20 = $3.30 + $2.40
= $6.00 = $5.70
2. Company A Company B
Daily call charge = $0.16 × x × t + $0.08 × y × t Daily call charge = $0.22 × (x + y) +
= $0.16xt + $0.08yt $0.08 × (x + y) × (t – 1)
= $0.14x + $0.14y +
$0.08xt + $0.08yt
4. Assume the caller does not make any calls during the day, i.e. x = 0.
Company A: $0.08yt
Company B: $0.14y + $0.08yt
Company A offers a better phone plan.
Assume the caller does not make any calls during the night, i.e. y = 0.
Company A: $0.16xt
Company B: $0.14x + $0.08xt
= x(0.16t – 0.14 – 0.08t)
Difference
= x(0.08t – 0.14)
If 0.08t – 0.14 = 0,
0.08t = 0.14
t = 1.75
If the phone call lasts for less than 1.75 min, Company A offers a better phone plan. If the phone call lasts for more
than 1.75 min, Company B offers a better phone plan.
Year Population
2001 4 138 000
2002 4 176 000
2003 4 114 800
2004 4 166 700
2005 4 265 800
2006 4 401 400
2007 4 588 600
2008 4 839 400
2009 4 987 600
2010 5 076 700
The table above shows the population of Singapore from 2001 to 2010.
1. Represent the data above using a graph.
Hint: You may use a spreadsheet program to help you plot and draw the graph.
2. Describe the trend of population growth of Singapore over these 10 years.
3. Based on your description in Question 2, estimate the population in the year 2011.
4. Could you predict the population of Singapore in the year 2020 using the data above? What are some possible events
or happenings you would need to consider as part of your assumptions when you make your prediction?
Solution
1.
Population of Singapore
6000000
5500000
5000000
4500000
4000000
Population
3500000
3000000
2500000
2000000
1500000
1000000
500000
0
2001 2002 2003 2004 2005 2006 2007 2008 2009 2010 2011
2. The graph shows that the population of Singapore increased quite steadily from 2001 to 2011 except for a decrease
from 2002 to 2003. The rate of increase was faster from 2007 to 2008.
3. We can project the line graph to the year 2011 to get an estimate as shown in the graph.
\ estimate population in 2011 = 5.2 million
4. The population for the year 2020 can be predicted if the rate of population increase remains the same from 2010 to
2020
227
H. Fund Raising
Your school is staging a carnival to raise funds on a Saturday from 10 a.m. to 5 p.m. . Your class has decided to set up a
stall to sell hamburgers. You are tasked to come up with a plan for the number of hamburgers to be sold and the price of
each hamburger. Your target is to raise as much money as possible for the school.
The following are points that you may consider to help you in your planning:
• W here and how are you going to purchase your hamburgers (bread and filling)? How much money would you
need? You may try to get somebody to sponsor your bread and filling.
• How many hamburgers do you expect to sell per hour? How many hamburgers would you need to prepare during
the seven hours of the carnival?
• You may have a target amount of money you would like to raise. After considering the expected cost prices that
you would incur, would you be able to achieve your target?
Solution
1. Other information to consider would be
• the costs of the bread and filling
• how much each burger costs to make
• the price of each burger to be sold
• target amount of money to be raised
• the estimated number of people coming for the carnival
• the number of people needed to prepare the burger
• giving discounts
2. Assume that all burgers would be sold.
3. Answer depends on method chosen.
4. Answer depends on method chosen.